Nephsap Article Pi - I 3
Nephsap Article Pi - I 3
Nephsap Article Pi - I 3
Guest Editor:
Michael Emmett, MD
Editorial Director:
Alice M. Sheridan, MD, FASN
Deputy Editor:
Martina M. McGrath, MBChB, FASN
nephSAP
EDITORIAL DIRECTOR, nephSAP Preface
Alice M. Sheridan, MD, FASN Launched in 2002, the Nephrology Self-Assessment Program (nephSAP®) is one of the premier educational
Brigham and Women’s Hospital activities of the American Society of Nephrology (ASN) Alliance for Kidney Health. nephSAP focuses on
Boston, MA
self-assessment and education and provides Continuing Medical Education (CME) credits and Maintenance
DEPUTY EDITOR, nephSAP of Certification (MOC) points for individuals certified by the American Board of Internal Medicine (ABIM).
Martina M. McGrath, MBChB, FASN Starting with Volume 19 (2020), nephSAP content is available electronically through the nephSAP website
Brigham and Women’s Hospital (nephsap.org). Content for Volume 18 and earlier, and all archived nephSAP content, are available on the
Boston, MA main ASN website (asn-online.org/nephsap).
GUEST EDITOR PROGRAM MISSION AND OBJECTIVES: nephSAP® provides a learning vehicle for physicians,
Michael Emmett, MD scientists, advanced practitioners, nurses, pharmacists, and other health professionals working and training in
Baylor University Medical Center the field of nephrology to renew and refresh their clinical knowledge, diagnostic, and therapeutic skills. This
Dallas, TX enduring material provides learners challenging, clinically-oriented questions based on case vignettes, detailed
learning objectives, a series of articles that review recent publications, and an editorial on an important
ASSOCIATE DIRECTOR OF and evolving topic. This combination of materials enables learners to rigorously assess their strengths and
ACCREDITATION AND EVALUATION weaknesses in the broad domain of nephrology.
Suzanne P. Armstrong, MA
EDUCATION: nephSAP® supports continuous improvement and lifelong learning by helping busy clinicians
American Society of Nephrology
and trainees at all levels stay current with the latest scientific and medical advances in kidney care. Each issue
Washington, DC
of nephSAP is dedicated to a specific theme, i.e., to a specific area of clinical nephrology, hypertension,
PROGRAM MANAGER OF dialysis, and transplantation, and consists of an editorial, topical articles, and self-assessment questions. All
SELF-ASSESSMENT clinically relevant and key elements of nephrology are reviewed and updated every two years. The authors of
Lisa Netha Xayavong, MA each issue digest and interpret key studies published since the release of the previous issue and integrate this
American Society of Nephrology new material with the body of existing information. Occasionally a special edition is produced to cover an
Washington, DC area not ordinarily addressed by core issues of nephSAP.
PAST EDITORS SELF-ASSESSMENT: An examination with thirty, single-best-answer questions follow the 60 to 100
Jeffrey S. Berns, MD, FASN pages of topical articles. This examination is available online with immediate feedback which includes answers
Ronald J. Falk, MD, FASN to all questions, brief discussions, and an updated bibliography.
Stanley Goldfarb, MD, PhD
Gerald A. Hladik, MD, FASN CME and MOC: Those answering 75% correctly can claim up to 10 CE credits and MOC points per issue.
Raymond R. Townsend, MD
Jerry Yee, MD, FASN
Send Your Feedback nephsap@asn-online.org nephSAP® ©2024 by The American Society of Nephrology
nephSAP Volume 23 4 Number 2 4 June 2024
Hypernatremia from Renal Sodium Gain .......................... 103 Polyuria ................................................................... 118
Hypernatremia from Renal Water Loss ............................. 104 Renal Tubular Acidosis ........................................... 118
Evaluation of Hypernatremia ................................................ 104 Salt-Wasting Tubulopathies ................................... 118
Arginine Vasopressin–Related Polyuria ................................ 104 Hypomagnesemia ................................................... 118
Diagnosis of Arginine Vasopressin–Related Polyuria ......... 105 Very Low-Calorie Diets .......................................... 118
Copeptin-Based Testing Approaches to Arginine Lupus Nephritis ...................................................... 118
Vasopressin–Related Polyuria ........................................... 107
Impact of Hypokalemia on Patient Health and Outcome .... 119
Treatment of Arginine Vasopressin–Related Polyuria ........ 107
Diagnosis and Management of Hypokalemia ....................... 120
Treatment of Hypernatremia from Water Loss ..................... 108
Diagnostic Approach to Hypokalemic Patients ................. 120
Treatment of Hypernatremia from Sodium Gain ................. 108
Management of Hypokalemic Patients .............................. 120
Hypernatremia in the ICU ..................................................... 108
References .............................................................................. 122
Permissive Hypernatremia ................................................ 109
Conclusion ............................................................................. 110
Updates in the Risk and Treatment of Hyperkalemia in
References ............................................................................... 110 CKD Patients ........................................................................ 125
Alice M. Sheridan
Pathophysiology, Evaluation, and Treatment of
Hypokalemia ......................................................................... 113 Learning Objectives ............................................................... 125
Gregor Lindner, Christoph Schwarz, and Svenja Ravioli
Introduction .......................................................................... 125
Learning Objectives ............................................................... 113 Pathophysiology .................................................................... 125
Introduction .......................................................................... 113 Cellular Shift .................................................................... 125
Potassium Content and Distribution .................................... 113 Decreased Excretion ......................................................... 125
Physiology of Potassium Homeostasis .................................. 113 Clinical Outcomes and Approach to Therapy ........ 126
Physiologic Adaptions to Potassium Depletion .................. 113 Treatment and Prevention of Recurrence .......................... 126
Cellular Shifting ...................................................... 114 Monitoring ....................................................................... 127
Potassium Handling by the Kidney ........................ 114 Conclusion ............................................................................. 127
Physiology of Potassium Supplementation ............ 114 References .............................................................................. 127
Etiology of Hypokalemia ....................................................... 116
Decreased Potassium Intake .............................................. 116 Metabolic Acidosis ................................................................ 128
Mengyao Tang and Andrew Z. Fenves
Cellular Shift ........................................................... 116
Extrarenal Potassium Loss ................................................. 117 Learning Objectives ............................................................... 128
Increased Urinary Loss ...................................................... 117 Introduction .......................................................................... 128
Diuretics .................................................................. 117 Respiratory Compensation .................................................... 128
Primary Increased Mineralocorticoid Activity ....... 117 Renal Response ...................................................................... 128
Nonreabsorbable Anions ........................................ 118 The Anion Gap and d AG/d HCO3 Ratio .............................. 129
Vomiting ................................................................. 118 High Anion Gap Metabolic Acidosis ..................................... 129
nephSAP Volume 23 4 Number 2 4 June 2024
The nephSAP editorial team and the KSAP editorial board extend their sincere appreciation to the following
reviewers. Their efforts and insights help improve the quality of these postgraduate education offerings.
Learning Objectives
1. Examine recent scientific advances in our understanding of the pathophysiology of disorders of fluid, electrolytes, and
acid-base balance.
2. Describe how the pathophysiology relates to the care of patients.
3. Analyze how recent studies related to fluid, electrolyte, and acid-base disorders can be applied to clinical decision making.
Target Audience:
Physicians, scientists, advanced practitioners, nurses, pharmacists, and other health professionals working and training in the
field of nephrology.
Accreditation Statement
In support of improving patient care, the American Society of Nephrology is jointly accredited by the
Accreditation Council for Continuing Medical Education (ACCME), the Accreditation Council for
Pharmacy Education (ACPE), and the American Nurses Credentialing Center (ANCC), to provide
continuing education for the healthcare team.
AMA Credit Designation Statement – Physicians: The American Society of Nephrology designates this
TM
enduring material for a maximum of 10.00 AMA PRA Category 1 Credits . Physicians should claim only the credit commensurate
with the extent of their participation in the activity.
Self-Assessment Examination
4 Answers with explanations are provided with a passing score after the first and/or second attempt.
4 This issue will be archived on June 1, 2026, and answers with explanations will be posted on the ASN website.
nephSAP Volume 23 4 Number 2 4 June 2024
Volume 19 4 Number 4 October 2020
Method of Participation
4 Read the entire issue that is supplemented by original articles in the reference lists.
4 Complete the online self-assessment examination.
4 Each participant is allowed two attempts to pass the examination (75% correct) for CME and MOC.
4 Upon completion, review your score and incorrect answers and print your certificate.
System Requirements
Compatible Browser and Software
The nephSAP website (nephsap.org) is formatted for cross-browser functionality and should display correctly in all currently
supported web browsers. Individual nephSAP articles are available as dynamic HTML or in Portable Document Format (PDF),
which requires Adobe Reader or comparable PDF viewing software.
Monitor Settings
The nephSAP website was designed to be viewed in a 1024 768 or higher resolution.
Medium or Combination of Media Used
The media used include content provided online as dynamic HTML or downloadable PDF, in addition to an online evaluation
and self-assessment examination.
Technical Support
If you have difficulty viewing any of the pages, please refer to the nephSAP technical support page for possible solutions. If you
continue having problems, contact ASN at nephsap@asn-online.org.
nephSAP Volume 23 4 Number 2 4 June 2024
Volume 19 4 Number 4 October 2020
EDITORS:
Alice M. Sheridan, MD, FASN—Employer(s): Brigham and Women's Hospital; No relevant financial relationship(s) with ineligible companies to disclose.
Martina M. McGrath, MBChB, FASN—Employer(s): Brigham and Women’s Hospital; VA Boston Medical Center; No relevant financial relationship(s) with
ineligible companies to disclose.
EDITORIAL AUTHOR:
Michael Emmett, MD—Employer(s): Baylor University Medical Center; Other Interests: I receive payment for my work as the editor of the Acid-Base and Electrolyte
related chapters in “UpToDate”.
AUTHORS:
Anand Achanti, MD—Employer(s): Medical University of South Carolina; Research Funding: Omeros- PI - IgA trial; Cure GN- PI; Kaneka Medical America LLC - sub-I
LDL apheresis for FSGS; Advisory or Membership Role: Travere Therapeutics: Expert Advisory Committee on IgA management; Alexion: Expert Advisory Committee
on TMA management.
Waleed A. Elsheikh, MD, MBBS—Employer(s): Medical University of South Carolina. No relevant financial relationships with ineligible companies to disclose.
Andrew Z. Fenves, MD, FASN—Employer(s): Massachusetts General Hospital and Harvard Medical School; Consultancy: Weber Gallagher & Associates Law Firm; Shook,
Hardy and Bacon Law Firm for expert testimony; Ownership Interest: Apple, Alphabet; Honoraria: UpToDate - royalties; Advance-Medical, Teladoc - expert opinion.
Alexandra Grand, MD—Employer(s): Baylor College of Medicine; Research Funding: Otsuka Pharmaceutical Co., Ltd.
Gregor Lindner, MD, MBA—Employer(s): Johannes Kepler University; Consultancy: Otsuka; Honoraria: Daiichi-Sankyo; Bayer; Otsuka; Advisory or Membership
Role: Otsuka.
Svenja Ravioli, MD—Employer(s): Johannes Kepler University; Honoraria: Otsuka.
Joshua Rein, DO, FASN—Employer(s): Icahn School of Medicine at Mount Sinai Hospital; James J. Peters Veterans Affairs Medical Center; Advisory or Member-
ship Role: Editorial Board - Kidney Medicine, International Journal of Nephrology; Co-Chair, Organizing Committee - Basic Research Forum for Emerging Kidney
Scientists.
Christoph Schwarz, MD—Employer(s): Pyhrn-Eisenwurzenklinikum Steyr; Honoraria: Otsuka.
Alice M. Sheridan, MD, FASN—Employer(s): Brigham and Women’s Hospital; Advisory or Membership Role: ASN nephSAP, Editorial Director.
Mengyao Tang, MD, MPH, FASN—Employer(s): Massachusetts General Hospital and Harvard Medical School; Research Funding: American Heart Association
(23POST1010825); DexCom (IIS-2023-007).
Bryan M. Tucker, DO, MS, FASN—Employer(s): Baylor College of Medicine. No relevant financial relationships with ineligible companies to disclose.
Madhusudan Vijayan, MBBS—Employer(s): Icahn School of Medicine at Mount Sinai Hospital. No relevant financial relationships with ineligible companies to
disclose.
ASN STAFF: Suzanne P. Armstrong, MA—Employer(s): American Society of Nephrology; No relevant financial relationship(s) with ineligible companies to disclose.
Lisa Netha Xayavong, MA—Employer(s): American Society of Nephrology. No relevant financial relationship(s) with ineligible companies to disclose.
Commercial Support
There is no commercial support for this issue.
Nephrology Self-Assessment Program - Vol 23, No 2, June 2024 81
Guest Editorial
Cystic Fibrosis and Hypoelectrolytemia with Metabolic Alkalosis
Michael Emmett, MD
Department of Medicine, Baylor University Medical Center, Dallas, Texas
Children with cystic fibrosis (CF) were probably recognized in the electrolyte profile of the sub-acute cases was: [Na] 124 mEq/L; [K]
Middle Ages when careful observers noted that if a child tasted salty 2.8 mEq/L; [Cl] 70 mEq/L; and [HCO3] 37 mEq/L. In many
when kissed, it was “hexed” and would likely die at a young age. reports the disorder was called “Pseudo-Bartter syndrome,” but i
From the early to mid-20th century, clinical and research efforts believe a much better term, proposed by Terlizzi et al., is
focused on gastrointestinal (GI) manifestations of pancreatic insuffi- “hypoelectrolytemia with metabolic alkalosis” (HMA) (10). Bartter
ciency, steatorrhea, and obstructive intestinal disease (meconium syndrome is an autosomal recessive disorder that results in a severe
ileus), and the disorder was named “fibrocystic disease of the impairment of renal tubule NaCl reabsorption and is therefore char-
pancreas.” Through the 1940s, CF was often lumped together with acterized by inappropriate renal NaCl wasting. In contrast, the elec-
several other chronic GI conditions, especially the disorder we now trolyte disorder and metabolic alkalosis that develops in the patients
recognize as celiac disease. Pulmonary pathology became increasingly with CF is characterized by very low urine [Cl]. True Pseudo-
emphasized by the late 1940s. The major center that dramatically Bartter syndrome may be caused by surreptitious ingestion of loop
advanced our current understanding of this disease was Columbia diuretics or by drugs such as aminoglycosides, which can damage
University’s Babies Hospital, where Dorothy Andersen and her distal tubules and thereby result in salt wasting by the kidney.
junior associate, Dr. Paul Di Sant’Agnese, made a number of semi- Since that review, at least 20 case reports and nine large series
nal clinical and pathologic observations (1). In 1983, Quinton of this disorder confirmed these findings. In the larger series, the
reported that a specific defect in chloride reabsorption existed in CF prevalence of HMA ranged from a high of 10% of infants with CF
(1). Then in 1989 Lap-Chjee Tsui working with Francis Collins in Turkey (11) to 1.6% in Italy (10). All the large series and most
identified the gene responsible for CF and showed that mutations case reports indicate that children with HMA are diagnosed at a
impaired the function of its product, the CF transmembrane con- younger age and usually have milder clinical pulmonary and gastro-
ductance regulator (CFTR) (2). intestinal (GI) disease than the children with traditional CF presen-
tations. The CF-generating mutation F508del, which is most
common in the United States and Europe, is less common in the
Infants and Children with Cystic Fibrosis and children with HMA and, when detected, is rarely homozygous (10).
Hypoelectrolytemia with Metabolic Alkalosis Again, whenever reported, urine electrolytes almost always reveal a
very low, or undetectable, [Cl2] and relatively high [K1].
During an unusually severe New York City summer heat wave in
1948, Andersen and her colleagues observed that five of ten infants
admitted with severe heat-related illness had fibrocystic disease of Adults with Cystic Fibrosis with Metabolic Alkalosis
the pancreas (as cystic fibrosis [CF] was then named) and suggested Smith et al., reported the first case of a previously undiagnosed adult
that infants with CF may be especially sensitive to heat (3). Of note, with CF-related hypoelectrolytemia with metabolic alkalosis (HMA)
two of the five infants with CF had marked metabolic alkalosis and in 1995 (12). A 24-year-old, previously “healthy” English infantry-
hypokalemia. Following this observation, Di Sant’Agnese investi- man, assigned to a Saudi Arabia military post, developed recurrent
gated why children with CF were especially susceptible to heat stress. “heat-related collapse” during hot summer months. Serum electro-
He quantitatively analyzed their sweat composition and discovered these lytes were: [Na1] 116 mEq/L; [K1] 2.7 mEq/L; and [Cl2] 57
patients had abnormally high sweat [Na1] and [Cl2], which made mEq/L. Arterial blood gas (ABG) and [HCO32] were not reported,
them very sensitive to a hot environment (4,5). So, Di Sant’Agnese but he surely had severe metabolic alkalosis. CF diagnosis was then
finally confirmed the old Middle Age observation! established (sweat: [Na1] 81 mEq/L; [Cl2] 102 mEq/L). Genetic
Over the next few years, the development of hyponatremia, studies showed F508del and R117H cystic fibrosis transmembrane
marked hypokalemia, and metabolic alkalosis was again noted in conductance regulator (CFTR) mutations. Pancreatic and lung
some children with CF (6,7). In 1971, Gottlieb emphasized that function were normal, but he was azoospermic. Two years later,
metabolic alkalosis was a complication of CF (8). Over the next four Bates et al., reported a remarkably similar case from Texas (13). A
decades, hundreds of case reports and several small series of this 17-year-old, “healthy” man fainted during July. He was volume
association were published. Scurati-Manzoni et al., systematically depleted; chemistries revealed: [Na1] 135 mEq/L; [K1] 2.2 mEq/L;
reviewed all published cases of this association through August 2013 [Cl2] 75 mEq/L; [HCO32] 40 mEq/L; BUN 56; and Cr 2.3 mg/dl.
and identified 172 acute or subacute and 90 chronic cases (9). Most Urine studies revealed: [Na1] 23 mEq/L; [Cl2] 12 mEq/L; and
were children younger than 2.5 years old, and, in the majority, a [K1] 44.9 mEq/L. Sweat [Cl] were high: 67 and 71 mEq/L. Of
diagnosis of CF had not been considered prior to presentation with note, genetic studies again showed F508del and R117H mutations—
the electrolyte disorder. The male/female ratio was 1.6. Average identical to the Smith report. Subsequently, thirteen very similar
intracellular HCO32 and H1. Type A ICs are the major proton-
secreting cells, type B ICs are the major HCO32 secreting cells, and
non–type A-non–type B ICs secrete H1 and/or HCO32. Systemic
metabolic acidosis increases the density and activity of type A ICs
while metabolic alkalosis increases type B IC density and activity.
Recently, ICs have also been identified as major participants in ECF
volume and BP regulation, as described below.
Figure 1 shows that type A ICs actively secrete H1 via V-type
H ATPase and, to a smaller extent, via H1-K1 ATPase. Cytoplas-
1
secretion of HCl generates an equimolar quantity of HCO32 in the reabsorption by these cells, which have a mechanism of NaCl reab-
ECF. The acid load moving down the GI tract stimulates antral D sorption that is independent of K1 secretion. To the extent that dis-
cells to release somatostatin, slowing gastric HCl secretion, and duo- tal renal NaCl reabsorption shifts to principal cells, renal K loss is
denal S cells to secrete secretin, which blunts gastric acid secretion magnified. Almost every report of urine electrolyte analysis in
and also stimulates HCO32-rich pancreatic and biliary secretions. patients with CF related HMA reveals high urine [K1] despite
Secretion of HCO32 into the GI lumen generates an equimolar marked hypokalemia. Type A ICs have the capacity to exchange H1
quantity of ECF H1. If the quantity of pancreatic and biliary for K1 and thereby participate in K1 conservation. Although not
HCO32 secretion equals the quantity of gastric H1 secretion, acid- yet studied or reported, it is possible that this exchange is also
base balance ensues. However, in the interval between gastric HCl reduced in CF patients, especially those with metabolic alkalosis.
secretion and pancreatic and biliary HCO32 secretion, systemic Defective type B IC activity also reduces or blocks distal
[HCO32] and pH increase, and, during this interval, the kidneys HCO32 secretion and thereby contributes to the maintenance of
normally respond by increasing HCO32 excretion. This sequence of metabolic alkalosis.
events is called the “alkaline tide.” It had been assumed that the
renal HCO32 excretory response was largely, or entirely, generated HCO32 Generation in CF
by the short-lived increment in serum HCO32 concentration and 1. Loss of large-volume, NaCl- and K-rich sweat, generates ECF
pH. Berg et al., have now shown that a major component of this volume contraction, secondary hyperaldosteronism, and
renal response is generated by the increment in systemic secretin hypokalemia.
levels and stimulation of a secretin receptor in the basolateral mem- 2. K1 shifts out of cells in exchange for H1 moving into cells,
brane of type B ICs (31,35). Secretin activation of these receptors generating ECF HCO32.
3. Hypokalemia and intracellular acidosis increases renal NH41
markedly increases intracellular cAMP levels (38), and this, in turn,
excretion, generating additional HCO32.
increases the activity and abundance of pendrin (31,34,39). There- 4. A minority of patients with HMA have also been vomiting,
fore, the urinary postprandial alkaline tide not only reflects a sys- and this may also contribute to the development of metabolic
temic increase in [HCO32] and pH but is also a response to alkalosis.
secretin stimulation of type B ICs. Berg et al., have also hypothe- 5. Systemic HCO32 generated by gastric HCl secretion, which is
sized that elevated secretin levels might be a general hormonal not offset by an equimolar quantity of pancreatic or biliary
response to metabolic alkalosis and alkalemia (35). H1 generation, plus possible loss of the normal postprandial
There have also been fascinating discoveries about anion urine alkaline tide probably also contributes to metabolic
exchanger AE4 (SLC4A9) in type B ICs. As noted, this transporter alkalosis generation.
can move HCO32 out of the cell together with either Na1 or K1
or can exchange HCO32 for Cl2. Recently, Vitzthum et al., discov- Maintenance of Metabolic Alkalosis
ered that this exchanger also functions as a systemic pH sensor in
1. CFTR dysfunction reduces or blocks HCO32 secretion by type
type B ICs (outlined in red) and thereby regulates HCO32 excre-
B ICs.
tion (40). 2. Hypokalemia increases proximal and distal renal tubule HCO32
All of these discoveries help explain why patients with CF are reabsorption.
particularly prone to the development of severe metabolic alkalosis 3. Secondary hyperaldosteronism increases principal cell Na1
when they experience any alkaline stress. reabsorption and generates a negative intraluminal potential
difference (PD). That electronegative PD enhances both K1
secretion and Cl2 reabsorption.
Generation and Maintenance of Metabolic Alkalosis 4. ECF volume and chloride depletion reduces distal nephron
in Patients with Cystic Fibrosis fluid volume and Cl2 delivery, which are required for type B
ICs to secrete HCO32.
Patients with cystic fibrosis (CF), especially adults, who develop
hypoelectrolytemia with metabolic alkalosis (HMA) have almost Aggressive ECF volume expansion (saline) and K1 repletion results
always sweated profusely. Defective cystic fibrosis transmembrane in renal excretion of NaHCO3, movement of K1 into cells in
conductance regulator (CFTR) in sweat ducts reduces Na1 and Cl2 exchange for H1, and correction of the metabolic alkalosis, despite
reabsorption, which markedly increases final sweat [Na1] and persistent type B IC dysfunction. Recent reports by Berg et al., also
[Cl2]. Therefore, a major potential generator of the metabolic alka- indicate that treatment with drugs such as elexacaftor-tezacaftor-
losis may be contraction alkalosis. However, the sum of the ivacaftor can ameliorate the tendency for metabolic alkalosis to
[HCO32] and potential [HCO32] (i.e., lactate salts) in high- develop (33).
volume CF sweat is typically quite high, at about 36 mEq/L (41).
This alkali loss will offset development of sweating-generated con-
traction alkalosis. Does Metabolic Alkalosis in Patients with Cystic
CF patients who experience large volume sweating will initially Fibrosis Contribute to Respiratory Acidosis?
develop hypokalemia because their sweat [K1] is usually quite high As noted, serum [HCO32] is increased in genetically engineered
(25–50 mEq/L) (42). The high-volume salt-rich sweating also gener- cystic fibrosis (CF) mice. Is this true of patients with CF? And, if
ates hypovolemia and secondary hyperaldosteronism. The high aldo- patients with CF have higher serum [HCO32], does this generate a
sterone levels stimulate distal Na reabsorption and renal K loss. compensatory increase in pCO2? Metabolic alkalosis can exacerbate
Abnormal, or deficient, CFTR in type B ICs reduces NaCl the hypercarbia of primary respiratory acidosis. Pedroli et al.,
Nephrology Self-Assessment Program - Vol 23, No 2, June 2024 85
References
1. Quinton PM: Physiological basis of cystic fibrosis: A historical perspec-
tive. Physiol Rev 79[Suppl]: S3–S22, 1999 PubMed
2. Marx JL. The CF gene hits the news. Science 245: 924, 1989 PubMed
3. Kessler WR, Andersen DH: Heat prostration in fibrocystic disease of the
pancreas and other conditions. Pediatrics 8: 648–656, 1951 PubMed
4. Darling RC, Disant’Agnese PA, Perera GA, Andersen DH: Electrolyte
abnormalities of the sweat in fibrocystic disease of the pancreas. Am J
Med Sci 225: 67–70, 1953 PubMed
5. Di Sant’Agnese PA, Darling RC, Perera GA, Shea E: Sweat electrolyte
disturbances associated with childhood pancreatic disease. Am J Med 15:
777–784, 1953 PubMed
6. Rendle-Short J; Archives Disease of Childhood: Fibrocystic disease of the
pancreas presenting with acute salt depletion. Arch Dis Child 31: 28–30,
1956 PubMed
7. Di Sant’Agnese PA: Salt depletion in cold weather in infants with cystic
fibrosis of the pancreas. JAMA 172: 2014–2021, 1960 PubMed
8. Gottlieb RP: Metabolic alkalosis in cystic fibrosis. J Pediatr 79: 930–936,
1971 PubMed
9. Scurati-Manzoni E, Fossali EF, Agostoni C, Riva E, Simonetti GD, Zano-
lari-Calderari M, et al.: Electrolyte abnormalities in cystic fibrosis: Systematic
review of the literature. Pediatr Nephrol 29: 1015–1023, 2014 PubMed
10. Terlizzi V, Padoan R, Leonetti G, Vitullo P, Tosco A, Taccetti G, et al.:
Cystic fibrosis and CFTR-related disorder with electrolyte imbalance at
diagnosis: clinical features and outcome in an Italian cohort. Eur J
Pediatr 182: 5275–5283, 2023 PubMed
Figure 3. Redrawn from Holland et al. (44). Arterial blood gas 11. Sismanlar Eyuboglu T, Dogru D, Çakır E, Cobanoglu N, Pekcan S,
Cinel G, et al.: Clinical features and accompanying findings of Pseudo-
(ABG) results in 14 cystic fibrosis (CF) patients with acute hypercap-
Bartter Syndrome in cystic fibrosis. Pediatr Pulmonol 55: 2011–2016,
nic deterioration and 49 chronic obstructive pulmonary disease 2020 PubMed
(COPD) patients with acute exacerbation. The findings are plotted 12. Smith HR, Dhatt GS, Melia WM, Dickinson JG: Cystic fibrosis present-
using a Stinebaugh-Austin diagram (48). ing as hyponatraemic heat exhaustion. BMJ 310: 579–580, 1995
PubMed
13. Bates CM, Baum M, Quigley R: Cystic fibrosis presenting with hypoka-
measured arterial blood gases (ABGs) in 199 children with CF and lemia and metabolic alkalosis in a previously healthy adolescent. J Am
Soc Nephrol 8: 352–355, 1997 PubMed
reported 8% had metabolic alkalosis, 5% had mixed metabolic alka-
14. Baird JS, Walker P, Urban A, Berdella M: Metabolic alkalosis and cystic
losis and respiratory acidosis, and 5% had respiratory acidosis (43). fibrosis. Chest 122: 755–756, 2002 PubMed
Subsequently, Holland et al., reported that 71% of 14 consecutive 15. Dave S, Honney S, Raymond J, Flume PA: An unusual presentation of
hypercapnic adults with CF exhibited mixed metabolic alkalosis and cystic fibrosis in an adult. Am J Kidney Dis 45: e41–e44, 2005 PubMed
respiratory acidosis compared with only 22% of 49 consecutive 16. Sweetser LJ, Douglas JA, Riha RL, Bell SC: Clinical presentation of met-
patients with chronic obstructive pulmonary disease exacerbation abolic alkalosis in an adult patient with cystic fibrosis. Respirology 10:
who had this mixed A-B disorder (44) (Figure 3). Subsequently, 254–256, 2005 PubMed
17. Augusto JF, Sayegh J, Malinge MC, Illouz F, Subra JF, Ducluzeau PH:
Al-Ghimlas et al. and Spoletini et al., reported similar findings
Severe episodes of extra cellular dehydration: An atypical adult presenta-
(45,46). Although it is very plausible that the metabolic alkalosis tion of cystic fibrosis. Clin Nephrol 69: 302–305, 2008 PubMed
exacerbated the respiratory acidosis, it remains unproven whether 18. Priou-Guesdon M, Malinge MC, Augusto JF, Rodien P, Subra JF, Bon-
preventing or treating the metabolic alkalosis would improve the neau D, et al.: Hypochloremia and hyponatremia as the initial presenta-
respiratory status of these patients (47). tion of cystic fibrosis in three adults. Ann Endocrinol (Paris) 71: 46–50,
In conclusion, the development of severe volume contraction 2010 PubMed
and hypokalemic metabolic alkalosis occurs relatively commonly in 19. Pavone MA, Solıs Padrones A, Muratore DG, Saiz M, Puig C: Hipona-
infants with CF and is often the first manifestation of the disease. tremia, hipopotasemia e insuficiencia renal aguda prerrenal como pre-
sentacion de fibrosis quıstica. [Hyponatraemia, hypopotassaemia and
These infants generally have a milder clinical course of CF. Rarely,
prerenal acute renal failure as a presentation of cystic fibrosis] Nefrologia
adults with previously undiagnosed and clinically mild forms of CF 30: 481–482, 2010 PubMed
develop severe volume contraction, hypokalemia, and metabolic 20. Vertolli U, Ruffatti A, Loreta De Giorgi M, Scapin V, Naso A, Calo LA:
alkalosis after exposure to a hot environment. Recent discoveries of A very unusual case of hypokalaemia. Clin Kidney J 6: 87–89, 2013
renal tubule dysfunction in patients with CF, in part related to PubMed
86 Nephrology Self-Assessment Program - Vol 23, No 2, June 2024
21. Raya CM, Zubillaga IP, Schneider P: Alcalosis metabolica con hiponatre- 35. Berg P, Svendsen SL, Ayasse N, Sorensen MV, Leipziger J: Secretin: A
mia, hipopotasemia e hipocloremia como forma de presentacion de fibro- hormone for HCO32 homeostasis [published online ahead of print
sis quıstica en un adulto. [Metabolic alkalosis with hyponatremia, Jan 25, 2024]. Pflugers Arch doi:10.1007/s00424-024-02906-3 2024
hypokalemia and hypochloremia as the initial presentation of cystic fibro- PubMed
sis in an adult patient] Med Clin (Barc) 143: 137–138, 2014 PubMed 36. Wall SM: Regulation of blood pressure and salt balance by pendrin-
22. Cao Y, Donaldson R, Lee D: “Summer hypokalemia” as an initial pre- positive intercalated cells: Donald Seldin Lecture 2020. Hypertension 79:
sentation of cystic fibrosis in a morbidly obese African American adult: 706–716, 2022 PubMed
Case report. BMC Nephrol 21: 462, 2020 PubMed 37. Varasteh Kia M, Barone S, McDonough AA, Zahedi K, Xu J, Soleimani
23. Ghimire S, Yerneni H, Oyadomari TA, Sedlacek M: Metabolic alkalosis and M: Downregulation of the Cl2/HCO32exchanger pendrin in kidneys of
cystic fibrosis: A case report. Ann Intern Med 173: 315, 2020 PubMed mice with cystic fibrosis: Role in the pathogenesis of metabolic alkalosis.
24. Raya Cruz M, Zubillaga IP, Schneider P: Metabolic alkalosis with hypona- Cell Physiol Biochem 45: 1551–1565, 2018 PubMed
tremia, hypokalemia and hypochloremia as the initial presentation of cystic 38. Procino G, Milano S, Carmosino M, Barbieri C, Nicoletti MC, Li JH,
fibrosis in an adult patient. Med Clin (Barc) 143: 137–138, 2014 PubMed et al.: Combination of secretin and fluvastatin ameliorates the polyuria
25. Choi JY, Muallem D, Kiselyov K, Lee MG, Thomas PJ, Muallem S: associated with X-linked nephrogenic diabetes insipidus in mice. Kidney
Aberrant CFTR-dependent HCO32 transport in mutations associated Int 86: 127–138, 2014 PubMed
with cystic fibrosis. Nature 410: 94–97, 2001 PubMed 39. Azroyan A, Morla L, Crambert G, Laghmani K, Ramakrishnan S,
26. Quinton PM: The neglected ion: HCO32. Nat Med 7: 292–293, 2001 Edwards A, et al.: Regulation of pendrin by cAMP: Possible involvement
PubMed in b-adrenergic-dependent NaCl retention. Am J Physiol Renal Physiol
27. Quinton PM: Cystic fibrosis: Impaired bicarbonate secretion and mucov- 302: F1180–F1187, 2012 PubMed
iscidosis. Lancet 372: 415–417, 2008 PubMed 40. Vitzthum H, Meyer-Schwesinger C, Ehmke H: Novel functions of the
28. Berg P, Svendsen SL, Sorensen MV, Larsen CK, Andersen JF, Jensen- anion exchanger AE4 (SLC4A9) [published online ahead of print Jan 29,
Fangel S, et al.: Impaired renal HCO32 excretion in cystic fibrosis. J Am 2024]. Pflugers Arch doi:10.1007/s00424-023-02899-5 2024 PubMed
Soc Nephrol 31: 1711–1727, 2020 PubMed 41. Bijman J, Quinton PM: Lactate and bicarbonate uptake in the sweat
29. Berg P, Jeppesen M, Leipziger J: Cystic fibrosis in the kidney: New les- duct of cystic fibrosis and normal subjects. Pediatr Res 21: 79–82, 1987
sons from impaired renal HCO32 excretion. Curr Opin Nephrol Hyper- PubMed
tens 30: 437–443, 2021 PubMed 42. Devlin J, Beckett NS, David TJ. Elevated sweat potassium, hyperaldos-
30. Berg P, Svendsen SL, Hoang TTL, Praetorius HA, Sorensen MV, Leipzi- teronism and pseudo-Bartter's syndrome: a spectrum of disorders associ-
ger J: Impaired renal HCO32 secretion in CFTR deficient mice causes ated with cystic fibrosis. J R Soc Med 82: 38–43, 1989 PubMed
metabolic alkalosis during chronic base-loading. Acta Physiol (Oxf) 231: 43. Pedroli G, Liechti-Gallati S, Mauri S, Birrer P, Kraemer R, Foletti-J€aggi
e13591, 2021 PubMed C, et al.: Chronic metabolic alkalosis: not uncommon in young children
31. Berg P, Svendsen SL, Sorensen MV, Schreiber R, Kunzelmann K, Leipzi- with severe cystic fibrosis. Am J Nephrol 15: 245–250, 1995 PubMed
ger J: The molecular mechanism of CFTR- and secretin-dependent renal 44. Holland AE, Wilson JW, Kotsimbos TC, Naughton MT: Metabolic
bicarbonate excretion. J Physiol 599: 3003–3011, 2021 PubMed alkalosis contributes to acute hypercapnic respiratory failure in adult cys-
32. Berg P, Andersen JF, Sørensen MV, Wang T, Malte H, Leipziger J: tic fibrosis. Chest 124: 490–493, 2003 PubMed
Alkalosis-induced hypoventilation in cystic fibrosis: The importance of 45. Al-Ghimlas F, Faughnan ME, Tullis E: Metabolic alkalosis in adults
efficient renal adaptation. Proc Natl Acad Sci U S A 119: e2116836119, with stable cystic fibrosis. Open Respir Med J 6: 59–62, 2012 PubMed
2022 PubMed 46. Spoletini G, Fitch G, Gillgrass L, Etherington C, Clifton I, Peckham
33. Berg P, Sorensen MV, Rousing AQ, Vebert Olesen H, Jensen-Fangel S, DG: Urinary bicarbonate and metabolic alkalosis during exacerbations in
Jeppesen M, et al.: Challenged urine bicarbonate excretion as a measure cystic fibrosis. ERJ Open Res 8: 00669–02021, 2022 PubMed
of cystic fibrosis transmembrane conductance regulator function in cystic 47. Bemand TJ, Chatoor R, Natale P, Strippoli G, Delaney A: Acetazol-
fibrosis. Ann Intern Med 175: 1543–1551, 2022 PubMed amide for metabolic alkalosis complicating respiratory failure with
34. Berg P, Jensen T, Andersen JF, Svendsen SL, Modvig IM, Wang T, et al.: chronic obstructive pulmonary disease or obesity hypoventilation syn-
Loss of the secretin receptor impairs renal bicarbonate excretion and drome: A systematic review. Thorax 78: 1004–1010, 2023 PubMed
aggravates metabolic alkalosis in mice during acute base-loading. J Am 48. Stinebaugh BJ, Austin WH: Acid-base balance—A pattern-analysis dia-
Soc Nephrol 34: 1329–1342, 2023 PubMed gram. N Engl J Med 275: 375, 1966 PubMed
Nephrology Self-Assessment Program - Vol 23, No 2, June 2024 87
Article
Hyponatremia
Bryan M. Tucker, DO, MS, FASN and Alexandra Grand, MD
Section of Nephrology, Baylor College of Medicine, Houston, Texas
Pathophysiology
Learning Objectives Hyponatremia can be evaluated in the context of the Edelman equa-
1. To describe the two main approaches to the workup of tion (21).
hyponatremia Edelman equation : Plasma ½Na51:11ðNae 1K e Þ=TBW225:6
2. To determine the differential diagnosis and pathophysiology of
where Edelman demonstrated that plasma water sodium is a func-
hyponatremia
tion of exchangeable sodium and potassium divided by total body
3. To evaluate the different medications that contribute to water.
Generally speaking, hypotonic hyponatremia can be attributed
medication-induced hyponatremia
to one or many of the following: high net free water intake,
4. To determine the best treatment options for hyponatremia impaired dilution of the urine, and low excretion of electrolyte free
5. To determine the risk factors for osmotic demyelination in water (22). Chronic kidney disease and aging can reduce the
amount of free-water excretion, thereby increasing the risk of hypo-
patients with hyponatremia
natremia (23). A reduction in exchangeable sodium and potassium
will lead to difficulty in excreting free water.
deficiency releases negative feedback on corticotropin-releasing hor- Polydipsia. Polydipsia, or water intoxication, occurs when the
mone and ADH production as well. An ADH-independent mecha- patient drinks too much water. A large systemic review identified
nism that decreases water clearance via a reduction of GFR occurs as that 52% of patients in this category have a psychiatric disorder
well (26). (38). Average intake of water consumed was 8 L/day presenting
with an average sodium of 118 mmol/L. The most common etiol-
Malnutrition ogy is psychogenic polydipsia. Using the Edelman equation, water
ingestion increases total body water, thereby diluting sodium, result-
Hyponatremia can also be caused by malnutrition, and its patho-
ing in a decreased serum sodium.
physiology is multifactorial. There is a reduction in water excretion.
This is due to reduced solute intake leading to a salt deficit, thereby Hypervolemic Hyponatremia
limiting the excretion of free water. Low potassium, which is also
usually present in these patients, can contribute to hyponatremia Cirrhosis is also a common cause of hypervolemic hyponatremia.
through sodium shift into the intracellular department. Inflamma- Because of the increased intrahepatic resistance, portal hypertension
develops. This increase in hepatic sinusoidal pressure leads to release
tion is also present, which may activate the hypothalamic receptors
of vasodilators, specifically nitric oxide and prostaglandins, which
and increase thirst (27).
then act on the peripheral vascular beds, resulting in splanchnic arte-
rial vasodilation (39). This pooling of blood leads to a compensatory
Coronavirus Disease 2019 increase in cardiac output and decrease in systemic vascular resis-
Hyponatremia has often been seen in coronavirus disease 2019 tance, resulting in reduced effective arterial blood flow. This will
(COVID-19) infections as well. Although these patients can present stimulate the renin-angiotensin-aldosterone system and sympathetic
with many different etiologies of hyponatremia, the most common nervous system. Nonosmotic release of ADH also occurs, which
hyponatremia presentations are hypovolemic and euvolemic hypona- increases water reabsorption, leading to hyponatremia (39).
tremias (28,29). The exact pathophysiology is likely multifactorial. Hyponatremia in heart failure is attributed to increased
Excess ADH secretion from a hypothalamic-pituitary-adrenal abnor- ADH secretion, sympathetic nervous system activation, and
mality and/or ectopic production from COVID pneumonia can be renin-angiotensin-aldosterone system activation (40). The reduced
seen (30). Recently, interleukin-6 (IL-6) has become of increasing cardiac output activates the baroreceptors and angiotensin II release,
interest in pathogenesis. Increased IL-6 levels are inversely related to thereby resulting in nonosmotic release of ADH (41). Moreover, in
serum sodium. IL-6 mediates ADH secretion and impairs osmoreg- the setting of renal hypoperfusion and reduced distal nephron flow,
ulatory ADH mechanisms in the brain (30). the V2R activation effect will be a more pronounced increase free
water reabsorption. As the cardiac index decreases, ADH levels
Hypothyroidism increase (42). This nonosmotic release of ADH seems to be the
main driving factor for hyponatremia in heart failure. An interesting
Hypothyroidism as a cause of hyponatremia is questionable. It has
association is also seen between hyponatremia and takotsubo syn-
been attributed to impaired free-water excretion and kidney sodium-
drome (TTS) where central noradrenergic activation leading to
handling changes. Of hyponatremic patients, less than 1% of cases increased sympathetic activity can lead to TTS as well as increased
are due to hypothyroidism (31). Unless myxedema coma is present, ADH resulting in hyponatremia (43). Hyponatremia seems to exac-
other hyponatremic etiologies should be considered (32). erbate TTS as well.
excluding hyperglycemic hyponatremia is necessary (46). The serum with a clinical assessment of the patient’s volume status and subse-
sodium can be corrected for the glucose concentration by adding quently obtaining urine studies to further distinguish the different
1.6 mmol/L to the measured sodium for every 100 mg/dl incremental causes (45).
increase in serum glucose above 100 mg/dl (47). Other potential
effective osmoles include mannitol, sorbitol, glycine, and hyperosmo- Hypervolemia. In patients with signs of volume overload (periph-
lar radiocontrast agents (46). eral edema, pulmonary edema, ascites, elevated jugular venous
Pseudohyponatremia is a subset of isotonic hyponatremia where pulse), urine sodium less than 20 mmol/L should prompt an evalua-
there are abnormally high concentrations of proteins or lipids in the tion for heart failure, cirrhosis, or nephrotic syndrome. A urine
blood, which interferes with the accurate measurement of sodium sodium greater than 30 mmol/L is more suggestive of renal failure
(45). Pseudohyponatremia continues to be a problem in laboratories as the cause or may indicate diuretic use.
using indirect ion-selective electrodes to measure sodium. Because
this measurement style utilizes a predilution step before the sodium Hypovolemia. In patients with signs of volume depletion (ortho-
measurement, the machine back-calculates the true sodium. This static hypotension, dry mucus membranes, decreased skin turgor),
calculation assumes the patient’s body water content is 93%. When urine sodium less than 20 mmol/L suggests extra renal losses, which
the patient’s body water content is more or less than 93%, the may include diarrhea, vomiting, or third spacing. A urine sodium
sodium result obtained via the indirect ion-selective measurement is greater than 30 mmol/L can be indicative of urinary sodium losses
not accurate (48). Hyperlipidemia and hyperproteinemia can result resulting from diuretic use, primary adrenal insufficiency (AI), or
in a reduction in the body’s water content, which may result in cerebral salt wasting, among others.
pseudohyponatremia when measuring with an indirect ion-selective
electrode. Measuring the serum sodium directly via point-of-care Euvolemia. In patients who appear euvolemic, urine osmolality
sodium or whole blood sodium overcomes this problem. ,100 mOsm/kg indicates maximally dilute urine and is suggestive
of water intake exceeding the excretory ability of the kidneys. A high
urine osmolality .100 mOsm/kg H2O can result from SIAD, AI, or
severe hypothyroidism. The criteria for diagnosing SIAD include clin-
4 Hyponatremia pathophysiology often involves ical euvolemia, an inappropriately concentrated urine (osmolality
excess total body water, a deficit of salt/protein or .100 mOsm/kg H2O), high urine sodium (.30 mEq/L), and the
exclusion of AI, severe hypothyroidism, and kidney disease (45). Spe-
a combination of both. cifically, in a prospective observational study, 14.2% of patients with
euvolemic hyponatremia had AI (49). Of these cases, secondary AI
was most frequently seen from previous steroid use. Only severe cases
of hypothyroidism with myxedema coma can be associated with clini-
Hypertonic Hyponatremia cally important hyponatremia, so it should be ruled out in select cases.
Hypertonic hyponatremia can occur when there are additional effec- There are numerous potential causes of SIAD, including lung disease,
tive osmoles present in the serum. Hypertonicity results when the central nervous diseases, ectopic antidiuretic hormone (ADH)-secret-
concentrations of the effective osmoles are so high that they cannot ing tumors, and a multitude of drugs (Tables 1 and 2).
be compensated by the shifts of water from the intracellular to the The volume exam is difficult and often inconsistent. Because of
extracellular fluid. The same osmoles (hyperglycemia, mannitol, these inaccurate or misleading physical exams, another algorithmic
radiocontrast agents) that cause isotonic hyponatremia can cause approach to hyponatremia was developed using point-of-care ultra-
hypertonic hyponatremia. sound (50,51). Other researchers found that serum creatinine may aid
in differentiating between hypovolemic and euvolemic hyponatremia.
Hypotonic Hyponatremia In hypovolemic hyponatremic patients, the mean serum creatinine
Once nonhypotonic hyponatremia is ruled out, there are two was 1.28 mg/dl versus 0.76 mg/dl in euvolemic hyponatremic
approaches described. The US expert panel recommendations start patients (52).
Categories Causes
Mechanism Medications
Drugs affecting sodium and water homeostasis Diuretics (thiazides, indapamide, amiloride, loop diuretics)
Drugs affecting water homeostasis Antidepressants (amitriptyline, SSRIs, monoamine oxidase inhibitors)
Antipsychotic drugs (phenothiazines, haloperidol)
Antiepileptic drugs (carbamazepine, oxcarbazepine, sodium valproate)
Anticancer agents (vincristine, vinblastine, cisplatin, carboplatin, intravenous
cyclophosphamide, melphalan, ifosfamide, methotrexate, interferon, levamisole,
monoclonal antibodies)
Potentiation of ADH Carbamazepine, lamotrigine, chlorpropamide, tolbutamide, intravenous
cyclophosphamide, NSAIDs
Reset osmostat Venlafaxine, carbamazepine
Adapted with permission from reference 56. ADH, antidiuretic hormone; NSAID, nonsteroidal anti-inflammatory drugs, SSRIs, Selective serotonin reuptake inhibitors.
Several studies have shown that the sensitivity and specificity of Other Laboratory Results
the volume exam are low, and, thus, starting with the volume exam Differentiating between SIAD and thiazide-induced hyponatre-
may not be the ideal approach. Therefore, the European guidelines
mia can be challenging. A recent study evaluating the differences
recommend obtaining urine osmolality first (Figure 1) (46). If the
urine osmolality is ,100 mOsm/kg, then excess water intake is the demonstrated that a fractional excretion of potassium (FEk)
likely cause of the hyponatremia. If urine osmolality is .100 .9% and urine calcium/creatinine ratio ,0.1 identified .90%
mOsm per/kg, then urine sodium can further characterize the etiol- of cases of thiazide-induced hyponatremia, whereas FEk ,9%
ogy. Urine sodium ,30 mmol/L indicates low effective arterial vol- and urine calcium/creatinine ratio .0.1 identified 75% of SIAD
ume as the cause, whereas urine sodium .30 mmol/L warrants an patients (53). Another study also determined that a urinary
evaluation of diuretic use and volume status. Please refer to Figure 1 calcium/cr ratio .0.15 can suggest SIAD (54).
for a representation of this approach.
US Expert Panel Self-induced acute water 3% NS 100 ml over Na increase 4–6 mmol/L 44
Recommendations intoxication, known duration 10 min 33 as
(2013) #24–48 h, intracranial needed
pathology or increased
intracranial pressure, seizures
or coma
European Clinical Na ,130 mmol/L with severe 3% NS 150 ml over Na increase 5 mmol/; if 45
Practice Guidelines (vomiting, cardiorespiratory 20 min symptoms persist,
(2014) distress, anormal and deep increase Na 1 mmol/L
somnolence, seizures, Glascow while looking for other
coma scale ,8) or moderately etiologies of
severe (nausea without symptoms; if none
vomiting, confusion, identified, increase Na
headache) symptoms 10 mmol/L in 24 h
Spanish Society of Na#120 mmol/L or Na,125 3% NS 100–150 ml Na increase 4–6 mmol/L 133
Endocrinology and mmol/L with symptoms or 2 ml/kg over
Nutrition (2023) 10–20 min
repeating as many
times as necessary
until goal is reached
Nephrology Self-Assessment Program - Vol 23, No 2, June 2024 91
Figure 1. Workup of hyponatremia. ECF, extracellular fluid. Reprinted from ref. 132, with permission.
Serum uric acid levels will typically be ,4 mg per dl in SIAD, SIAD (.12%) and low in hypovolemia. Similarly, urea levels and frac-
whereas levels .5 mg/dl are usually observed in patients with hypovo- tional excretion of urea can be of use: low urea levels ,10 mg/dl and
lemia. The fractional excretion of uric acid is expected to be high in high fractional excretion of urea (.55%) are seen with SIAD (55).
92 Nephrology Self-Assessment Program - Vol 23, No 2, June 2024
Medication-Induced Hyponatremia and it often presents within 10 days of administration (64,66). The
In the workup of hyponatremia, one must always keep in mind mechanism of cisplatin-associated hyponatremia is less clear but is
medications, because they can lead to a variety of presentations and attributed to renal salt wasting (RSW), leading to hypovolemic
produce hyponatremia in a variety of ways (Table 2). hyponatremia (61). The incidence of immune checkpoint inhibitor
Hyponatremia is a well-recognized complication of diuretics. (e.g., nivolumab, ipilimumab, pembrolizumab) associated hypona-
Loop diuretics inhibit sodium chloride reabsorption in the thick tremia varies from 1% to 61%, with the highest incidence seen with
ascending loop of Henle, thereby impairing the generation of the ipilimumab (64,67). The incidence of severe hyponatremia (Na
medullary concentrating gradient. This reduces the concentrating abil- ,124 mmol/L) is 6%, occurring, on average, 164 days after drug
ity of the kidneys—thus the decreased risk of hyponatremia (56). initiation (67). The mechanism behind immune check point
Thiazides and thiazide-like diuretics are the most common type of inhibitor–associated hyponatremia is attributed to SIAD. Although
diuretics that induce hyponatremia (56). Hyponatremia is reported to less common, immune endocrinopathies can also be the cause (64).
occur in 5%–30% of hospitalized patients who take thiazides (57). Several antidepressants and antipsychotics are associated with
The outpatient 2-year cumulative incidence of hyponatremia is hyponatremia, including SSRIs, serotonin-noradrenaline reuptake
3.83% on thiazides alone and 3.51% on a thiazide-renin–angiotensin inhibitors (SNRIs), tricyclic antidepressants, and mirtazapine. In a
system inhibitor combination pill (58). In a systematic review and retrospective case-control study in China, the incidence of SSRI or
meta-analysis encompassing data from 2840 cases of thiazide-induced SNRI-associated hyponatremia was 1.3%, with an average duration
hyponatremia leading to hospitalization, patients had a mean age of of onset of 7 days and an average sodium level of 107 mmol/L (68).
75 years, and 79% were women. Mean sodium was 116 mmol/L This study also found that sertraline use was significantly higher in
and average time from initiation to diagnosis of thiazide-induced the hyponatremia group compared with a control group, who were
hyponatremia was 19 days. There were high rates of coprescription patients on SSRIs or SNRIs but without hyponatremia (68). A
of other medications associated with hyponatremia such as nonste- 2017 review shows the incidence to be higher, at 15% (69). The
roidal anti-inflammatory drugs (NSAIDs) and selective serotonin risk of hyponatremia associated with SSRIs is highest within the first
reuptake inhibitors (SSRIs), suggesting a possible additive effect week of initiation, with an adjusted odds ratio of 29; this risk
from other medications known to cause hyponatremia. Although decreases to 2.1 at week 13 (70). Risk factors seen were older age,
not as common, hyponatremia can still occur months or years later female sex, low body weight, lower baseline sodium, and concomi-
(59). Risk factors include older age and female sex (57,59). Two tant use of diuretics (69). A retrospective study in Japan reviewed
phenotypes exist of patients with diuretic induced hyponatremia: 366,232 patients .60 years old who were started on an antidepressant.
one consistent with volume depletion and another that simulates A significant decrease in serum sodium was seen within 30 days of
SIAD (60). Serum uric acid levels may help differentiate between initiation of SSRI/SNRI but not for noradrenergic and specific seroto-
the two. Serum uric acid level ,4 mg/dl was associated with SIAD, nergic antidepressants. The mechanism behind this is attributed to
and serum uric acid level .4 mg/dl was associated with volume SIAD. ADH is released by central vasopressin neurons that are modu-
depletion (60). Thiazide induced hyponatremia might have multiple lated by serotonin, noradrenaline, and dopamine. Antidepressant action
mechanisms, including sodium-chloride cotransporter inhibition on these modulators may be directly responsible for hyponatremia via
sodium loss and impaired urinary dilution, AQP2 upregulation via its actions on the serotonin transporter. Pharmacoepidemiologic-
prostaglandin E2 mediation, and extrarenal causes such as low solute pharmacodynamic analysis shows that a decrease in serum sodium
intake, excess water intake and hypokalemia coexistence resulting in levels is significantly correlated with serotonin transporter binding
transcellular cation exchange (61). After drug withdrawal, normali- (71). Another study demonstrated that fluoxetine and sertraline do
zation of sodium and normal urine diluting capacity can take 1–2 not increase ADH levels and thus may act directly on the V2 recep-
weeks (57). tor (V2R) (61). Hyponatremia improved within 2 weeks of medica-
Hyponatremia is a common complication in cancer patients tion discontinuation (69).
with a prevalence of up to 40%, with the most common etiology In an epilepsy cohort, the medications most associated with
being SIAD (62). Although many cancers can cause SIAD, a multi- hyponatremia were carbamazepime, valproate, phenytoin, pheno-
tude of medications these patients take are also a common etiology of barbital, benzodiazepines, and antipsychotics, whereas concomi-
hyponatremia. Common medications that can cause or contrib- tant zonisamide use was associated with a lower risk (72).
ute to hyponatremia in this population include chemotherapeutic Oxcarbazepine can cause hyponatremia, with a prevalence of 61%,
agents (cyclophosphamide, vincristine, vinblastine, cisplatin, mel- and a severe hyponatremia prevalence of 19% (73). The average
phalan), morphine, immunomodulators (immunoglobulins, inter- time on oxcarbazepine before hyponatremia was diagnosed was 8.7
feron, interleukin-2, levamisole), and nonsteroidal anti-inflammatories, years; with each year a patient is on oxcarbazepine, the risk of hypo-
to name a few (62). Case reviews and retrospective data demonstrate natremia increases by 1.3 (73). The reason for this late onset is
that cyclophosphamide-induced hyponatremia develops on an average unclear. The mechanism of oxcarbazepine-induced hyponatremia is
cyclophosphamide dose of 20–30 mg/kg and presents with an average attributed to two mechanisms, a direct effect on the collecting
serum sodium of 111 mmol/L with incidence between 9% and 52% tubules increasing the responsiveness of ADH and inhibiting prosta-
(63,64). Cyclophosphamide and ifosfamide may directly stimulate the glandin E2 formation (73). Prostaglandin E2 reduces production of
vasopressin 2 (V2) receptor without the increase in ADH (65). Vincris- cAMP, thereby reducing aquaporin migration to the apical
tine causes SIAD via increasing ADH levels. Animal studies show a plasma membrane. By inhibiting prostaglandin E2, cAMP inhibi-
direct toxic effect on the neurohypophysis and hypothalamic system tion is gone, and more aquaporin migration occurs, leading to
(61). The incidence of cisplatin-associated hyponatremia is 11%, more water permeability.
Nephrology Self-Assessment Program - Vol 23, No 2, June 2024 93
Proton pump inhibitors are also associated with hyponatremia. serum sodium less than 120 mmol/L, overcorrection incidence was
In a retrospective case-control study, hyponatremia was more com- 41% (86). Overcorrection risk factors include a lower serum sodium
mon when patients used proton pump inhibitors at least 30 days (,110 mEq/L), chronic alcoholism, severe symptoms of hyponatre-
before a hospitalization (74). Analysis of data from the Food and mia, use of 3% saline, younger age, women, urine sodium less than
Drug Administration database of Adverse Event Reporting System 30 mmol/L, and hypokalemia (85–87). A systematic review of
reported an odds ratio of 3.948 for the development of hyponatre- osmotic demyelination cases showed it was more common in females,
mia in patients taking pantoprazole (75). Omeprazole and esome- and patients with a lower sodium in patients with a fast correction
prazole showed an adjusted odds ratio of 1.23 for the development rate (88). The review also showed that, in 10% of cases, demyelin-
of hyponatremia regardless of treatment duration, although, if ation occurred in patients with sodium correction less than 10
recently initiated, the adjusted odds ratio was 6.87, which gradually mmol/L. It has been recommended that, in patients with sodium less
declined with a longer duration of use (76). than 115 mmol/L, a correction rate of ,8 mmol per /L per day is
needed because of this risk (89). A retrospective study in the United
Kingdom evaluated 112 patients who received hypertonic saline for
Hyponatremia Treatment severe symptomatic hyponatremia found overcorrection at 24 hours
Acute Treatment and 48 hours (.10 and .18 mmol/L) in 44.9% and 19.6% of
patients, respectively, with no cases of ODS (90). A large multicenter
Symptomatic hyponatremia is a medical emergency and requires
cohort study evaluated the risk of ODS in five academic hospitals in
urgent treatment to prevent cerebral edema and death. Treatment
Toronto, Canada from 2010 to 2020. In total, 22,858 hospitalizations
recommendations are summed up in Table 3, but, generally, recom-
with sodium ,130 mmol/L were included. The mean initial sodium
mendations are to give 100–150 ml hypertonic saline (3% saline)
was 125 mmol/L, 17.7% were found to have rapid correction of
over 10–20 minutes and repeated up to 2–3 times until desired
sodium (.8 mmol/L in 24 hours), and ODS occurred in only 0.5%
sodium goal is reached (usually 5 mmol/L or until symptoms
of patients (91). Another interesting result of this study was that there
resolve) (1). The European and American Guidelines recommend a
was no relationship between the rapid correction of sodium and ODS,
sodium correction of no more than 10 mmol/L per day and
with the majority ODS cases (seven of 12) having a slow rate of cor-
8 mmol/L per day, respectively, but, as osmotic demyelination syn- rection. This study was retrospective and limited to the Toronto area,
drome (ODS) risk increases, a lower sodium correction goal is thus limiting its generalizability, and risk factors for ODS were not
recommended (45,46). Hypertonic saline can be given through cen- assessed. Other limiting factors for this study was that nearly 90% of
tral and peripheral lines (77,78). Recently, there was a case of hyper- the patients had a sodium of .120 mmol/L and the ODS diagnosis
tonic saline given via the intraosseous route successfully (79). Rapid was based on imaging. In clinical practice the diagnosis of ODS is clin-
intermittent bolus of 3% saline is preferred over a slow continuous ical and imaging may not become positive until weeks after ODS
infusion (80). Both delivery methods were effective and safe, but symptoms appear.
the rapid intermittent bolus showed a better efficacy in reaching These studies have led to arguments that current guidelines are
sodium goal within an hour (80). Treating severe hyponatremia too restrictive and suggested a sodium correction of 8–10 mmol/L
(,120 mmol/L) with normal saline as compared with 3% saline per day and 15–20 mmol/L per 48 hours (92). This argument is
was associated with longer length of stay (81). When giving 3% based on the rarity of ODS and the general improvement in out-
saline, body weight ,60 kg is associated with overcorrection, and comes with faster correction of symptomatic hyponatremic patients
body weight .100 kg is associated with undercorrection (82). A ret- as seen in these newer studies. This is countered by a larger group of
rospective study in The Netherlands describing 130 patients with authors pointing out the flaws in the Toronto study as stated above.
severe hyponatremia (Na,120 mmol/L) showed a bolus of 250 ml They further discuss that the appropriate methodology to diagnose
3% saline as compared with 100 ml is more effective at increasing ODS in clinical studies is a detailed chart review until better diag-
serum sodium $5 mmol/L within 4 hours without increasing risk nostic strategies emerge in the future. Patients with ODS risk factors
of overcorrection (83). A recent, large, retrospective, multicenter should have sodium corrected by #8 mmol/L per day. Although
(208 US hospitals) study evaluated 1024 patients admitted to inten- ODS is extremely uncommon, the fact that it can happen is reason
sive care units (ICUs) with sodium ,120 mmol/L, and compared enough for a cautious approach to hyponatremia treatment, thus fol-
outcomes in those with correction of hyponatremia .8 mmol/L per lowing current treatment guidelines, to be continued (93).
day versus #8 mmol/L per day (84). In this study, 44% of patients To reduce the risk of overcorrection, a proactive strategy has
rapidly corrected (.8 mmol/L per day), which was associated with a been suggested by some authors, called the 1-deamino-8-D-arginine
decrease in adjusted mortality, length of hospital stay, and ICU stay. vasopressin (desmopressin; DDAVP) clamp (94). This strategy
Subanalysis found correction rates of 8–12 mmol/L per day and .12 involves starting desmopressin 2–4 mcg intravenously (i.v.) or sub-
mmol/L per day as compared with ,8 mmol/L per day were associ- cutaneously every 6–8 hours when 3% NS is given. Therapeutic
ated with decreased length of stay; 376 patients had high risk factors relowering of sodium is recommended for high-risk ODS patients
for ODS, and 52% of them overcorrected, yet no cases of ODS were despite low-quality evidence (95). Sodium relowering should be
identified. considered with severe hyponatremia (Na less than 120 mmol/L),
Osmotic demyelination is a toxic encephalopathy primarily chronic hyponatremia or unknown hyponatremia duration when
affecting the white matter. Myelinated nerve sheaths and oligodendro- rapid sodium correction occurs, especially in the setting of ODS risk
cytes are damaged. Hyponatremia overcorrection can lead to osmotic factors or when early ODS symptoms are present. Sodium should
demyelination. In severe hyponatremic patients (,125 mmol/L), be brought down just below sodium correction limit. Recommenda-
overcorrection incidence was 14% (85); in another population with tions on the method of relowering is with i.v. dextrose 5% in water
94 Nephrology Self-Assessment Program - Vol 23, No 2, June 2024
at a rate of 3 ml per kg per hour and DDAVP 2–4 mcg i.v. or sub- increased serum sodium after 4 days compared with placebo (106).
cutaneously every 6–8 hours (95). A retrospective study using an The same investigators carried out a study of 14 outpatients with
infusate and fluid loss formula based on the Edelman equation chronic SIAD. In a 4-week placebo-controlled trial, serum sodium
showed better correction rates with less over- and undercorrection as increased by 4 mmol/L in the treatment group, and this increase was
evaluated in a retrospective study (96). associated with a small improvement in neurocognitive testing (107).
In a post hoc analysis, 4740 patients enrolled in the DAPA-HF
Chronic Treatment (Dapagliflozin in Patients with Heart Failure and Reduced Ejection
Treatment of chronic or “asymptomatic” hyponatremia remains Fraction) study, which examined dapagliflozin versus placebo in
clinically important because data shows improvement in cognitive patients with heart failure with reduced ejection fraction, dapaglifo-
function and comprehensive geriatric assessment parameters (which zin use resulted in more patients developing hyponatremia at day
include mental stare exam, activities of daily living, hand grip 14, but, at 1 year, this pattern reversed, and less patients had hypo-
strength) (6), gait, cognitive function, quality of life (97,98), bone natremia (108). In another study of 285 patients admitted with
formation index (99) and survival (20). A recent prospective cohort acute heart failure, treatment with dapaglifozin increased inpatient
study evaluated 26 patients with chronic hyponatremia, using brain serum sodium by 48 hours, but no difference was seen between
MRI and cognitive testing before and after hyponatremia treatment groups at time of hospital discharge (109). In the heart failure popu-
(100). After an average serum sodium change from 118 mmol/L to lation, persistent hyponatremia is significantly associated with
135 mmol/L, a significant reduction in brain tissue volume (mostly increased risk of death or hospitalization for acute heart failure
in hippocampus), neuronal activity, and synchronization across all within 60 days (108, 110). The improvement in heart failure associ-
gray matter was observed, with associated improvements in measures ated with SLT2 inhibitors may contribute to the improvement in
of cognitive function. sodium seen as well.
In a retrospective review of hospitalized patients with type 2 dia-
Fluid Restriction betes taking SGLT2 inhibitors compared with patients with diabetes
Fluid restriction is the recommended first-line treatment across all not on SGLT2 inhibitors, where cohorts were matched for age, sex,
guidelines. The hyponatremia registry shows that monotherapy admitting diagnosis, and history of heart failure, no significant differ-
treatment with just fluid restriction failed to increase sodium by ence in the prevalence of admission hyponatremia was seen. The
5 mmol/L in over half of the patients (44). One can predict the investigators concluded that, in this analysis, treatment with SGLT2i
response to fluid restriction using the Furst equation ratio (urine did not prevent the development of hyponatremia (111).
sodium1urine potassium/plasma sodium). When this ratio is ,0.5,
fluid restriction ,1000 ml is recommended; when the ratio is Urea/Protein Supplementation
0.5–1, fluid restriction ,500 ml is recommended; and, if the ratio Urea acts as an osmotic diuretic to increase serum sodium. A pro-
is .1, then fluid restriction is unlikely to be effective (101). The spective study evaluated urea in SIAD patients, and saw normona-
American guidelines also give a number of criteria that may predict tremia in 62% of patients after an average treatment of 5 days with
a poor response to fluid restriction, which include high urine osmolal- a dose of 0.25–0.5 g per kg (112). However, it needs to be noted
ity (.500 mOsm/kg H2O), sum of urinary sodium and potassium is that this was a small study with only 13 patients and did not have a
greater than serum sodium, 24-hour urine volume ,1500 ml, and control group. Another retrospective observational study showed
increase in serum sodium ,2 mmol/L in 24–48 hours on fluid restric- urea improved hyponatremia from an average of 125 mmol/L to
tion ,1 L/day (45). A randomized study showed that, after 3 days of 134 mmol/L in approximately 3 months of treatment (113). A
fluid restriction, sodium increased by 3 mmol/L as compared with recent retrospective observational study in an SIAD population
1 mmol/L in the no treatment group, but, on day 30, there was no resulted in a correction of hyponatremia in 44% of patients with an
difference between groups (102). There currently is an ongoing trial average urea dose of 22.5 g per day for an average of 6 days of treat-
comparing fluid restriction to tolvaptan (103). ment (114). Another retrospective study found average sodium
Salt (NaCl) supplementation and loop diuretics have also been increase was 2 mmol/L per day with a median dose of urea 30 g/day
used to treat syndrome of inappropriate diuresis (SIAD). However, (115). A large systemic review, encompassing data primarily from
a recent open label randomized controlled trial of 92 patients with observational studies (many without control groups) evaluating
serum sodium ,130 mmol/L, randomized to fluid restriction alone, treatment with urea in 537 SIAD patients, showed an average
fluid restriction with furosemide or fluid restriction with furosemide increase in sodium by 9.6 mmol/L after 5 days (116). In a proof-of-
and NaCl supplementation did not show any difference in sodium concept study, 90 g of protein powder daily increase sodium on
correction by day 4, with an average increase of 5 mmol/L across all average by 3 mmol/L in 7 days in an SIAD population (117). Pro-
groups (104). tein supplementation was comparable to urea.
A recent retrospective study evaluated the treatment with urea
Sodium-Glucose Transport Protein 2 Inhibitors in a heart failure population of 34 patients (118). A mean dose of
Sodium-glucose transport protein 2 (SGLT2) inhibitors block the urea used was 22.5 g per day, and average sodium improved from
SGLT2 cotransporter in the proximal tubule, which leads to signifi- 126 mmol/L to 136 mmol/L in an average of 4.2 days (118). The
cant glycosuria, thereby increasing free-water excretion (105). Recently, same group followed up this study with a larger retrospective obser-
they have been investigated in the treatment of chronic SIAD. In a vational study with 49 patients a control group (47 patients) show-
Swiss trial of 88 hospitalized patients with SIAD and serum sodium ing an average of 5 days to sodium normalization (114). A common
,130 mmol/L, treatment with empagliflozin led to significantly problem with urea is the taste, with many patients complaining
Nephrology Self-Assessment Program - Vol 23, No 2, June 2024 95
about distaste and dysgeusia (116). In one retrospective study, over consisted of standard medical care for a maximum of 18 months. It
50% of the patients stopped taking urea because of the taste (115). A is a multicenter, randomized, open-label, pragmatic clinical trial.
recent randomized controlled trial demonstrated that a commercial Hyponatremia normalization rate was significantly better in the
formulation containing citrus was better tolerated than the galenic for- albumin group compared to the control group (45% vs 28%,
mulation (119). But this study was funded by the company that made P50.042 at 1 month) and a lower incidence of hyponatremia was
the citrus product. seen as well (incidence rate ratio 0.245 (CI 0.167 to 0.359, P,0.001)
(126). A review covering 18 meta-analyses evaluated i.v. albumin
Tolvaptan treatment for hyponatremia in patients undergoing large-volume para-
Tolvaptan is a V2 receptor (V2R) antagonist, and thereby directly centesis. Although the results were not unanimous, the higher-quality
inhibits the V2R. A retrospective analysis of the Hyponatremia studies determined that i.v. albumin can decrease the incidence of
Registry including 3460 patients showed that the most common tol- hyponatremia in cirrhotic patients undergoing large-volume paracent-
vaptan dose was 7.5 mg (46%) followed by 15 mg (31%), then esis (127). An international position statement and large meta-analysis
3.75 mg (20%), and showed an average sodium correction of state that low-quality evidence exists, but the current evidence suggests
3.4 mmol per L per day (120). A smaller, multicenter, retrospective that albumin infusions can be considered for prevention and treat-
study of 100 patients from eight medical centers who received at ment of hyponatremia in cirrhotic patients (128, 129). A retrospective
least two doses of tolvaptan found the mean tolvaptan dose to be study included 156 cirrhotic patients admitted for $5 days with
12.7 mg, with mean treatment time of 28 days with an average cor- sodium #133 mmol/L who were given octreotide compared with a
rection of sodium from baseline sodium of 123 mmol/L to discharge control group not given octreotide in relation to the change in
sodium of 133 mmol/L (121). A retrospective Japanese study sodium (130). The octreotide group had a lower baseline sodium
reviewed tolvaptan use in 586 SIAD patients (122). The starting compared with control (127 versus 128.5, P50.003). The results
dose was either 3.75 mg or 7.5 mg daily, and, interestingly, the time showed that octreotide administration was associated with a signifi-
to correction, averaging 4 days, was similar between the starting cant change of sodium at days 5 (6.6 versus 3.5, P,0.001), 7 (7.84
doses. The main difference was the higher overcorrection rate seen versus 4.33, P,0.001), and 10 (8 versus 5.2, P,0.001) (130).
in the 7.5-mg dose as compared with the 3.75-mg dose (122). Although the change in sodium was significant with octreotide, the
Another retrospective study reviewed tolvaptan use (dose ,15 mg/d) actual sodium level was significantly improved at days 5 (133 versus
in SIAD patients for an average of 27 months and found that it nor- 132, P50.003) and 7 (134 versus 132, P50.001) but not on day 10
malized sodium without any major side effects (98). A meta-analysis (135 versus 133, P50.075).
confirms the efficacy of hyponatremia treatment with tolvaptan; despite Treatment of medication-induced hyponatremia generally involves
the increased risk of overcorrection, no case of osmotic demyelination removing the culprit. The Association of Medicine and Psychiatry pub-
was seen (123). Compared to fluid restriction, 7.5 mg/day of tolvaptan lished an expert consensus on management recommendations (131).
was more effective in correcting hyponatremia in SIAD (124). Thus
long-term, low-dose tolvaptan may be a promising treatment, but more
prospective studies would need to be done. 4 Acute symptomatic hyponatremia is a medical
Tolvaptan is also approved for the treatment of hyponatremia emergency requiring 3% normal saline, whereas
in heart failure. Tolvaptan has been shown to decrease body weight,
increase urine volume, increase serum sodium, and reduce dyspnea,
chronic hyponatremia treatment can be accom-
but there is no reduction in mortality or rehospitalizations. Most plished through the appropriate diagnosis and
studies do not show any improvements in long-term mortality out- specific treatment of the etiology.
comes (42).
Other
Albumin and octreotide. A retrospective study evaluated two cir- References
rhotic cohorts; the first included 2414 cirrhotic patients admitted to 1. Adrogue HJ, Tucker BM, Madias NE: Diagnosis and management of
the hospital between 2010 and 2014, and the second cohort was hyponatremia: A review. JAMA 328: 280–291, 2022 PubMed
339 cirrhotic patients without a malignancy admitted to the hospital 2. Kaplon-Cieslicka A, Benson L, Chioncel O, Crespo-Leiro MG, Coats
in 2014–2021. The development and improvement of hyponatre- AJS, Anker SD, et al.: Hyponatraemia and changes in natraemia during
hospitalization for acute heart failure and associations with in-hospital
mia was evaluated as related to intravenous (i.v.) albumin infusions.
and long-term outcomes - From the ESC-HFA EORP Heart Failure
In both cohorts, the incidence of hyponatremia and the rate of Long-Term Registry. Eur J Heart Fail 25: 1571–1583, 2023 PubMed
improvement of hyponatremia were both significantly improved 3. Murashima M, Ambe K, Aoki Y, Kasugai T, Tomonari T, Ono M,
(125). Another study evaluated the long-term administration of et al.: Epidemiology and predictors of hyponatremia in a contemporary
albumin in patients with ascites. This was a post-hoc analysis of the cohort of patients with malignancy: A retrospective cohort study. Clin
ANSWER (Long-term albumin administration in decompensated Kidney J 16: 2072–2081, 2023 PubMed
cirrhosis) study database. The ANSWER trial evaluated the long- 4. Leveille E, Aljassar M, Beland B, Saeedi RJ, Marcoux J: Determinants
of hyponatremia following a traumatic brain injury. Neurol Sci 43:
term effects of albumin in patients with cirrhosis and uncomplicated 3775–3782, 2022 PubMed
ascites that required diuretic treatment. The treatment group con- 5. Kumar RV, Manchekar MP, Kashid M, Rajauria S, Rai SK: Does hypona-
sisted of standard medical treatment 1 IV albumin twice a week for tremia pose a risk factor for hip fractures in the elderly? Can a primary phy-
two weeks, then weekly, then monthly and the control group sician prevent it? J Family Med Prim Care 12: 1843–1848, 2023 PubMed
96 Nephrology Self-Assessment Program - Vol 23, No 2, June 2024
6. Kapoor M, Pathania M, Dhar M: Serum sodium improvement: Change 26. Hoenig MP, Lecker SH: Pattern recognition versus pathogenesis: Elec-
in comprehensive geriatric assessment parameters in geriatric patients trolytes in a patient with adrenal insufficiency. Clin J Am Soc Nephrol
with hyponatremia. BMC Geriatr 23: 666, 2023 PubMed 17: 585–587, 2022 PubMed
7. Tzoulis P, Yavropoulou MP: Association of hyponatremia with bone 27. Baez G, Chirio M, Pisula P, Seminario E, Carasa N, Philippi R, et al.:
mineral density and fractures: A narrative review. Ther Adv Endocrinol Hyponatremia and malnutrition: A comprehensive review. Ir J Med Sci
Metab 14: 20420188231197921, 2023 PubMed 193: 1043–1046, 2023 PubMed
8. Llorens P, Miro O, Veguillas Benito M, Fernandez C, Jacob J, Burillo- 28. de Haan L, Ten Wolde M, Beudel M, Olde Engberink RHG, Appel-
Putze G, et al.: Plasma sodium concentration in older patients as an indi- man B, Haspels-Hogervorst EK, et al.; Dutch COVID-PREDICT
cator of severity in emergencies: Results from the Emergency Department study group: What is the aetiology of dysnatraemia in COVID-19 and
and Elder Needs-16 study. Emergencias 35: 279–287, 2023 PubMed how is this related to outcomes in patients admitted during earlier and
9. Hoffmann DB, Popescu C, Komrakova M, Welte L, Saul D, Lehmann later COVID-19 waves? A multicentre, retrospective observational study
W, et al.: Chronic hyponatremia in patients with proximal femoral frac- in 11 Dutch hospitals. BMJ Open 13: e075232, 2023 PubMed
tures after low energy trauma: A retrospective study in a level-1 trauma 29. Pazos-Guerra M, Ruiz-Sanchez JG, Perez-Candel X, Lopez-Nevado C,
center. Bone Rep 12: 100234, 2020 PubMed Hernandez-Olmeda F, Cuesta-Hernandez M, et al.: Inappropriate therapy
10. Kapoor M, Dhar M, Pathania M: The assessment of baseline compre- of euvolemic hyponatremia, the most frequent type of hyponatremia in
hensive geriatric assessment parameters in geriatric patients with varying SARS-CoV-2 infection, is associated with increased mortality in COVID-
severity of hyponatremia at a tertiary care center. Cureus 14: e21516, 19 patients. Front Endocrinol (Lausanne) 14: 1227059, 2023 PubMed
2022 PubMed 30. Habas E, Ali E, Habas A, Rayani A, Ghazouani H, Khan F, et al.:
11. Krolicka A, Letachowicz K, Adamik B, Doroszko A, Kaliszewski K, Hyponatremia and SARS-CoV-2 infection: A narrative review. Medicine
Kilis-Pstrusi
nska K, et al.: Dysnatremia in COVID-19 patients–An (Baltimore) 101: e30061, 2022 PubMed
analysis of the COLOS study. J Clin Med 12: 2802, 2023 PubMed 31. Chu C-H, Chien W-C, Liu C-C, Chung C-H, Chen Y-C, Kuo F-C,
12. Ayus JC, Negri AL, Moritz ML, Lee KM, Caputo D, Borda ME, et al.: et al.: An enigma of hypothyroidism and hyponatremia coexistence: A
Hyponatremia, inflammation at admission, and mortality in hospital- nationwide population-based retrospective study. BMC Public Health
ized COVID-19 patients: A prospective cohort study. Front Med (Lau- 23: 1889, 2023 PubMed
sanne) 8: 748364, 2021 PubMed 32. Chen J: Is there a causal relationship between hypothyroidism and
13. Ng PY, Cheung RYT, Ip A, Chan WM, Sin WC, Yap DY-H, et al.: A ret- hyponatremia? Ther Adv Endocrinol Metab 14: 20420188231180983,
rospective cohort study on the clinical outcomes of patients admitted to 2023 PubMed
intensive care units with dysnatremia. Sci Rep 13: 21236, 2023 PubMed 33. Li L, Guo Y, Chen C, Wang Z, Liu Z: Mechanisms of hyponatremia
14. Khan A, Khan Z, Khan S, Ullah A, Ayub G, Tariq MN, et al.: Fre- and diabetes insipidus after acute spinal cord injury: A critical review.
quency of hyponatremia and its impact on prognosis in ischemic stroke. Chin Neurosurg J 9: 32, 2023 PubMed
Cureus 15: e40317, 2023 PubMed 34. Maesaka JK, Imbriano LJ, Grant C, Miyawaki N: High prevalence of
15. Deenadayalan V, Karki S, Patel B, Kumi D, Zia M: Association of hypo- renal salt wasting induced by haptoglobin-related protein without signal
natremia with clinical outcomes in patients admitted with acute pulmo- peptide is linked to new syndrome of salt wasting in Alzheimer disease.
nary embolism. J Thromb Thrombolysis 55: 737–741, 2023 PubMed Kidney Res Clin Pract 43: 33–46, 2023 PubMed
16. Sarastri Y, Zebua JI, Lubis PN, Zahra F, Lubis AC: Admission hypona- 35. Maesaka JK, Venkatesan J, Piccione JM, Decker R, Dreisbach AW,
traemia as heart failure events predictor in patients with acute heart fail- Wetherington J, et al.: Plasma natriuretic factor(s) in patients with
ure. ESC Heart Fail 10: 2966–2972, 2023 PubMed intracranial disease, renal salt wasting and hyperuricosuria. Life Sci 52:
17. Cole JH, Highland KB, Hughey SB, O’Shea BJ, Hauert T, Goldman 1875–1882, 1993 PubMed
AH, et al.: The association between borderline dysnatremia and periop- 36. Maesaka JK, Imbriano LJ, Pinkhasov A, Muralidharan R, Song X,
erative morbidity and mortality: Retrospective cohort study of the Russo LM, et al.: Identification of a novel natriuretic protein in patients
American College of Surgeons National Surgical Quality Improvement with cerebral-renal salt wasting–Implications for enhanced diagnosis.
Program Database. JMIR Perioper Med 6: e38462, 2023 PubMed Am J Med Sci 361: 261–268, 2021 PubMed
18. Pinter J, Genser B, Moissl U, Stuard S, Kooman J, Canaud B, et al.: 37. Murthy SB, Caplan J, Levy AP, Pradilla G, Moradiya Y, Schneider EB,
Hyponatraemia and fluid overload are associated with higher risk of et al.: Haptoglobin 2-2 genotype is associated with cerebral salt wasting
mortality in dialysis patients. Nephrol Dial Transplant 38: 2248–2256, syndrome in aneurysmal subarachnoid hemorrhage. Neurosurgery 78:
2023 PubMed 71–76, 2016 PubMed
19. Kim TH, Yim HJ, Jung YK, Song DS, Yoon EL, Kim HY, et al.: New 38. Rangan GK, Dorani N, Zhang MM, Abu-Zarour L, Lau HC, Munt A,
prognostic model for hospitalized patients with alcoholic cirrhosis and Mad- et al.: Clinical characteristics and outcomes of hyponatraemia associated
drey’s discriminant function ,32. Hepatol Int 18: 500–508, 2023 PubMed with oral water intake in adults: A systematic review. BMJ Open 11:
20. Singh P, Arora S, Singh D, Kalra S, Singh A, Arora U, et al.: Hypona- e046539, 2021 PubMed
tremia and outcome: Is severity more important than etiology? Cureus 39. Rondon-Berrios H, Velez JCQ: Hyponatremia in cirrhosis. Clin Liver
15: e42808, 2023 PubMed Dis 26: 149–164, 2022 PubMed
21. Rohrscheib M, Sam R, Raj DS, Argyropoulos CP, Unruh ML, Lew 40. Sorodoc V, Asaftei A, Puha G, Ceasovschih A, Lionte C, Sirbu O, et al.:
SQ, et al.: Edelman revisited: Concepts, achievements, and challenges. Management of hyponatremia in heart failure: Practical considerations.
Front Med (Lausanne) 8: 808765, 2021 PubMed J Pers Med 13:140, 2023 PubMed
22. Buchkremer F, Schuetz P, Mueller B, Segerer S: Classifying hypotonic 41. Kaplon-Cieslicka A, Soloveva A, Mareev Y, Cabac-Pogorevici I, Ver-
hyponatremia by projected treatment effects – A quantitative 3-dimensional brugge FH, Vardas P: Hyponatraemia in heart failure: Time for new
framework. Kidney Int Rep 8: 2720–2732, 2023 PubMed solutions? Heart 108: 1179–1185, 2022 PubMed
23. Zettl J, Jaafar A, Shourick J, Tack I, Vallet M: Alteration of the maxi- 42. Ishikawa SE, Funayama H: Hyponatremia associated with congestive
mal renal water excretion capacity with kidney function and age in heart failure: Involvement of vasopressin and efficacy of vasopressin
human. Int Urol Nephrol 56: 1429–1438, 2023 PubMed receptor antagonists. J Clin Med 12:1482, 2023 PubMed
24. Adrogue HJ, Madias NE: The syndrome of inappropriate antidiuresis. 43. Yalta K, Palabıyık O, Gurdogan M, Yetkın E: Hyponatremia and takot-
N Engl J Med 389: 1499–1509, 2023 PubMed subo syndrome: A review of pathogenetic and clinical implications.
25. Warren AM, Grossmann M, Christ-Crain M, Russell N: Syndrome of Heart Fail Rev 29: 27–44, 2023 PubMed
inappropriate antidiuresis: From pathophysiology to management. Endocr 44. Verbalis JG, Greenberg A, Burst V, Haymann J-P, Johannsson G, Peri
Rev 44: 819–861, 2023 PubMed A, et al.: Diagnosing and treating the syndrome of inappropriate
Nephrology Self-Assessment Program - Vol 23, No 2, June 2024 97
antidiuretic hormone secretion. Am J Med 129: 537.e9–537.e23, 2016 using the Japanese Adverse Drug Report (JADER) database. Medicina
PubMed (Kaunas) 59:166, 2023 PubMed
45. Verbalis JG, Goldsmith SR, Greenberg A, Korzelius C, Schrier RW, 67. Seethapathy H, Rusibamayila N, Chute DF, Lee M, Strohbehn I,
Sterns RH, et al.: Diagnosis, evaluation, and treatment of hyponatremia: Zubiri L, et al.: Hyponatremia and other electrolyte abnormalities in
Expert panel recommendations. Am J Med 126: S1–S42, 2013 PubMed patients receiving immune checkpoint inhibitors. Nephrol Dial Trans-
46. Spasovski G, Vanholder R, Allolio B, Annane D, Ball S, Bichet D; plant 36: 2241–2247, 2021 PubMed
Hyponatraemia Guideline Development Group, et al.: Clinical practice 68. Zhao Z, Zhao F, Jin P, Hu X, Tian C, Liu D, et al.: SSRI/SNRI-
guideline on diagnosis and treatment of hyponatraemia. Nephrol Dial induced hyponatremia: A case series of 26 patients in a single institution
Transplant 29 Suppl 2: i1–i39, 2014 PubMed from 2018 to 2020. Psychiatr Q 94: 113–125, 2023 PubMed
47. Hillier TA, Abbott RD, Barrett EJ: Hyponatremia: Evaluating the correc- 69. Petrykiv S, De Jonge L, Arts M: Hyponatremia associated with selective
tion factor for hyperglycemia. Am J Med 106: 399–403, 1999 PubMed serotonin-reuptake inhibitors. Eur Psychiatr 41: S758–S758, 2017
48. Aziz F, Sam R, Lew SQ, Massie L, Misra M, Roumelioti M-E, et al.: 70. Mannheimer B, Falhammar H, Calissendorff J, Skov J, Lindh JD:
Pseudohyponatremia: Mechanism, diagnosis, clinical associations and Time-dependent association between selective serotonin reuptake inhi-
management. J Clin Med 12:4076, 2023 PubMed bitors and hospitalization due to hyponatremia. J Psychopharmacol 35:
49. Kumar A, Ghosh M, Jacob JJ: Prevalence of adrenal insufficiency 928–933, 2021 PubMed
among patients with euvolemic hyponatremia. Endocr Connect 10: 71. Nagashima T, Hayakawa T, Akimoto H, Minagawa K, Takahashi U,
1623–1631, 2021 PubMed Asai S: Identifying antidepressants less likely to cause hyponatremia:
50. Sirin I, Cigsar G, Sonmez BM: The role of ultrasonographic inferior Triangulation of retrospective cohort, disproportionality, and pharmaco-
vena cava measurement in the volume-based classification of patients dynamic studies. Clin Pharmacol Ther 111: 1258–1267, 2022 PubMed
with hyponatremia. J Ultrasound Med 42: 2391–2401, 2023 PubMed 72. Yamamoto Y, Ohta A, Usui N, Imai K, Kagawa Y, Takahashi Y, et al.:
51. Rahman LR, Melson E, Alousi SA, Sardar M, Levy MJ, Shafiq S, et al.: Incidence trends and risk factors for hyponatremia in epilepsy patients:
Point-of-care ultrasound is a useful adjunct tool to a clinician’s assess- A large-scale real-world data study. Heliyon 9: e18721, 2023 PubMed
ment in the evaluation of severe hyponatraemia. Clin Endocrinol (Oxf) 73. Ciauskaite J, Gelziniene G, Jurkeviciene G: Oxcarbazepine and hypona-
100: 595–601, 2024 PubMed tremia. Medicina (Kaunas) 58:559, 2022 PubMed
52. Ruiz-Sanchez JG, Cuesta M, Gomez-Hoyos E, Cardenas-Salas J, Rubio- 74. El-Alali E, Al Jaber E: Association of proton pump inhibitor use and
Herrera MA, Martınez-Gonzalez E, et al.: Changes in serum creatinine significant hyponatremia–A US population-based case-control study.
levels can help distinguish hypovolemic from euvolemic hyponatremia. Proc (Bayl Univ Med Cent) 35: 434–436, 2022 PubMed
Medicina (Kaunas) 58: 851, 2022 PubMed 75. Nair HP, Kulkarni AR, Eswaran M, Subeesh V: Pantoprazole associated
53. Bassi V, Apuzzi V, Sodano M, Fattoruso O: Euvolemic hypotonic dyspepsia hypocalcemia and hyponatremia: A disproportionality analysis
hyponatremia in SIAD and thiazide-treated patients: Similarities and in FDA adverse event reporting system (FAERS) database. Arab J Gas-
differences. J Nephrol 37: 527–529, 2023 PubMed troenterol 24: 1–4, 2023 PubMed
54. Decaux G, Musch W: The urine calcium/creatinine ratio and uricemia 76. Issa I, Skov J, Falhammar H, Calissendorff J, Lindh JD, Mannheimer
during hyponatremia of different origins: Clinical implications. J Clin B, et al.: Time-dependent association between omeprazole and esome-
Med 12:723, 2023 PubMed prazole and hospitalization due to hyponatremia. Eur J Clin Pharmacol
55. Filippatos TD, Liamis G, Christopoulou F, Elisaf MS: Ten common 79: 71–77, 2023 PubMed
pitfalls in the evaluation of patients with hyponatremia. Eur J Intern 77. Madieh J, Hasan B, Khamayseh I, Hrizat A, Salah T, Nayfeh T, et al.:
Med 29: 22–25, 2016 PubMed The safety of intravenous peripheral administration of 3% hypertonic
56. Liamis G, Milionis H, Elisaf M: A review of drug-induced hyponatre- saline: A systematic review and meta-analysis. Am J Med Sci 366: 135–
mia. Am J Kidney Dis 52: 144–153, 2008 PubMed 142, 2023 PubMed
57. Filippone EJ, Ruzieh M, Foy A: Thiazide-associated hyponatremia: 78. Moritz ML, Ayus JC, Nelson JB: Administration of 3% sodium chloride
Clinical manifestations and pathophysiology. Am J Kidney Dis 75: 256– and local infusion reactions. Children (Basel) 9:1245, 2022 PubMed
264, 2020 PubMed 79. Juarez A, Barr M, Golden T: Hypertonic saline administration via
58. Mehta M, Goldfarb DS: Cumulative incidence of thiazide-induced intraosseous access during symptomatic hyponatremia. Cureus 15:
hyponatremia: A population-based cohort study. Ann Intern Med 177: e41731, 2023 PubMed
1–11, 2024 PubMed 80. Baek SH, Kim S: Optimal treatment with hypertonic saline in patients
59. Barber J, McKeever TM, McDowell SE, Clayton JA, Ferner RE, Gordon with symptomatic hyponatremia: A perspective from a randomized clin-
RD, et al.: A systematic review and meta-analysis of thiazide-induced ical trial (SALSA trial). Kidney Res Clin Pract 39: 504–506, 2020
hyponatraemia: Time to reconsider electrolyte monitoring regimens after PubMed
thiazide initiation? Br J Clin Pharmacol 79: 566–577, 2015 PubMed 81. Zurnuq S, Aljurfi M, Albekery M, Shawaqfeh MS, Elmubark A, Vasu-
60. Liamis G, Christidis D, Alexandridis G, Bairaktari E, Madias NE, Elisaf devan S, et al.: Hyponatremia management among patients admitted to
M: Uric acid homeostasis in the evaluation of diuretic-induced hypona- tertiary hospital: A retrospective evaluation. SAGE Open Med 10:
tremia. J Investig Med 55: 36–44, 2007 PubMed 20503121221095333, 2022 PubMed
61. Kim GH: Pathophysiology of drug-induced hyponatremia. J Clin Med 82. Pelouto A, Refardt JC, Christ-Crain M, Zandbergen AAM, Hoorn EJ:
11:5810, 2022 PubMed Overcorrection and undercorrection with fixed dosing of bolus hyper-
62. Fibbi B, Marroncini G, Naldi L, Anceschi C, Errico A, Norello D, et al.: tonic saline for symptomatic hyponatremia. Eur J Endocrinol 188: 322–
Hyponatremia and cancer: From bedside to benchside. Cancers (Basel) 330, 2023 PubMed
15:1197, 2023 PubMed 83. Massop K, Haverkort DA, Bech AP, de Boer H: NaCl 3% bolus ther-
63. Bruining DM, van Roon EN, de Graaf H, Hoogendoorn M: Cyclo- apy as emergency treatment for severe hyponatremia: Comparison of
phosphamide-induced symptomatic hyponatraemia. Neth J Med 69: 100 ml vs 250 ml. J Clin Endocrinol Metab 108: e521–e526, 2023
192–195, 2011 PubMed PubMed
64. Pelletier K, Skrtic M, Kitchlu A: Cancer therapy-induced hyponatremia: 84. Kinoshita T, Mlodzinski E, Xiao Q, Sherak R, Raines NH, Celi LA,
A case-illustrated review. J Onco-Nephrol 5: 70–78, 2021 PubMed et al.: Effects of correction rate for severe hyponatremia in the intensive
65. Kim S, Jo CH, Kim GH: The role of vasopressin V2 receptor in drug- care unit on patient outcomes. J Crit Care 77: 154325, 2023 PubMed
induced hyponatremia. Front Physiol 12: 797039, 2021 PubMed 85. Sumi H, Imai N, Shibagaki Y: Incidence and risk factors of overcorrec-
66. Tamura N, Ishida T, Kawada K, Jobu K, Morisawa S, Yoshioka S, et al.: tion in patients presenting with severe hyponatremia to the emergency
Risk factors for anticancer drug-induced hyponatremia: An analysis department. Clin Exp Nephrol 26: 1086–1091, 2022 PubMed
98 Nephrology Self-Assessment Program - Vol 23, No 2, June 2024
86. George JC, Zafar W, Bjcaloiu ID, Chang AR: Risk factors and out- 106. Refardt J, Imber C, Sailer CO, Jeanloz N, Potasso L, Kutz A, et al.: A
comes of rapid correction of severe hyponatremia. Clin J Am Soc randomized trial of empagliflozin to increase plasma sodium levels in
Nephrol 13: 984–992, 2018 PubMed patients with the syndrome of inappropriate antidiuresis. J Am Soc
87. Yang H, Yoon S, Kim EJ, Seo JW, Koo J-R, Oh YK, et al.: Risk factors Nephrol 31: 615–624, 2020 PubMed
for overcorrection of severe hyponatremia: A post hoc analysis of the 107. Refardt J, Imber C, Nobbenhuis R, Sailer CO, Haslbauer A, Monnerat
SALSA trial. Kidney Res Clin Pract 41: 298–309, 2022 PubMed S, et al.: Treatment effect of the SGLT2 inhibitor empagliflozin on
88. Bastos AP, Rocha PN: Osmotic demyelination as a complication of chronic syndrome of inappropriate antidiuresis: Results of a random-
hyponatremia correction: A systematic review. J Bras Nefrol 46: 47–55, ized, double-blind, placebo-controlled, crossover trial. J Am Soc Nephrol
2023 PubMed 34: 322–332, 2023 PubMed
89. Tandukar S, Sterns RH, Rondon-Berrios H: Osmotic demyelination 108. Yeoh SE, Docherty KF, Jhund PS, Petrie MC, Inzucchi SE, K€ober L,
syndrome following correction of hyponatremia by ,/510 mEq/L per et al.: Relationship of dapagliflozin with serum sodium: Findings from
day. Kidney360 2: 1415–1423, 2021 PubMed the DAPA-HF trial. JACC Heart Fail 10: 306–318, 2022 PubMed
90. Arshad MF, Iqbal A, Weeks J, Fonseca I, Munir A, Bennet W: Hyper- 109. Charaya K, Shchekochikhin D, Agadzhanyan A, Vashkevich M, Chas-
tonic saline for severe symptomatic hyponatraemia: Real-world findings hkina M, Kulikov V, et al.: Impact of dapagliflozin treatment on serum
from the UK. Endocr Connect 11: e220007, 2022 PubMed sodium concentrations in acute heart failure. Cardiorenal Med 13: 101–
91. MacMillan TE, Shin S, Topf J, Kwan JL, Weinerman A, Tang T, et al.: 108, 2023 PubMed
Osmotic demyelination syndrome in patients hospitalized with hypona- 110. Ruocco G, Verbrugge FH, Nuti R, Palazzuoli A: Hyponatremia in acute
tremia. NEJM Evid 2: EVIDoa2200215, 2023 PubMed heart failure in relation to hematocrit levels: Clinical relevance and prog-
92. Ayus JC, Moritz ML: Hyponatremia treatment guidelines—Have they nostic implication. Cardiorenal Med 8: 259–270, 2018 PubMed
gone too far? NEJM Evid 2: EVIDe2300014, 2023 PubMed 111. Monnerat S, Atila C, Refardt J, Christ-Crain M: Prevalence of admis-
93. Sterns RH, Rondon-Berrios H, Adrogue HJ, Berl T, Burst V, Cohen DM; sion hyponatremia in patients with diabetes treated with and without
PRONATREOUS Investigators, et al.: Treatment guidelines for hyponatre- an SGLT2 inhibitor. J Endocr Soc 7: bvad011, 2023 PubMed
mia: Stay the course. Clin J Am Soc Nephrol 19: 129–135, 2023 PubMed 112. Woudstra J, de Boer MP, Hempenius L, van Roon EN: Urea for hypo-
94. Rondon-Berrios H: Diagnostic and therapeutic strategies to severe natraemia due to the syndrome of inappropriate antidiuretic hormone
hyponatremia in the intensive care unit [published online ahead of print secretion. Neth J Med 78: 125–131, 2020 PubMed
Oct 11, 2023]. J Intensive Care Med doi:10.1177/08850666231207334
113. Perello-Camacho E, Pomares-Gomez FJ, Lopez-Penabad L, Mirete-
2023 PubMed
Lopez RM, Pinedo-Esteban MR, Domınguez-Escribano JR, et al.: Clin-
95. Rondon-Berrios H: Therapeutic relowering of plasma sodium after
ical efficacy of urea treatment in syndrome of inappropriate antidiuretic
overly rapid correction of hyponatremia: What is the evidence? Clin J
hormone secretion. Sci Rep 12: 10266, 2022 PubMed
Am Soc Nephrol 15: 282–284, 2020 PubMed
114. Martınez Gonzalez AM, Rodeiro Escobar P, Llopiz Castedo J, Dıaz
96. Nagase K, Watanabe T, Nomura A, Nagase FN, Iwasaki K, Nakamura
Vazquez M, Sanchez Juanas FL, Villar Carballo M, et al.: Effectiveness
Y, et al.: Predictive correction of serum sodium concentration with for-
of urea administration for the treatment of hyponatremia in heart fail-
mulas derived from the Edelman equation in patients with severe hypo-
ure. Med Clin (Engl) 162: 56–59, 2023 PubMed
natremia. Sci Rep 13: 1783, 2023 PubMed
115. Hammonds WM, Keating EA, Smetana ME, Smetana KS, Bond MM:
97. Lawless SJ, Thompson C, Garrahy A: The management of acute
Safety and efficacy of urea for hyponatremia. Hosp Pharm 57: 365–369,
and chronic hyponatraemia. Ther Adv Endocrinol Metab 13:
20420188221097343, 2022 PubMed 2022 PubMed
98. Bondanelli M, Aliberti L, Gagliardi I, Ambrosio MR, Zatelli MC: 116. Wendt R, Fenves AZ, Geisler BP: Use of urea for the syndrome of
Long-term low-dose tolvaptan efficacy and safety in SIADH. Endocrine inappropriate secretion of antidiuretic hormone: A systematic review.
82: 390–398, 2023 PubMed JAMA Netw Open 6: e2340313, 2023 PubMed
99. Monnerat S, Refardt J, Potasso L, Meier C, Christ-Crain M: An 117. Monnerat S, Atila C, Baur F, Santos de Jesus J, Refardt J, Dickenmann
increase in plasma sodium levels is associated with an increase in osteo- M, et al.: Effect of protein supplementation on plasma sodium levels in
blast function in chronic SIAD. J Clin Endocrinol Metab 108: e1027– the syndrome of inappropriate antidiuresis: A monocentric, open-label,
e1033, 2023 PubMed proof-of-concept study–The TREASURE study. Eur J Endocrinol 189:
100. Suarez V, Picotin R, Fassbender R, Gramespacher H, Haneder S, Persi- 252–261, 2023 PubMed
gehl T, et al.: Chronic hyponatremia and brain structure and function 118. Martinez A, Rodrıguez A, Corral M, Reyes E, Rodrıguez S: Hyponatre-
before and after treatment [published online ahead of print Jan 4, mia treatment with oral urea in heart failure. Endocrinol Diabetes Nutr
2024]. Am J Kidney Dis doi:10.1053/j.ajkd.2023.11.007 2024 PubMed (Engl Ed) 69: 303–304, 2022 PubMed
101. Furst H, Hallows KR, Post J, Chen S, Kotzker W, Goldfarb S, et al.: 119. Colombo F, Milani M: Palatability of two different formulations of
The urine/plasma electrolyte ratio: A predictive guide to water restric- urea for the treatment of hyponatremia: A double-blind, randomized,
tion. Am J Med Sci 319: 240–244, 2000 PubMed cross-over study. Clin Nutr ESPEN 58: 350–354, 2023 PubMed
102. Garrahy A, Galloway I, Hannon AM, Dineen R, O’Kelly P, Tormey 120. Cukoski S, Osterholt T, Suarez V, Hackl MJ, Grundmann F, Burst V,
WP, et al.: Fluid restriction therapy for chronic SIAD; results of a pro- et al.: Sodium evolution in hyponatraemia: A mixed effects model anal-
spective randomized controlled trial. J Clin Endocrinol Metab 105: ysis of the Hyponatraemia Registry. Eur J Endocrinol 188: 526–535,
dgaa619, 2020 PubMed 2023 PubMed
103. Warren AM, Grossmann M, Hoermann R, Zajac JD, Russell N: Tolvap- 121. Pose-Reino A, Runkle de la Vega I, de Jong-Laird A, Kabra M, Lindner U:
tan versus fluid restriction in acutely hospitalised patients with moderate- Real-world, non-interventional, retrospective study (SAMPLE) of tolvaptan
profound hyponatraemia (TVFR-HypoNa): Design and implementation in patients with hyponatraemia secondary to the syndrome of inappropriate
of an open-label randomised trial. Trials 23: 335, 2022 PubMed antidiuretic hormone secretion. Adv Ther 38: 1055–1067, 2021 PubMed
104. Krisanapan P, Vongsanim S, Pin-On P, Ruengorn C, Noppakun K: 122. Hagiwara D, Matsukawa M, Tasaki J, Nakamura Y, Arima H: A retro-
Efficacy of furosemide, oral sodium chloride, and fluid restriction spective study on tolvaptan prescription in clinical practice in patients
for treatment of syndrome of inappropriate antidiuresis (SIAD): An with syndrome of inappropriate secretion of antidiuretic hormone
open-label randomized controlled study (The EFFUSE-FLUID trial). (SIADH) using the Japanese claims database. Endocr J 70: 1195–1205,
Am J Kidney Dis 76: 203–212, 2020 PubMed 2023 PubMed
105. Tang H, Xu C, Zhang P, Luo T, Huang Y, Yang X: A profile of 123. Krisanapan P, Tangpanithandee S, Thongprayoon C, Pattharanitima P,
SGLT-2 inhibitors in hyponatremia: The evidence to date. Eur J Pharm Kleindienst A, Miao J, et al.: Safety and efficacy of vaptans in the treat-
Sci 184: 106415, 2023 PubMed ment of hyponatremia from syndrome of inappropriate antidiuretic
Nephrology Self-Assessment Program - Vol 23, No 2, June 2024 99
hormone secretion (SIADH): A systematic review and meta-analysis. J 129. Bai Z, Mendez-Sanchez N, Romeiro FG, Mancuso A, Philips CA,
Clin Med 12:5483, 2023 PubMed Tacke F, et al.: Use of albumin infusion for cirrhosis-related complica-
124. Kleindienst A, Georgiev S, Schlaffer SM, Buchfelder M: Tolvaptan versus tions: An international position statement. JHEP Rep 5: 100785, 2023
fluid restriction in the treatment of hyponatremia resulting from SIADH PubMed
following pituitary surgery. J Endocr Soc 4: bvaa068, 2020 PubMed 130. Ismail B, Charnigo R, Ali SM, Alkhairi B, Benrajab K, Singh H, et al.:
125. Bai Z, Xu W, Chai L, Zheng X, Mendez-Sanchez N, Philips CA, et al.: Impact of octreotide on sodium level in cirrhotic inpatients with hypo-
Effects of short-term human albumin infusion for the prevention and natremia: A retrospective study. Eur J Gastroenterol Hepatol 35: 1394–
treatment of hyponatremia in patients with liver cirrhosis. J Clin Med 1401, 2023 PubMed
12:107, 2022 PubMed 131. Pinkhasov A, Xiong G, Bourgeois JA, Heinrich TW, Huang H, Cor-
126. Zaccherini G, Baldassarre M, Tufoni M, Nardelli S, Piano S, Alessandria C, iolan S, et al.: Management of SIADH-related hyponatremia due to
et al.; ANSWER Study Investigators: Correction and prevention of hypona- psychotropic medications - An expert consensus from the Association
tremia in patients with cirrhosis and ascites: Post hoc analysis of the of Medicine and Psychiatry. J Psychosom Res 151: 110654, 2021
ANSWER study database. Am J Gastroenterol 118: 168–173, 2023 PubMed PubMed
127. Zheng X, Bai Z, Wang T, Romeiro FG, Mancuso A, Philips CA, et al.: 132. Hoorn EJ, Zietse R: Diagnosis and treatment of hyponatremia: Compi-
Human albumin infusion for the management of liver cirrhosis and its lation of the guidelines. J Am Soc Nephrol 28: 1340–1349, 2017
complications: An overview of major findings from meta-analyses. Adv PubMed
Ther 40: 1494–1529, 2023 PubMed 133. Galindo DEB, Ruiz-Sanchez JG, Martınez AF, Runkle de la Vega I,
128. Bai Z, Wang L, Lin H, Tacke F, Cheng G. Qi X: Use of human albu- Ferrer Garcıa JC, Ropero-Luis G, et al.: Consensus document on the
min administration for the prevention and treatment of hyponatremia management of hyponatraemia of the Acqua Group of the Spanish
in p,tients with liver cirrhosis: A systematic review and meta-analysis. Society of Endocrinology and Nutrition. Endocrinol Diabetes Nutr (Engl Ed)
J Clin Med 11:5928, 2022 PubMed 70: 7–26, 2023 PubMed
100 Nephrology Self-Assessment Program - Vol 23, No 2, June 2024
Article
Hypernatremia: Pathophysiology, Evaluation, Treatment,
and Updates
Madhusudan Vijayan, MBBS
Barbara T. Murphy Division of Nephrology, Department of Medicine, Icahn School of Medicine at Mount Sinai
Hospital, New York, New York
Institute for Critical Care Medicine, Icahn School of Medicine at Mount Sinai Hospital, New York, New York
Joshua Rein, DO, FASN
Barbara T. Murphy Division of Nephrology, Department of Medicine, Icahn School of Medicine at Mount Sinai
Hospital, New York, New York
Renal Division, Department of Medicine, James J. Peters Veterans Affairs Medical Center, Bronx, New York
Prevalence of Hypernatremia
Hypernatremia will not develop if thirst sensation is intact and the
individual has access to water. Hence, individuals who are at
increased risk of developing hypernatremia include anyone with
impaired access to water, such as infants or adults with dementia,
immobility, or physical disabilities. Among patients with CKD, a
urinary concentration defect may predispose to hypernatremia from
urinary water losses. In fact, an electronic health record–based study
of 655,493 United States veterans with non–dialysis-dependent
CKD reported a plasma sodium of 145–149.9 mEq/L in 12,807
(1.9%) patients and $150 mEq/L in 482 (0.07%) patients. They
also reported increased odds of mortality in patients having serum
sodium 145.1–150 mEq/L (hazard ratio [HR] 1.33; 95% confidence
interval [95% CI] 1.28 to 1.38) and serum sodium .150 mEq/L
(HR 1.56; 95% CI, 1.33 to 1.83) compared with patients with a
serum sodium between 136 and 145 mEq/L (8). Because of the lack
of water intake while sleeping, early morning hypernatremia may be a
marker of a urinary concentration defect and subclinical CKD,
although not studied (9). The prevalence of hypernatremia in outpati-
Figure 1. (A) Physiologic release of vasopressin. (1) Osmosensory ents with stable CKD is low, because thirst sensation and access to
neurons in the subfornical organ (SFO) and organum vasculosum free water likely compensates for any urinary concentrating defect.
lamina terminalis (OVLT) are activated by increased serum osmolal- Advanced age is associated with increased odds of hypernatremia in
ity causing thirst stimulation and hypothalamic production of pre- the CKD population, similar to the general population (8).
provasopressin. (2) Preprovasopressin undergoes cleavage to form Among hospitalized patients, the prevalence of hypernatremia
vasopressin, copeptin, and neurophysin II, which are copackaged in ranges between 1% and 4% (10). A recent retrospective single-
vesicles during axonal transport. (3) Vasopressin is released from center study of 1.9 million patients hospitalized between 2000 and
axon terminals into the systemic bloodstream. (B) Arginine vasopres- 2018 showed that 3% of inpatients presented with a plasma sodium
sin deficiency. (1 and 2) Acquired injury to the cell bodies of neurons concentration .145 mEq/L. The incidence of in-hospital mortality
that produce PPV in the supraoptic nucleus (SON) and paraventri- was 12% in hypernatremic and 2% in normonatremic patients (10).
cular nucleus (PVN), or to neuronal axons in the pituitary stalk, will Patients with hypernatremia were significantly older than normona-
lead to decreased or no production of vasopressin. (3) Defects in vesi- tremic patients (65.5618.7 versus 57.9619.2 years, respectively;
cle exocytosis at the axon terminal impair vasopressin secretion into P,0.001), and the odds of in-hospital mortality also increased
the systemic circulation. Reprinted from ref. 7, with permission from with age. There is also growing interest in the association between
Elsevier. Central diabetes insipidus has been replaced with arginine AKI and hypernatremia. A retrospective study of 160 hospitalized
vasopressin deficiency to accommodate change in nomenclature. patients with de novo AKI (detected using an in-hospital alert sys-
tem) showed that inpatients who died had significantly higher
as the antidiuretic hormone. These vesicles are released from the pos- sodium levels at AKI onset compared with survivors (145.762.13
terior pituitary into the circulation. versus 138.860.636 mEq/L, respectively; P50.003) (11). Hyperna-
tremia at AKI onset may serve as a potential prognostic marker of
Activation of Water Reabsorption in the Distal Nephron mortality. Critically ill patients admitted to the ICU may have a
Circulating AVP acts on vasopressin type 2 receptors (V2R) on the higher likelihood of developing hypernatremia. ICU patients devel-
basolateral surface of principal cells in the collecting duct, which oping hypernatremia have complex pathophysiology and multiple
102 Nephrology Self-Assessment Program - Vol 23, No 2, June 2024
Effects of Hypernatremia
Although the entire body is exposed to the effects of elevated plasma
osmolality, the principal organ affected is the brain, because neuron
and glial cell function is very sensitive to changes in cellular volume
from osmolar shifts. In a hypernatremic extracellular environment,
intracellular water is lost and cells shrink, followed by an adaptive
intracellular accumulation of organic and inorganic osmolytes to
restore normal cell volume (5). This process is termed as regulatory
volume increase, which is functionally the opposite effect of the
regulatory volume decrease seen as an adaptive response to hypona-
tremia or resolving hypernatremia (5). Such regulatory volume
changes are important neurologic defenses against osmolar shifts.
However, it is unclear how long it will take for such brain adapta-
tion. Acute increases in plasma sodium concentrations may result in
cellular shrinkage or even intracranial hemorrhage, resulting in signs
and symptoms that may be mild, such as lethargy, weakness, irrita-
bility, and twitching, or severe such, as seizures and coma (13).
Osmotic demyelination syndrome with features of central pon-
tine and/or extrapontine myelinolysis is being increasingly reported
among hypernatremic patients. In a case series of 30 patients who
developed central nervous system injuries from hypernatremia,
osmotic demyelination syndrome was reported in 80% (n524) of
cases, with symptoms ranging from altered mental status (n523) to
Figure 2. (A) Physiologic effects of vasopressin on principal cells. motor deficit such as quadriparesis or quadriplegia (n520) to myo-
Vasopressin stimulates V2R that activates intracellular signaling clonic or secondary generalized tonic-clonic seizures (n513), and
by activating adenylate cyclase to increase intracellular cyclic cranial nerve dysfunction (n513) (14). The initial Na level at pre-
adenosine monophosphate (cAMP). cAMP promotes protein sentation ranged from 142 to 200 mEq/L (mean 175.1616.8 mEq/L).
kinase A (PKA)-mediated AQP2 gene transcription. Subse- Among these 30 cases, eight (26.7%) patients had satisfactory recov-
quently, intracellular vesicles that store AQP2 channels are fused ery, 17 patients (56.7%) had persistent neurologic deficits with signif-
with the apical membrane, allowing for transcellular water reab- icant morbidities, and five patients (16.7%) died within 12 hours to
sorption. (B) Arginine vasopressin deficiency. Because of a defect 3 weeks of presentation (14).
Nephrology Self-Assessment Program - Vol 23, No 2, June 2024 103
Figure 3. Hypernatremia can develop from renal and extrarenal sodium gains and/or water loss that are out of proportion to water intake. Water
loss occurs through the respiratory, integumentary, genitourinary, and gastrointestinal systems that are in contact with the external environment.
Reprinted from ref. 7, with permission from Elsevier. Diabetes insipidus has been replaced by AVP-related polyuria to accommodate change in
nomenclature.
104 Nephrology Self-Assessment Program - Vol 23, No 2, June 2024
impervious, such as the ascending limb of the loop of Henle. If osmotic diuresis (20). Urine sodium and potassium concentrations
osmoregulation is intact, thirst and free-water reabsorption in the also help us in calculating urinary electrolyte-free water clearance,
collecting duct mediated by arginine vasopressin (AVP) should pre- which may reflect the serum sodium trajectory (i.e., excessive urinary
vent the development of hypernatremia. Hence, hypernatremia can free-water clearance would be associated with worsening hypernatre-
develop in conditions with excessive renal sodium reabsorption only mia). If an AVP-related disorder is suspected, further evaluation is
when it is not accompanied by a proportional gain of free water, required and is discussed in detail below.
that is, lack of access to free water, presence of a urinary concentrat-
ing defect, or a reset osmostat. The mild hypernatremia (approxi-
mately 145 mEq/L) seen in primary aldosteronism is believed to be Arginine Vasopressin–Related Polyuria
secondary to a reset osmostat; that is, a higher level of plasma osmo- Diabetes insipidus (DI) occurs because of a defect in physiologic
lality is required to stimulate AVP secretion, which may be a protec- osmoregulation through the arginine vasopressin/aquaporin 2 (AVP/
tive mechanism in the setting of hypertension and mildly expanded AQP2) pathway, manifesting as an aquaretic polyuria independent
extracellular volume (16). Cushing syndrome, which is also associ- of plasma osmolality. It is caused by a defect in AVP secretion or
ated with excess mineralocorticoid activity, is also reported to cause action, or its accelerated degradation (Table 1). Recently, an interna-
hypernatremia, which may be mediated by the same effect (17). tional group of experts from endocrinology, nephrology, and pediat-
AKI is often associated with hypernatremia due to positive sodium rics have proposed changing the name of DI to arginine vasopressin
balance from reduced renal excretion of sodium with or without a deficiency (AVP-D) and arginine vasopressin resistance (AVP-R),
urinary concentration defect (11). respectively, for central and nephrogenic etiologies, to avoid confu-
sion with the more common term “diabetes” that is used to refer to
Hypernatremia from Renal Water Loss diabetes mellitus (21). Subsequently, the Systematized Nomencla-
ture of Medicine–Clinical Terminology February 2024 edition
Hypernatremia may develop from renal loss of electrolyte-free water
renamed DI as arginine vasopressin–related polyuria with two speci-
from osmotic diuresis or defects in the AVP/ aquaporin 2 (AQP2)
fic subtypes: AVP-D and AVP-R (22). In a recent survey of 1034
signaling pathway. Although low urine osmolality may be useful in
patients with central DI (CDI), 823 (80%) patients encountered
identifying renal water losses, urine osmolality may be high in some
a situation where a healthcare professional confused CDI with
cases, because ineffective osmoles in the urine contribute signifi-
diabetes mellitus, and 884 (85%) patients supported renaming the
cantly to urine osmolality but increase electrolyte-free water loss and
disease (23).
tend to raise plasma sodium concentrations. Resolving acute tubular
necrosis (ATN) or urinary obstruction may lead to hypernatremia
from urinary loss of “ineffective” uremic osmoles (18). Disorders of
4 The Systematized Nomenclature of Medicine–
AVP related renal water loss are discussed separately below (Arginine
Vasopressin–Related Polyuria). Clinical Terminology February 2024 edition renamed
diabetes insipidus as arginine vasopressin-related
polyuria with two specific subtypes: arginine
Evaluation of Hypernatremia
The diagnostic approach to hypernatremia requires a thorough his-
vasopressin deficiency and arginine vasopressin
tory and physical examination. History should inquire about thirst resistance.
sensation, access to water, urinary volume, medication list, and past
medical history. A careful physical examination must include the
assessment of hydration and volume status, orthostatic vital signs if AVP-D can occur because of failure of the osmosensory regions
possible, and cardiopulmonary and mental status examination. to sense hyperosmolality or an inherited or acquired impairment in
Plasma osmolality may be presumed to be high in most cases and is AVP synthesis, processing, neuronal trafficking, or secretion (Figure 1).
often not necessary to make the diagnosis. Exceptions include when An impairment in osmoreceptors may also affect the thirst response;
pseudonormonatremia is suspected (in the setting of hypertriglyceri- hence this condition has been termed as adipsic DI (or more appropri-
demia, hyperparaproteinemia, or hyperglycemia) or when there is ately as adipsic AVP-D) (24). Although this is a rare condition, the
rapidly rising serum glucose and/or urea levels, which may also con- lack of thirst response predisposes these patients to the rapid develop-
tribute to encephalopathy. Although urea is an ineffective osmole, ment of severe hypernatremia and associated neurologic complications.
its movement across the blood-brain barrier is more restricted, and However, most cases with AVP-D occur from abnormalities in the
hence rapid fluctuations in urea levels may also lead to osmolar shifts synthesis and release of AVP, resulting in aquaresis and increase in
(19). Urine osmolality should be checked in all patients with hyper- serum osmolality, which is sensed by intact osmosensory function
natremia. In most cases, hypernatremia must lead to a maximally resulting in thirst (polydipsic AVP-D). AVP-D can be transient or per-
concentrated urine (.700 mOsm/kg) if osmoregulation is intact, manent and vary in severity, which is either complete (nearly 100%
thus indicating extrarenal water losses or electrolyte gains (20). If defect) or partial AVP-D (some capacity for AVP production, although
urine is very dilute (,300 mOsm/kg), an arginine vasopressin inadequate).
(AVP)-related disorder must be suspected. For urine osmolality Gestational DI occurs because of placental production of vaso-
between 300 and 600 mOsm/kg, the diagnosis may include either pressinase, which begins at week 7 of pregnancy and increases
an AVP-related disorder or osmotic diuresis (20). A 24-hour urinary with placental growth. At peak levels, circulating AVP is reduced
osmolar excretion .1000 Osm/day in this situation may indicate by 80%–85%, resulting in submaximal and inadequate water
Nephrology Self-Assessment Program - Vol 23, No 2, June 2024 105
Table 1. Overview of the causes of arginine vasopressin deficiency, degradation, and resistance
conservation, this manifesting as polyuria (7). Because of an A 24-hour urine collection is often required to confirm and accu-
intact thirst mechanism, polydipsia compensates for urinary rately quantify the polyuria. Once the degree of polyuria is deter-
water loss, and normonatremia is usually maintained. Hence, mined, the next step is to assess the urinary concentrating ability
gestational DI is usually a self-limited condition, which resolves and evaluate for excess urinary osmolar excretion. Polyuria may
4–6 weeks after delivery (7). However, in patients with impaired develop classically because of an osmotic diuresis (from salt, glucose,
access to water or underlying subclinical forms of AVP deficiency nitrogenous substances such as urea and dietary proteins, or exoge-
and/or resistance, placental secretion of vasopressinase may nous substances such as mannitol) or aquaresis. An elevated urine
result in hypernatremia and clinical symptoms and signs of osmolality .600 mOsm/kg or .1000 mOsm/day is suggestive of
dehydration (25). an osmotic diuresis, whereas urine osmolality ,300 mOsm/kg
AVP-R arises because of a kidney defect in either vasopressin (often ,200 mOsm/kg) is suggestive of aquaresis from either pri-
type 2 receptors (V2R) or AQP2 function (Figure 2). Despite mary polydipsia or AVP-related polyuria (20). The presence of
appropriate osmosensory function and release of AVP, osmoregula- hypernatremia favors AVP-related polyuria over primary polydipsia.
tion fails because of impairments in urinary water conservation. However, differentiating between the two is complicated for the fol-
Similar to AVP-D, AVP-R can also vary in duration and severity lowing reasons:
(complete or partial) depending upon the renal response to V2R 1. Patients with AVP-related polyuria have low urine osmolality
activation from AVP or its synthetic analog 1-deamino-8-D-AVP and may have normal plasma osmolality if they have ade-
(desmopressin). The most common cause of acquired AVP-R is quate water intake.
chronic lithium use in the setting of bipolar disorder, which may be 2. Patients with primary polydipsia may have normal plasma
irreversible in some patients. osmolality and low urine osmolality, due to appropriate
osmosensation and AVP suppression.
Diagnosis of Arginine Vasopressin–Related Polyuria 3. Although uncommon, patients with primary polydipsia may
become hypernatremic during periods of severe prolonged
Clinically, AVP-related polyuria falls within the umbrella of water restriction.
polydipsia-polyuria syndromes, which also includes primary polydip- 4. Patients with chronic primary polydipsia may have blunting
sia and osmotic diuresis. Polyuria is generally defined as urine of renal medullary hypertonicity and downregulation of
output .3 L per day in adults, may also be defined as .50 ml/kg AQP2, resulting in aquaresis despite the action of AVP, which
per day, which may be more useful at extremes of weight (26). may mimic AVP-R (27).
106 Nephrology Self-Assessment Program - Vol 23, No 2, June 2024
5. Patients may have partial forms of AVP-D or AVP-R, which inhibitors, desmopressin, nonsteroidal anti-inflammatory drugs, gluco-
leads to partially retained ability to concentrate the urine when corticoids, carbamazepine, and chlorpropamide) (27). Baseline plasma
required. However, these cases are more difficult to diagnose osmolality, plasma sodium concentration, copeptin levels, and urine
and may only manifest during periods of excess water loss osmolality are obtained, and the water deprivation may be started over-
and/or inadequate water intake during stress or exertion.
night. The patient needs to be on strict nil per os measures throughout
As indicated above, the major differentiating factor for AVP- the test. After the overnight phase, morning samples are collected with
related polyuria from primary polydipsia is the failure of osmoregu- the same measurements, and repeated every 2 hours, while the patient
lation as the primary factor resulting in polyuria (i.e., the presence remains nil per os. If there are symptoms of dizziness, hypotension,
of an inappropriately low urine osmolality despite high plasma intractable thirst, seizures, severe hypernatremia (.150 mEq/L), or
osmolality). This failure of osmoregulation can be detected by any loss of .3% of body weight, further testing may be canceled (27). If
of the following three tests: indirect water deprivation test, direct water restriction alone fails to achieve a required sodium concentration
water deprivation test, and arginine- or hypertonic saline-stimulated $145 mEq/L and plasma osmolality .295 mOm, then 2% or 3%
copeptin measurement. sodium chloride (hypertonic saline) may be infused.
The indirect water deprivation test (Figure 4), described in the If urine osmolality increases to .800 mOs/kg, urine concentrat-
1970s, has been the diagnostic gold standard for AVP-related polyuria, ing ability is intact, osmoregulatory failure is ruled out, and the diag-
but newer tests are being developed that may have superior accuracy nosis is probably primary polydipsia (although mild partial AVP-D
(28). The basic premise behind the water deprivation test is that hyper- cannot fully be ruled out). If urine osmolality is ,300 mOsm/kg,
natremia .145 mEq/L and hyperosmolality .295 mOsm/kg should there is evidence of complete osmoregulatory failure, and AVP-related
stimulate the release of AVP in patients with intact osmoregulatory polyuria can be diagnosed. The next step in such cases is to determine
mechanisms, leading to urinary concentration. In patients with AVP- the ability of the kidney to respond to V2R activation with the
related polyuria, where the osmoregulatory mechanisms are impaired, administration of desmopressin 2 mcg (preferably intravenously) and
there is a lack of urinary concentration, which is pathologic from either serial hourly measurements of urine and plasma osmolality. While a
reduced AVP secretion (AVP-D) or AVP action (AVP-R). The term doubling of urine osmolality is suggestive of complete AVP-D, an
“indirect” refers to using urinary concentration as a surrogate to infer increase in urine osmolality of less than 15% is suggestive of complete
AVP action, rather than directly measuring AVP concentration or AVP-R. An increase in urine osmolality by 15%–45% with urine
activity. Patients often need to be hospitalized to perform the test. osmolality remaining less than 300 mOsm/kg is suggestive of partial
Medications that could influence urine output or AVP action need AVP-R. An increase in urine osmolality by 15%–100% to greater
to be discontinued (diuretics, sodium glucose transport protein-2 than 300 mOsm/kg is suggestive of partial AVP-D.
Figure 4. The water deprivation test can help diagnose DI, distinguish between a central and nephrogenic defect, and assess the severity of dis-
ease. IV, intravenous; SQ, subcutaneous; UOsm, urinary osmolality. Used with permission from NephSIM (www.nephsim.com). Reprinted
from ref. 7, with permission from Elsevier. Central and nephrogenic diabetes insipidus have been replaced by arginine vasopressin deficiency
and resistance, respectively, to accommodate change in nomenclature.
Nephrology Self-Assessment Program - Vol 23, No 2, June 2024 107
Copeptin-Based Testing Approaches to Arginine More recently, in 2023, an international, noninferiority trial of
Vasopressin–Related Polyuria 158 adult patients with polydipsia and hypotonic polyuria or a
known diagnosis of AVP-D compared hypertonic saline stimulation
Plasma AVP has a short half-life of 5–10 minutes, is largely platelet
on one day to arginine stimulation on another day with subsequent
bound, and has low ex vivo stability, which makes accurate measure-
measurement of copeptin levels (33). The prespecified copeptin cut-
ment difficult (7). Hence, plasma copeptin, which is secreted in
off was 3.8 pmol/L for arginine stimulation and 3.9 pmo/L for the
equimolar amounts with AVP, has twice the half-life of AVP, and
hypertonic saline stimulation group (achieved sodium target of
has greater ex vivo stability, can serve as a surrogate for AVP
$149 mEq/L). Hypertonic saline–stimulated copeptin testing was
(7,27,29). The physiologic functions of copeptin are largely
more accurate than arginine stimulation (95.6% versus 74.4%). The
unknown, but it is believed to be a structural chaperone for the for-
arginine stimulation test was quicker, was preferred by more patients
mation of preprovasopressin (PPV) (29).
in this study, and could be performed in the outpatient setting.
However, the authors could not determine a single reliable cutoff
4 Copeptin-based diagnostic approaches are being copeptin value to differentiate between AVP-D and primary poly-
dipsia (33). The diagnostic specificity was high (.90%) when the
increasingly studied in hypotonic polyuria. They
copeptin level was #3.0 pmol/L (for AVP-D) or .5.2 pmol/L (for
may be more accurate and may avoid the need primary polydipsia), whereas copeptin levels between 3 and 5.2
for water deprivation, which may improve patient pmol/L were less accurate in differentiating between the two.
tolerability.
Treatment of Arginine Vasopressin–Related Polyuria
AVP-D treatment depends on access to water, ability to drink, and
severity of the polyuria. For patients with mild polydipsic AVP-D
Copeptin-based diagnostic approaches are being increasingly
with access to water, nocturnal desmopressin is the drug of choice
studied in hypotonic polyuria. Baseline copeptin levels may be high
(29). Desmopressin can be administered orally, sublingually, or
in AVP-R, with a cutoff of $21.4 pmol/L proposed as a diagnostic
intranasally. Patients with severe AVP-D often require desmopressin
tool with 100% sensitivity and specificity, without need for water
twice a day. The route of therapy and dose needs to be individual-
deprivation (29,30). However, to differentiate between primary
ized for every patient, depending upon severity of the deficiency,
polydipsia and AVP-D, a stimulation test is required—water depri-
access to water, and tolerance for frequent urination. Desmopressin
vation and infusion of hypertonic saline or arginine hydrochloride.
disrupts normal osmoregulation, and patients can develop hypona-
An international multicenter study of 156 participants across
tremia from excess free-water consumption, which must be empha-
11 tertiary medical centers in Switzerland, Germany, and Brazil
sized to patients (29). Patients must also be educated in recognizing
compared water deprivation testing to copeptin measurements after
early symptoms of hyponatremia. To decrease the risk of hyponatre-
hypertonic saline infusion to differentiate between various causes of
mia, some authors have proposed routinely skipping desmopressin
hypotonic polyuria (31). In the hypertonic saline infusion group,
doses at prespecified intervals, a method referred to as “desmopressin
patients received a 250-ml bolus of 3% hypertonic saline, followed
escape” (29). A recent large survey of 1034 patients with AVP-D
by an infusion at 0.15 ml/kg per minute. Plasma osmolality,
sodium, urea, and glucose levels were measured every 30 minutes reported that patients who follow this approach to allow intermit-
tent aquaresis had significantly lower prevalence of hyponatremia
using serum and point-of-care whole blood samples. Once a target
sodium of 150 mEq/L was achieved on the point-of-care whole compared with those who do not skip doses regularly (odds ratio
blood sample, serum copeptin was measured, and patients were sub- [OR] 0.55; [95% CI, 0.39 to 0.77], P50.006) (23). Adipsic patients
sequently given enteral free water (30 ml/kg) and a 500-ml bolus of with AVP-D are difficult to treat, because they experience rapid fluc-
5% dextrose water solution (D5W). Significantly higher diagnostic tuations in serum sodium levels due to lack of thirst sensation and
accuracy was achieved with the hypertonic saline infusion test versus inability to replace urinary free-water losses (34). These patients need
the water deprivation test (96.5% versus 76.6%, P,0.001) (31). to follow strict desmopressin dosing and meticulously monitor water
Derived from a post hoc analysis, a copeptin cutoff level $6.5 pmol/L intake, urine output, and body weight daily (34).
had a diagnostic accuracy of 97.9%, sensitivity of 94.9%, and specifi-
city of 100% in differentiating between primary polydipsia and AVP-
D (31). The study participants were also more likely to report favorably 4 To decrease the risk of hyponatremia, some authors
on the tolerability of the hypertonic saline infusion test over the water have proposed routinely skipping desmopressin
deprivation test. doses, a method referred to “desmopressin escape.”
Arginine hydrochloride infusion can result in stimulation of
both anterior and posterior pituitary, resulting in release of copeptin,
growth hormone, and other pituitary hormones (29). In a study, 96 AVP-R is more challenging to treat. A low-solute diet (includ-
patients with polyuria-polydipsia syndrome received 0.5 g/kg infu- ing salt and protein) is often prescribed to reduce the dietary
sion of L-arginine hydrochloride (21%) in 500 ml 0.9% sodium osmotic load and its associated obligatory urinary volume, along
chloride solution with subsequent serial measurements of copeptin with adequate hydration to replace urinary free-water losses (35). Thia-
(32). A cutoff copeptin level of 3.8 pmol/L measured 60 minutes zide diuretics may paradoxically reduce urine output through inducing
after the infusion had a diagnostic accuracy of 93% to differentiate mild hypovolemia and stimulating sodium and water reabsorption in
between primary polydipsia and AVP-D (32). the proximal convoluted tubule by the renin-angiotensin-aldosterone
108 Nephrology Self-Assessment Program - Vol 23, No 2, June 2024
system (35). Among patients treated with lithium, amiloride may Treatment of Hypernatremia from Sodium Gain
prevent lithium entry through the epithelial sodium channel into In primary aldosteronism, hypernatremia, which may be mediated
principal cells and attenuate lithium-mediated AVP-R (36). Addi- through a reset osmostat, is often corrected by a medical or surgical
tionally, nonsteroidal anti-inflammatory drugs such as indomethacin treatment of the aldosteronism (16). Massive ingestion of hypertonic
or cyclooxygenase-2 inhibitors may enhance urinary concentration salt solutions such as soy sauce can cause life-threatening acute
through increased expression of Na-K-2Cl cotransporter type 2 and hypernatremia with cerebral shrinkage that necessitates rapid reduc-
AQP2 (37,38). tion in plasma sodium concentration through large volumes of
enteral free water and/or D5W intravenous boluses (44,45).
Treatment of Hypernatremia from Water Loss
In most cases, hypernatremia is treated with enteral free water or Hypernatremia in the ICU
parenteral hypotonic solutions. Enteral route involves improving Hypernatremia is commonly encountered in the ICU in up to 26%
access to free water and encouraging the patient to drink to thirst. of patients (46). ICU-related hypernatremia is known to be associ-
Parenteral repletion can be done using 5% dextrose water solution ated with poor outcomes. A recent retrospective study of 162,026
(D5W) or 0.45% sodium chloride. The goal of treatment is to patients admitted to the ICU between January 2010 and June 2022
restore the free-water deficit and replace ongoing electrolyte-free showed that 5.6% of patients had sodium .145 mEq/L at the time
water losses. Traditionally, the rates of plasma sodium correction for of ICU admission, with 27% higher odds of ICU mortality (OR,
patients with chronic hypernatremia (present for a duration of .48 1.27; 95% CI, 1.19 to 1.36) and 52% higher odds of in-hospital
hours) was recommended at ,0.5 mEq/L per hour to decrease the mortality (OR, 1.52; 95% CI, 1.43 to 1.62) (12). Hypernatremic
risk of cerebral edema from osmotic shifts (39). However, a 2017 patients also had a longer ICU length of stay (LOS), hospital LOS,
retrospective study of 122 patients with community-acquired hyper- and increased healthcare costs. In another large observational study
natremia and 327 patients with hospital-acquired hypernatremia in of 207,702 ICU patients, hypernatremia developed in 4.3% of
a single center in New York City revealed similar neurologic out- patients and was independently associated with a 40% increase in
comes between patients who had rapid (.0.5 mEq/L per hour) and hospital mortality and a 28% increase in ICU LOS (47). From ani-
slow (#0.5 mEq/L per hour) correction in hypernatremia (40). mal and retrospective human studies, hypernatremia has been associ-
Importantly, there were no cases of cerebral edema or seizures that ated with negative chronotropic effects, myocardial dysfunction, and
were suspected to be caused by hypernatremia correction. In fact, changes in cerebral volume (46). Because of a lack of randomized
the medical literature does not have a single convincing case report clinical trials, it is unclear whether hypernatremia is directly respon-
of cerebral edema from hypernatremia correction in an adult (41). sible for the elevated mortality or through its association with disease
However, infants may be more susceptible to seizures from rapid severity.
rehydration. More recently, a 2023 retrospective study of 4265 ICU-related hypernatremia has long been attributed to a net
hypernatremic patients admitted to a single center in Tel Aviv, Israel loss of electrolyte-free water, regardless of the underlying fluid bal-
between 2007 and 2021 showed that rapid correction .0.5 mEq/L ance or volume status. Maintenance fluids tend to be isotonic,
per hour was associated with lower 30-day mortality than slower whereas ongoing fluid losses may be hypotonic, and the lack of
correction (,0.5 mEq/L per hour) (31.8% [109 of 343] versus access to water for critically ill patients fails to correct this imbalance
50.7% [1990 of 3922]; P,0.001) (42). Moreover, rapid sodium (46). This is technically correct, since electrolyte-free water repre-
correction was associated with shorter hospital length of stay, and sents a theoretical volume; a patient may have a net electrolyte free-
there were no differences in neurologic complication rates, none of water loss despite overall water and weight gain. However, there is a
which were attributed to sodium correction. difference between the traditional electrolyte-free water approach
Most hospital centers target a sodium reduction rate of #0.5 and the tonicity approach to understanding sodium disorders (48).
mEq/L per hour; however, there is no convincing evidence to follow The electrolyte-free water approach fails to account for the positive
this strategy. If sodium correction rate exceeds 0.5 mEq/L per hour, sodium balance that is frequently present in ICU patients with AKI.
we do not recommend raising the plasma sodium concentration. If Hypernatremia is often associated with high total-body sodium con-
patients with severe hypernatremia (.160 mEq/L) require hemodi- tent with normal or increased total-body water content (49).
alysis, we recommend a more gradual clearance session using the Sodium balance in critically ill patients is poorly understood; how-
maximum allowed sodium bath (150 mEq/L) to minimize rapid ever, multiple mechanisms may be at play: activation of antinatriure-
osmolar changes over a period of 3–4 hours. Concurrent reduction tic hormones such as the renin-angiotensin-aldosterone system from
in glucose levels in patients with severe hyperglycemia (.500 mg/dl) decreased effective circulating volume, diminished renal blood flow
should be slow (50–75 mg/dl per hour), because cerebral edema and glomerular filtration, acute tubular necrosis (ATN) resulting in
attributed to rapid lowering of glucose in adults has been reduced tubular sodium handling, administration of enteral or paren-
described in several case reports (43). In these patients who have teral feeding with electrolytes to patients with chronic malnutrition,
hypernatremia after correction for hyperglycemia, we recommend and replacement of hypotonic fluid losses with isotonic intravenous
conservative sodium reduction goals (,0.5 mEq/L/hour), which fluids. In addition to positive sodium balance, osmoregulatory failure
may be prudent to minimize the risk of cerebral edema from may be present. Impaired arginine vasopressin (AVP) action may be
reduction in plasma concentrations of two ineffective osmoles caused by azotemia, hypoalbuminemia, and/or effect of exogenous
(sodium and glucose) that are poorly transported across the blood- glucocorticoids in the critically ill (49). Administration of loop diure-
brain barrier. tics to treat hypervolemia may modestly blunt the renal medullary
Nephrology Self-Assessment Program - Vol 23, No 2, June 2024 109
hypertonicity, leading to impaired water conservation and thus aquar- supplementation for the following reason: electrolyte-free water
esis. Table 2 summarizes the various mechanisms of hypernatremia in losses during the polyuric phase of ATN may further worsen hyper-
the critically ill. natremia. When free-water administration is planned, enteral free
A recent prospective observational study of 98 patients admit- water may be the preferred initial route. Parenteral administration
ted to the ICU with an anticipated LOS .48 hours reported a high with D5W may be appropriate if enteral free-water supplementation
median sodium intake of 8–11 g during the first 24 hours of ICU is not feasible or effective. A recent retrospective single-center study
admission and a reduced renal sodium excretion likely due to ongo- of 256 hypernatremic ICU patients in Japan estimated that the
ing critical illness and AKI (net positive sodium balance) (50). Inter- change in sodium concentration was –2.25 (95% CI, 2.76 to 1.74)
estingly, no statistically significant difference was observed in fluid mEq/L per L of parenteral D5W versus –0.89 (–-1.57 to –0.21] per
balance between those who developed hypernatremia (defined as L of enteral free water (52). Often overlooked, hypernatremic
.143 mEq/L) and those who did not. Urine osmolality tended to patients receiving total parenteral nutrition may need to have
be lower in patients who developed hypernatremia, despite elevated sodium and/or potassium content eliminated or reduced for a favor-
copeptin levels, but statistical significance was observed only on able tonicity balance.
some days. The authors hypothesized this may have been a result of Lastly, AVP deficiency (AVP-D) is commonly seen in up to
transient partial urinary concentration defects resulting from kidney 49% of brain-dead patients, which may result in hypernatremia
tubulo-interstitial injury. Besides sodium, potassium balance in the (53). If the patient is a potential organ donor, desmopressin may be
ICU is another understudied topic, which could affect plasma administered to such patients to maintain normonatremia and favor-
sodium concentrations. More studies on sodium and potassium bal- able hemodynamics for organ preservation (54).
ance and urinary concentration defects in the ICU are required.
Permissive Hypernatremia
4 Hypernatremia in critically ill patients is not In some cases, hypernatremia may be intentionally targeted by the
explained by a simple water deficit. It is multifac- intensivist, such as in patients with increased intracranial pressure
torial; positive electrolyte balance, acute tubular from traumatic brain injury (TBI). Options for osmotherapy to
necrosis, and nutritional factors probably play decrease intracerebral volume and thereby increase cerebral perfusion
pressure in this setting include mannitol, 3% hypertonic saline infu-
important roles. sion, 8.4% sodium bicarbonate bolus, 20% hypertonic saline infu-
sion, and 23.4% hypertonic saline bolus. There are no specific
Treatment of hypernatremia in the ICU is controversial. Inten- guidelines from the Brain Trauma Foundation for effective osmother-
sivists fear that administration of free water in edematous patients apy and sodium targets in TBI, and institutional practices vary. A
with lung injury may prolong weaning time from the ventilator. recent meta-analysis of three studies showed a 35.9% (95% CI, 15.0
Nephrologists argue that electrolyte-free water supplementation pre- to 56.9) reduction in intracranial pressure in TBI patients receiving
dominantly goes to the intracellular compartment and may theoreti- 3% hypertonic saline (55). However, there are no RCTs demonstrat-
cally carry minimal risk of exacerbating pulmonary edema (46). ing a positive impact on functional outcomes and/or mortality. In a
There are no randomized controlled trials (RCTs) to address this recent RCT of 370 patients, treatment with continuous infusions of
problem. Because of its association with mortality, hypernatremia 20% hypertonic saline did not result in a significantly better differ-
tends to get treated more aggressively. We advocate for an individu- ence in 6-month mortality and neurologic status (56). A significantly
alized approach to each patient, avoiding severe hypernatremia lower intracranial pressure in the first 2 days was observed in the
(.150 mEq/L) and giving consideration to respiratory status, vol- intervention group but increased after day 4, which may have been
ume status, and kidney function. Universal electrolyte-free water due to brain adaptation to prolonged hypernatremia by accumulating
administration in ICU patients with mild hypernatremia (145–150 intracellular osmolytes, a phenomenon that could potentially worsen
mEq/L), edema from capillary leak, and ongoing AKI could poten- brain swelling when the hypernatremia is treated. In a recent
tially worsen fluid overload, another well-recognized factor associ- propensity-matched study of 580 patients admitted to the neurosurgi-
ated with increased mortality (51). Improving hemodynamics and cal ICU, moderate and severe hypernatremia were significantly associ-
kidney function may be a good indication to begin free-water ated with in-hospital mortality (adjusted OR, 4.58 [2.15 to 9.75] and
Decrease effective arterial blood volume, leading to avid sodium Lack of access to water
retention
Acute kidney injury, leading to sodium and potassium retention Insensible water losses
Replacement of hypotonic fluid losses with isotonic fluids Arginine vasopressin resistance
Electrolyte content in tube feeds and parenteral nutrition Use of loop diuretics, leading to decrease in medullary
concentration gradient
110 Nephrology Self-Assessment Program - Vol 23, No 2, June 2024
6.93 [3.46 to 13.90]) and 28-day mortality (adjusted OR, 3.51 [1.54 deficiency (AVP-D), formerly known as central diabetes insipidus
to 7.98] and 10.60 [5.1 to 21.9]), respectively (57). (CDI), is treated with desmopressin, whereas the treatment for
There is some interest in targeting permissive hypernatremia in AVP-R is more complicated. Finally, it is important to recognize
acute respiratory distress syndrome. In a randomized control study that excess plasma sodium concentration is only part of the picture.
of 40 patients with moderate-to-severe acute respiratory distress syn- Because of electroneutrality, hypernatremia must be associated with
drome, 75% of patients who received hypertonic saline infusion to elevated anion concentrations, predominantly chloride. However,
target a plasma sodium level of 145–150 mEq/L were successfully the pathophysiological relevance of hyperchloremia remains poorly
extubated, compared with 35% in the control group (P50.02) (58). understood.
Some animal studies showed that hyperosmolality could suppress
neutrophil activation, mitigate lung injury, and promote alveolar References
epithelial cell repair (59,60). 1. Bie P: Mechanisms of sodium balance: Total body sodium, surrogate
When permissive hypernatremia is targeted, and the patient is variables, and renal sodium excretion. Am J Physiol Regul Integr Comp
on continuous renal replacement therapy (CRRT), concurrent Physiol 315: R945–R962, 2018 PubMed
administration of 3% hypertonic saline is often required. The rate 2. Appel LJ: Salt intake and hypertension. Uptodate. Available at https://
of infusion can be calculated using the following formula (61). www.uptodate.com/contents/salt-intake-and-hypertension?search=salt%
20intake%20and%20hypertension&source=search_result&selectedTitle=1%
Estimated 3% infusion rate 7E150&usage_type=default&display_rank=1. Accessed Feb 18, 2024
3. Rose B, Post T: Clinical Physiology of Acid-Base and Electrolyte Disorders,
½Target plasma sodium concentration New York, McGraw-Hill Education, 2001
2CRRT sodium concentrationÞ ðQD6QRFÞ 4. Danziger J, Zeidel ML: Osmotic homeostasis. Clin J Am Soc Nephrol 10:
5
ð513 2 target sodium concentrationÞ 852–862, 2015 PubMed
5. Delpire E, Gagnon KB: Water homeostasis and cell volume maintenance
where QD is quantity of dialysis fluid, and QRF is quantity of and regulation. Curr Top Membr 81: 3–52, 2018 PubMed
6. Bhave G, Neilson EG: Body fluid dynamics: Back to the future. J Am
replacement fluid.
Soc Nephrol 22: 2166–2181, 2011 PubMed
This formula does not account for factors that could alter the 7. Patel N, Patel D, Farouk SS, Rein JL: Salt and water: A review of hyper-
tonicity balance, such as nutrition, urinary electrolyte-free water clear- natremia. Adv Kidney Dis Health 30: 102–109, 2023 PubMed
ance, gastrointestinal and insensible water losses, and tonicity of intra- 8. Kovesdy CP, Lott EH, Lu JL, Malakauskas SM, Ma JZ, Molnar MZ,
venous fluids and medication diluents (62). Additionally, interactions et al.: Hyponatremia, hypernatremia, and mortality in patients with
of plasma sodium with sodium reservoirs such as bones and cartilage chronic kidney disease with and without congestive heart failure. Circula-
tion 125: 677–684, 2012 PubMed
are not considered (63). Consequently, the desired plasma sodium
9. Sarafidis P, Ferro CJ, Ortiz A: Hypernatremia and subclinical chronic
concentration may not be attained. In these situations, the rate of 3% kidney disease. Eur Heart J 43: 4436–4437, 2022 PubMed
hypertonic saline infusion needs to be adjusted on the basis of the 10. Arzhan S, Roumelioti ME, Litvinovich I, Bologa CG, Myers OB, Unruh
plasma sodium trajectory (i.e., the rate of infusion must be increased ML: Hypernatremia in hospitalized patients: A large population-based
if target sodium concentration is not reached and reduced if the target study. Kidney360 3: 1144–1157, 2022 PubMed
is exceeded). However, this formula still serves as a useful starting 11. Marahrens B, Damsch L, Lehmann R, Matyukhin I, Patschan S,
point while starting concurrent hypertonic saline therapy with Patschan D: Increased serum sodium at acute kidney injury onset pre-
dicts in-hospital death. J Clin Med Res 15: 90–98, 2023 PubMed
CRRT. It is crucial to note that, should CRRT stop functioning for 12. Ng PY, Cheung RYT, Ip A, Chan WM, Sin WC, Yap DYH: A retro-
any reason, the hypertonic saline infusion must also be held; other- spective cohort study on the clinical outcomes of patients admitted to
wise, it would lead to rapid increases in plasma sodium concentration. intensive care units with dysnatremia. Sci Rep 13: 21236, 2023 PubMed
13. Sterns RH: Disorders of plasma sodium–Causes, consequences, and cor-
Conclusion rection. N Engl J Med 372: 55–65, 2015 PubMed
14. Ismail FY, Szollics A, Szolics M, Nagelkerke N, Ljubisavljevic M: Clini-
Hypernatremia may arise because of a net gain in water-free electro- cal semiology and neuroradiologic correlates of acute hypernatremic
lytes or a net loss of electrolyte-free water. Because of the compulsive osmotic challenge in adults: A literature review. AJNR Am J Neuroradiol
nature of the thirst response, hypernatremia is rare, except in those 34: 2225–2232, 2013 PubMed
with impaired access to water. Hypernatremia in hospitalized 15. Svensson S, Olin AC, Hellgren J: Increased net water loss by oral compared
patients is associated with increased mortality and healthcare costs. to nasal expiration in healthy subjects. Rhinology 44: 74–77, 2006 PubMed
Hypernatremia may be a result of decreased water intake, or factors 16. Gregoire JR: Adjustment of the osmostat in primary aldosteronism.
Mayo Clin Proc 69: 1108–1110, 1994 PubMed
related to renal or extrarenal water loss or sodium gain. arginine 17. Wassel E, Umeh CA, Giberson C, Anand SK, Nguyen A, Porter H, et al.:
vasopressin (AVP)-related disorders are associated with osmoregula- Elevated adrenocorticotropic hormone, hypercortisolism, and marked
tion failure and may need to be diagnosed with water deprivation hypernatremia. Cureus 13: e19714, 2021 PubMed
tests. Hypertonic saline- or arginine-stimulated measurement of 18. Bodonyi-Kovacs G, Lecker SH: Electrolyte-free water clearance: A key to
plasma copeptin levels appears to be a promising method of diagnos- the diagnosis of hypernatremia in resolving acute renal failure. Clin Exp
ing arginine vasopressin–related polyuria. In the ICU, hypernatre- Nephrol 12: 74–78, 2008 PubMed
19. Sterns RH, Silver SM, Hix JK: Urea for hyponatremia? Kidney Int 87:
mia develops because of complex mechanisms, including excessive
268–270, 2015 PubMed
sodium retention, and it is unclear what the optimal sodium target 20. Sterns R: Etiology and evaluation of hypernatremia in adults. Uptodate.
is in ICU patients with AKI. In most cases, hypernatremia is treated Available at https://www.uptodate.com/contents/etiology-and-evaluation-
with enteral free-water or parenteral hypotonic solutions. AVP of-hypernatremia-in-adults. Accessed Feb 15, 2024
Nephrology Self-Assessment Program - Vol 23, No 2, June 2024 111
21. Arima H, Bichet DG, Cheetham T, Christ-Crain M, Drummond J, 41. Sterns RH: Evidence for managing hypernatremia: Is it just hyponatre-
Gurnell M, et al.: Changing the name of diabetes insipidus. Pituitary 25: mia in reverse? Clin J Am Soc Nephrol 14: 645–647, 2019 PubMed
777–779, 2022 PubMed 42. Feigin E, Feigin L, Ingbir M, Ben-Bassat OK, Shepshelovich D: Rate of
22. SNOMED International: SNOMED CT February 2024 international correction and all-cause mortality in patients with severe hypernatremia.
edition - SNOMED International release notes. Available at https:// JAMA Netw Open 6: e2335415, 2023 PubMed
confluence.ihtsdotools.org/display/RMT/SNOMED+CT+February+2024+ 43. Namatame K, Igarashi Y, Nakae R, Suzuki G, Shiota K, Miyake N, et al.:
International+Edition+-+SNOMED+International+Release+notes. Accessed Cerebral edema associated with diabetic ketoacidosis: Two case reports.
Feb 18, 2024. Acute Med Surg 10: e860, 2023 PubMed
23. Atila C, Loughrey PB, Garrahy A, Winzeler B, Refardt J, Gildroy P, 44. Carlberg DJ, Borek HA, Syverud SA, Holstege CP: Survival of acute
et al.: Central diabetes insipidus from a patient’s perspective: Manage- hypernatremia due to massive soy sauce ingestion. J Emerg Med 45: 228–
ment, psychological co-morbidities, and renaming of the condition: 231, 2013 PubMed
Results from an international web-based survey. Lancet Diabetes Endocri- 45. Sakamoto A, Hoshino T, Boku K, Hiraya D, Inoue Y: Fatal acute hyper-
nol 10: 700–709, 2022 PubMed natremia resulting from a massive intake of seasoning soy sauce. Acute
24. Cuesta M, Hannon MJ, Thompson CJ: Adipsic diabetes insipidus in Med Surg 7: e555, 2020 PubMed
adult patients. Pituitary 20: 372–380, 2017 PubMed 46. Chand R, Chand R, Goldfarb DS: Hypernatremia in the intensive care
25. Iwasaki Y, Oiso Y, Kondo K, Takagi S, Takatsuki K, Hasegawa H, et al.: unit. Curr Opin Nephrol Hypertens 31: 199–204, 2022 PubMed
Aggravation of subclinical diabetes insipidus during pregnancy. N Engl J 47. Waite MD, Fuhrman SA, Badawi O, Zuckerman IH, Franey CS: Inten-
Med 324: 522–526, 1991 PubMed sive care unit-acquired hypernatremia is an independent predictor of
26. Di Iorgi N, Napoli F, Allegri AEM, Olivieri I, Bertelli E, Gallizia A, increased mortality and length of stay. J Crit Care 28: 405–412, 2013
et al.: Diabetes insipidus–Diagnosis and management. Horm Res Paediatr PubMed
77: 69–84, 2012 PubMed 48. Davids MR, Edoute Y, Mallie JP, Bichet DG, Halperin ML: Body com-
27. Gubbi S, Hannah-Shmouni F, Koch CA, Verbalis JG: Diagnostic testing partment volumes and composition after giving a vasopressin antagonist:
for diabetes insipidus. In: Endotext [Internet]. Available at: http://www. Changes are revealed by a tonicity balance. Nephrol Dial Transplant off
ncbi.nlm.nih.gov/books/NBK537591/. Accessed Jan 24, 2024 Transplant 17: 300–303, 2002 PubMed
28. Miller M, Dalakos T, Moses AM, Fellerman H, Streeten DH: Recogni- 49. Kahn T: Hypernatremia without water depletion. Clin Nephrol 76:
tion of partial defects in antidiuretic hormone secretion. Ann Intern Med 130–135, 2011 PubMed
73: 721–729, 1970 PubMed 50. van IJzendoorn M, de Vries L, van den Born J, Buter H, Navis G,
29. Refardt J, Atila C, Christ-Crain M: New insights on diagnosis and Boerma C: Renal function is a major determinant of ICU-acquired
treatment of AVP deficiency. Rev Endocr Metab Disord, 2023 PubMed hypernatremia: A balance study on sodium handling. J Transl Int Med 8:
30. Timper K, Fenske W, K€ uhn F, Frech N, Arici B, Rutishauser J, et al.: 165–176, 2020 PubMed
Diagnostic accuracy of copeptin in the differential diagnosis of the 51. Messmer AS, Zingg C, M€ uller M, Gerber JL, Schefold JC, Pfortmueller
polyuria-polydipsia syndrome: A prospective multicenter study. J Clin CA: Fluid overload and mortality in adult critical care patients—A sys-
Endocrinol Metab 100: 2268–2274, 2015 PubMed tematic review and meta-analysis of observational studies. Crit Care Med
31. Fenske W, Refardt J, Chifu I, Schnyder I, Winzeler B, Drummond J, 48: 1862–1870, 2020 PubMed
et al.: A copeptin-based approach in the diagnosis of diabetes insipidus. 52. Suzuki R, Uchino S, Sasabuchi Y, Kawarai Lefor A, Shiotsuka J, Sanui
N Engl J Med 379: 428–439, 2018 PubMed M: Enteral free water vs. parenteral dextrose 5% in water for the
32. Winzeler B, Cesana-Nigro N, Refardt J, Vogt DR, Imber C, Morin B, treatment of hypernatremia in the intensive care unit: A retrospective
et al.: Arginine-stimulated copeptin measurements in the differential cohort study from a mixed ICU. J Anesth 37: 868–879, 2023 PubMed
diagnosis of diabetes insipidus: A prospective diagnostic study. Lancet 53. Nair-Collins M, Northrup J, Olcese J: Hypothalamic-pituitary function
Lond Engl 394: 587–595, 2019 PubMed in brain death: A review. J Intensive Care Med 31: 41–50, 2016
33. Refardt J, Atila C, Chifu I, Ferrante E, Erlic Z, Drummond JB, et al.: PubMed
Arginine or hypertonic saline-stimulated copeptin to diagnose AVP defi- 54. Guesde R, Barrou B, Leblanc I, Ourahma S, Goarin JP, Coriat P, et al.:
ciency. N Engl J Med 389: 1877–1887, 2023 PubMed Administration of desmopressin in brain-dead donors and renal func-
34. Kothari V, Cardona Z, Eisenberg Y: Adipsic diabetes insipidus. Handb tion in kidney recipients. Lancet Lond Engl 352: 1178–1181, 1998
Clin Neurol 181: 261–273, 2021 PubMed PubMed
35. Lopes A, Campos A. D C, Marques Sim~oes J, Jord~ao A: Lithium- 55. Gharizadeh N, Ghojazadeh M, Naseri A, Dolati S, Tarighat F, Soleiman-
induced arginine vasopressin resistance (AVP-R): A case of chronic expo- pour H: Hypertonic saline for traumatic brain injury: A systematic
sure to lithium. Cureus 15: e41677, 2023 PubMed review and meta-analysis. Eur J Med Res 27: 254, 2022 PubMed
36. Bedford JJ, Weggery S, Ellis G, McDonald FJ, Joyce PR, Leader JP, 56. Roquilly A, Moyer JD, Huet O, Lasocki S, Cohen B, Dahyot-Fizelier C,
et al.: Lithium-induced nephrogenic diabetes insipidus: Renal effects of et al.; Atlanrea Study Group and the Societe Française d’Anesthesie
amiloride. Clin J Am Soc Nephrol 3: 1324–1331, 2008 PubMed Reanimation (SFAR) Research Network: Effect of continuous infusion of
37. Monnens L, Jonkman A, Thomas C: Response to indomethacin and hypertonic saline vs standard care on 6-month neurological outcomes in
hydrochlorothiazide in nephrogenic diabetes insipidus. Clin Sci (Lond) patients with traumatic brain injury: The COBI randomized clinical trial.
66: 709–715, 1984 PubMed JAMA 325: 2056–2066, 2021 PubMed
38. Kim GH, Choi NW, Jung JY, Song JH, Lee CH, Kang CM, et al.: 57. Lee YI, Ahn J, Ryu JA: Clinical outcomes associated with degree of
Treating lithium-induced nephrogenic diabetes insipidus with a COX-2 hypernatremia in neurocritically ill patients. J Korean Neurosurg Soc 66:
inhibitor improves polyuria via upregulation of AQP2 and NKCC2. Am 95–104, 2023 PubMed
J Physiol Renal Physiol 294: F702–709, 2008 PubMed 58. Bihari S, Prakash S, Dixon DL, Cavallaro E, Bersten AD: Induced hyper-
39. Adrogue HJ, Madias NE: Hypernatremia. N Engl J Med 342: 1493– natremia in patients with moderate-to-severe ARDS: A randomized con-
1499, 2000 PubMed trolled study. Intensive Care Med Exp 9: 33, 2021 PubMed
40. Chauhan K, Pattharanitima P, Patel N, Duffy A, Saha A, Chaudhary 59. Angle N, Hoyt DB, Coimbra R, Liu F, Herdon-Remelius C, Loomis W,
K, et al.: Rate of correction of hypernatremia and health outcomes in et al.: Hypertonic saline resuscitation diminishes lung injury by suppres-
critically ill patients. Clin J Am Soc Nephrol 14: 656–663, 2019 sing neutrophil activation after hemorrhagic shock. Shock Augusta Shock
PubMed 9: 164–170, 1998 PubMed
112 Nephrology Self-Assessment Program - Vol 23, No 2, June 2024
60. Wang S, Singh RD, Godin L, Pagano RE, Hubmayr RD: Endocytic 62. Sterns RH: Formulas for fixing serum sodium: Curb your enthusiasm.
response of type I alveolar epithelial cells to hypertonic stress. Am J Phy- Clin Kidney J 9: 527–529, 2016 PubMed
siol Lung Cell Mol Physiol 300: L560–568, 2011 PubMed 63. Hessels L, Oude Lansink-Hartgring A, Zeillemaker-Hoekstra M, Nijsten
61. Yessayan LT, Szamosfalvi B, Rosner MH: Management of dysnatremias with MW: Estimation of sodium and chloride storage in critically ill patients:
continuous renal replacement therapy. Semin Dial 34: 472–479, 2021 PubMed A balance study. Ann Intensive Care 8: 97, 2018 PubMed
Nephrology Self-Assessment Program - Vol 23, No 2, June 2024 113
Article
Pathophysiology, Evaluation, and Treatment of Hypokalemia
Gregor Lindner, MD, MBA
Department of Emergency Medicine, Kepler Universit€atsklinikum GmbH, Johannes Kepler University, Linz, Austria
Christoph Schwarz, MD
Department of Internal Medicine & Nephrology, Pyhrn-Eisenwurzen Klinikum Steyr, Steyr, Austria
Svenja Ravioli, MD
Department of Emergency Medicine, Kepler Universit€atsklinikum GmbH, Johannes Kepler University, Linz, Austria
intake. The human body has developed different strategies to survive Renal potassium resorption is stimulated by hypokalemia. In
periods of fasting by reducing daily potassium loss; however, these type A intercalated cells, potassium is reabsorbed via H1-K1-
periods must be followed by potassium intake again to fully replen- ATPase type 2 (HKA2) in patients with chronic potassium deple-
ish the potassium storages of the body. tion. This process is stimulated by progesterone, the secretion of
which is enhanced during chronic potassium depletion (21).
Cellular Shifting. Intracellular potassium storages are depleted to Recently, Laasad et al. (22) showed that growth factor GDF15 is
maintain the extracellular potassium concentration in times of low upregulated in the kidney and colon during potassium depletion.
or no potassium intake. A low potassium intake or hypokalemia This mediates a proliferation of the IC-A cells and increases the
decreases the activity of the Na-K-ATPase in muscle cells and there- renal capacity to reabsorb potassium via HKA2. This effect was
fore decreases the cellular potassium reuptake (12,14). This mecha- completely blunted by GDF15 knockout animals.
nism prevents a rapid fall of the serum potassium concentration, but
leads to severe depletion of total body potassium storages over time. Physiology of Potassium Supplementation. Natural sources of
When serum potassium levels decline to 3 or 2 mEq/L, total potas- potassium are fruits and vegetables but also meat. In fruits and vegeta-
sium losses are 200–400 mEq or .800 mEq, respectively (15). bles, potassium is delivered mostly with an organic anion (source of
alkali), whereas, in meat potassium, it is delivered with a source of
Potassium Handling by the Kidney. During low potassium intake, acid (23). Additionally, potassium chloride may become an important
the kidney reduces daily potassium elimination to a minimum of at source of potassium in the future. A randomized trial including
least 10–30 mEq/d (16). Potassium is freely filtered in the glomeru- 20,995 high–cardiovascular risk individuals showed that an NaCl sub-
lus and mainly reabsorbed in the proximal tubule and the loop of stitute containing 75% NaCl and 25% KCl decreased the risk of
Henle (10). Approximately 10% of the filtrated potassium reaches stroke, major cardiovascular events, and death (24). However, the risk
the distal tubular section, which is the main region of potassium of hyperkalemia was not completely assessed in this trial.
secretion or adsorption. In potassium depletion states, the kidney Increasing the oral potassium intake raises the potassium con-
decreases potassium secretion and enhances potassium reabsorption. centration in stool only slightly, pointing out that intestinal potas-
This process is partly regulated by hormones. Renin secretion is sium uptake increases almost directly with increasing intake (25).
stimulated by hypokalemia and enhances the production of angio- However, the kidney has a great potential to excrete potassium after
tensin II, whereas aldosterone secretion is suppressed (17). a potassium-rich meal. A rapid dephosphorylation (inactivation) of
In DCT1 cells, low potassium concentration is “detected” by the NCC in the DCT1 enhances distal delivery of sodium and
Kir4.1/5.1 potassium channels and induces an upregulation of the increases urine flow, thereby providing the basis for sodium-
sodium chloride cotransporter (NCC), which reduces the distal potassium exchange in more distal tubular sections (26). In
delivery of sodium and urine flow (18) and thus impairs potassium DCT2/CNT, hyperkalemia induces an mTORC2 pathway, leading
secretion distally. Recent data in mineralocorticoid receptor (MR) to activation of ENaC and ROMK-dependent potassium excretion
knockout animals showed that, in DCT2, the epithelial sodium (27). This mechanism is independent of aldosterone (but not of
channel (ENaC) is constitutionally active. The authors conclude MR), because this part of the nephron has only low activity of 11b-
that aldosterone-independent stimulation of the MR likely occurs hydroxy steroid dehydrogenase type 2 (28). In addition, potassium
via glucocorticoid stimulation of the MR in the setting of very low is secreted on an aldosterone-dependent manner in principal cells of
levels of 11b-hydroxy steroid dehydrogenase 2 activity (which pre- the cortical collecting duct (29).
vents MR activation by glucocortiocoid), which maintains a basal As mentioned above, intercalated cells are also involved in renal
secretion of potassium via renal outer medullary potassium channel potassium secretion and adsorption, but these processes depend on
(ROMK) (19). In hypokalemic states, the low distal delivery of the source of the accompanying anion. Interestingly, little is known
sodium is accompanied by suppression of ROMK expression in about the effect of different potassium sources on intestinal resorp-
DCT2 and principal cells of the collecting duct. High levels of tion, transmembrane shifting, and renal excretion. Potassium chloride
angiotensin II and hypokalemia per se negatively regulate ROMK and citrate lead to a similar increase of about 0.7 mEq/L in serum
expression (20). potassium concentration after 60 minutes in healthy volunteers (30).
In principal cells of the cortical collecting duct, the ENaC This effect is not observed when potassium chloride is delivered
expression and potassium secretion is under the control of aldoste- together with a carbohydrate-based meal, supporting the importance
rone. Because of low aldosterone levels, ENaC activation is low and of insulin in cellular uptake of potassium (31). Kaliuresis is immedi-
impairs the electrogenic sodium retention and potassium secretion ately seen after a potassium-rich meal, even before the anticipated
via ROMK (20). effect of aldosterone on renal potassium excretion is expected, which
In b-intercalated cells, the chloride-bicarbonate exchanger pen- may be induced by a still unknown kaliuretic gut factor (31). The
drin is upregulated by angiotensin II. Together with the sodium- amount of kaliuresis depends on the accompanying anion. After a
dependent HCO3-Cl exchanger (NDCBE), an isoelectric retention potassium alkali load, not only is an electrogenic potassium secretion
of NaCl is established, further decreasing the availability of an elec- induced in the primary cells of the collecting duct via ENaC,
trogenic sodium–potassium exchange (17). The isoelectric uptake of ROMK, and BK stimulation, but also an electroneutral potassium
NaCl via NCC and pendrin impairs sodium delivery and flow and and bicarbonate secretion is induced in b-intercalated cells. This effect
therefore reduces potassium secretion via flow-dependent large con- is the result of a higher expression of pendrin together with the potas-
ductance and calcium-activated (big) potassium channels (BK). sium chloride transporter KCC3a (Figure 1) (32,33). An attenuated
These mechanisms together are also responsible for the salt-sensitive effect on renal potassium elimination was observed after treatment
hypertension in the case of low potassium intake. with potassium chloride: There is reduced stimulation of ENaC and
Nephrology Self-Assessment Program - Vol 23, No 2, June 2024 115
A Basal potassium
ROMK
intake K+
K+ secretion mediated
Na+
by principal cells
ENaC
K+
BK
B High KCI intake C High KHCO3 intake
PP-IC
Response mediated by K+ - Response mediated by K+ - secret
secreting machinery in PCs machineries in PCs and PP- IC
ROMK ROMK
K+
K+
ENaC ENaC
Na+ K+ K+
Na+
KCC3a KCC3a
K+
K+
Cl- Cl- K+ Cl- Cl-
HCO3-
HCO3- Pendrin
Pendrin K+
K+
BK BK K+
PP-IC PP-IC Stimulation of the
electroneutral machinery
secreting K+ with HCO3-
Figure 1. The kidney adapts to KCl and KHCO3 salts differently. (A) Distribution of Na1 and K1 transporters in principal cells (PCs) and
pendrin-positive intercalated cells (PP-ICs) with basal K1 intake. Note the electrogenic Na1/K1 exchange, mediated by ROMK channels,
large-conductance BK channels, and ENaC in PCs, and the electroneutral K1-secreting machinery driven by pendrin/KCC3a coupling in
PP-ICs. (B) Under high KCl intake, the increase in K1 secretion occurs via the activation of the electrogenic K1-secreting machinery in PCs.
(C) Under high KHCO3 intake, the enhanced kaliuretic response is mediated by robust activation of the electrogenic K1-secreting machinery
in PCs and the electroneutral K1-secreting machinery in PP-ICs. Parallel activation of pendrin and KCC3a has the capacity to drive KHCO3
secretion by recycling Cl2. Dashed black arrows indicate the direction of ion movement through the indicated channels and transporters. Green
triangles indicate the relative magnitude of K1 secretion mediated by different cell types in the distal nephron. The illustration was created with
BioRender.com. Reprinted from ref. 32, with permission.
ROMK due to potassium chloride compared with potassium alkali in It is noteworthy, that all these studies were mostly performed in
the cortical collecting duct, but not in the DCT2. Further, the acidi- animals or healthy probands with normal serum potassium levels.
fying effect of potassium chloride reduces serum bicarbonate, sup- Thus, it remains speculative that potassium substitution with chlo-
presses the pendrin expression, does not activate the KCC3a, and, ride raises total body potassium stores more effectively than alkaliz-
therefore, reduces the renal potassium excretion (32). ing potassium sources.
116 Nephrology Self-Assessment Program - Vol 23, No 2, June 2024
Mechanism Details
Decreased potassium intake Prolonged period of very low potassium intake (mostly only a contributing factor)
Cellular shift Insulin: be aware of absolute potassium depletion despite normal/high potassium values in diabetic
ketoacidosis
b-adrenergic stimulation: via exogenous b-adrenergic medication or hyperadrenergic state (e.g., alcohol
withdrawal)
Alkalemia (,0.4 mEq/L decrease in [K1] for each 0.1 unit increase in pH)
Hypokalemic periodic paralysis (characterized by proximal accentuated weakness or paralysis developing
over minutes or hours that last minutes to days)
Extrarenal potassium loss Potassium losses via stool during prolonged episodes of diarrhea (often in combination with reduced
potassium intake)
Potassium losses via sweat under circumstances of extreme sweating (e.g., exercise in hot and humid
weather)
Renal potassium loss Diuretics, especially loop and thiazide(-like): hypokalemia and metabolic alkalosis if urine sodium and
chloride concentrations are increased
Increased mineralocorticoid activity: combination of hypokalemia, arterial hypertension, and metabolic
alkalosis
Nonreabsorbable anions: bicarbonate (e.g., in vomiting), ketoanions; penicillin derivatives induce
hypokalemia by an increased delivery of sodium to the collecting duct
Polyuria: large volumes of urine despite low potassium concentration can induce hypokalemia
Bartter/Gitelman syndrome: urinary sodium and potassium wasting paired with metabolic alkalosis and
preserved chloride excretion
Hypomagnesemia: often associated, but potentially induces hypokalemia itself by increased open potassium
channels
Nephrology Self-Assessment Program - Vol 23, No 2, June 2024 117
for each 0.1 unit increase in pH), severe hyperventilation can lead to inhibited by angiotensin II in the collecting tubule (48). This coun-
relevant hypokalemia (10,40). terbalances the stimulatory effect of aldosterone on potassium
Hypokalemic periodic paralysis is another cause of hypokalemia secretion.
triggered by a shift of potassium to the intracellular space. Affected Urinary potassium excretion consequently falls to an equilib-
patients report episodes of proximal accentuated weakness or paraly- rium with potassium intake as a response to hypokalemia. This
sis that develop over minutes or hours and last for minutes up to compensatory mechanism keeps potassium at a low but stable level
days (41). Attacks are triggered by rest after strenuous exercise or and is mediated by a reduction in the activity of the potassium secre-
carbohydrate-rich evening meals. Exposure to cold, stress or agita- tory channels (e.g., ROMK) triggered by an increase in angiotensin
tion, salt intake, glucocorticoids, alcohol, and anesthetic procedures II activity (48–50).
are additional provoking factors (41). The pathophysiologic mecha- The causes of urinary potassium loss and consequent hypokale-
nism of hypokalemic periodic paralysis is a rapid influx of potassium mia are as follows.
into cells (10). The disease can be distinguished between an acquired
form, usually observed in patients with hyperthyroidism (42), and a Diuretics. Diuretics are a common cause of electrolyte disorders
familial form. and hypokalemia. Any diuretic with a site action before the potas-
sium secretory site (i.e., loop and thiazide[-like] diuretics, carbonic-
anhydrase blockers) will increase distal delivery of sodium and water
Extrarenal Potassium Loss
and induce volume depletion. As a consequence, urine potassium
Potassium may be lost in significant amounts via gastric or intestinal excretion will increase, and hypokalemia can develop if potassium
secretions, ultimately resulting in hypokalemia. excretion is greater than intake. An increase in distal sodium delivery
Although potassium concentrations in the stool are low and is coupled to an increase in mineralocorticoid levels and not only
depend on the etiology of diarrhea (15–60 mmol/L), intestinal loss increases potassium secretion but also H1 secretion, resulting in the
could be great in the case of large stool volumes (3–10 L/d). Hypo- generation of metabolic alkalosis (10). Therefore, diuretics should
kalemia occurs frequently in patients with prolonged episodes of be considered as a cause in patients with hypokalemia and metabolic
infectious diarrhea or in those with prolonged diarrhea associated alkalosis if sodium and chloride concentrations are increased in the
with diseases such as villous adenoma or vasoactive intestinal urine. A recent study found that thiazide(-like) diuretics are a strong,
peptide-secreting tumor (25,43,44). Potassium loss can be aggra- independent predictor for the presence of hypokalemia on admission
vated by the development of hyperchloremic acidosis impeding max- to the hospital (51). Furthermore, the effect was type and dose
imal reduction of renal potassium excretion due to an inhibitory dependent, with chlorthalidone bearing the highest risk for hypoka-
effect of acidemia on proximal sodium reabsorption with consequent lemia (odds ratio [OR], 22.7; 95% CI, 9.26 to 55.79; P,0.0001)
potassium excretion due to distal sodium delivery (10). Alkalosis followed by indapamide (OR, 4.7; 95% CI, 2.99 to 7.54; P,0.0001),
and urinary potassium loss are observed if chloride is predominantly metolazone (OR, 2.6; 95% CI, 1.63 to 4.15; P,0.0001), and hydro-
lost via stool (i.e., chloride diarrhea) (45). chlorothiazide (OR, 2.23; 95% CI, 1.89 to 2.62; P,0.0001) (51).
Potassium concentration in sweat is low at 5–10 mEq/L. Still,
hypokalemia can be the result of excess perspiration, for example, in Primary Increased Mineralocorticoid Activity. Primary increased
strenuous exercise or warm and humid climate (46). Relevant potas- mineralocorticoid activity can be caused by an increase in aldoste-
sium loss might also occur via perspiration in patients with cystic rone or nonaldosterone mineralocorticoid secretion and a persistent
fibrosis (47). mineralocorticoid-like effect, respectively (10). Patients usually pre-
sent with hypokalemia, hypertension, and metabolic alkalosis (10).
Increased Urinary Loss American College of Cardiology/American Heart Association guide-
Urinary potassium excretion occurs mostly via secretion in the distal lines suggest screening for primary hyperaldosteronism in patients
nephron (48). Principal cells in the connecting and cortical collect- with hypertension and hypokalemia (52). In a large study from Can-
ing tubule are mainly responsible for secretion of potassium (48). ada, screening rates for hyperaldosteronism in patients with concomi-
tant arterial hypertension and hypokalemia were very low, even if
Hypokalemia due to increased urine potassium loss mainly occurs
more than three antihypertensive agents were prescribed (53). Other
via two mechanisms:
studies confirm the delay in diagnosis of hypertension due to hyperal-
Increased sodium (and water) load is delivered to the collect- dosteronism even with hypokalemia being present (54). These find-
ing tubule. ings are especially important because hyperaldosteronism appears to
Aldosterone stimulates reabsorption of sodium via the epithe- be a common cause of hypertension and is often underrecognized,
lial sodium channel (ENaC). Consequently, a relative electro- and data show that risk for cardiovascular events is increased com-
negativity is created in the lumen, which, in turn, fosters pared with patients with “true” essential hypertension (55). Causes of
passive sodium secretion from tubular cells into the lumen
pseudohyperaldosteronism include a gain of function of the mineralo-
through potassium channels (e.g., renal outer medullary potas-
sium channel [ROMK]).
corticoid receptor (MR) (Geller syndrome), Liddle syndrome (an
autosomal dominant mutation of the ENaC encoding gene resulting
Increased mineralocorticoid activity and distal flow are present in activation and excess sodium reabsorption), cortisol excess (e.g., as
concurrently in most cases of hypokalemia and renal potassium loss. caused by 11b-HSD2 deficiency or cortisol overproduction), and
Angiotensin II stimulates sodium reabsorption in the proximal and deoxycorticosterone excess as caused by both 11b-hydroxylase defi-
distal tubules, which indirectly reduces distal sodium and water ciency and 17a-hydroxylase deficiency, two rare autosomal recessive
delivery in these cases. Potassium secretory channels are also forms of congenital adrenal hyperplasia (56).
118 Nephrology Self-Assessment Program - Vol 23, No 2, June 2024
Nonreabsorbable Anions. Nonreabsorbable anions can induce Salt-Wasting Tubulopathies. Salt-wasting tubulopathies induce
hypokalemia by increasing distal sodium delivery. Common causes hypokalemia by increasing the distal delivery of sodium to the collect-
include bicarbonate in vomiting, ketoanions, or drugs such as penicil- ing duct, also inducing hyperaldosteronism via volume depletion. Bart-
lin derivatives (57). Loss of gastric secretions induces hypokalemia by ter and Gitelman syndromes can be considered genetic equivalents to
increasing delivery of sodium bicarbonate to the collecting duct and is treatment with loop or thiazide diuretics, respectively (11). The syn-
usually apparent via the history and clinical evaluation of the patient. dromes are characterized by urinary sodium and potassium wasting
Evaluation of urine electrolytes together with urine pH can help to paired with metabolic alkalosis and preserved chloride excretion. Gitel-
determine the causative anion (10): although a high urine sodium man syndrome is far more common than Bartter syndrome (approxi-
and low chloride concentration paired with an alkaline pH indicate mately 100-fold) and usually diagnosed in childhood (11). A recent
bicarbonaturia, an acid urine pH implies ketoanions or drugs. in-depth review summarizes the clinical presentation, genetics, and
mechanisms of Gitelman syndrome in detail (58). Figure 2 depicts the
Vomiting. Although gastric secretions contain little potassium, pathophysiologic mechanisms of Bartter and Gitelman syndromes.
hypokalemia may develop in concomitant urinary potassium loss: loss
of gastric acid (e.g., via vomiting) induces metabolic alkalosis/alkalemia. Hypomagnesemia. Hypomagnesemia is a common concurrent find-
The increased bicarbonate concentration leads to a higher filtered ing in hypokalemia. It is reported that hypomagnesemia can be found
bicarbonate load and consequent transport of bicarbonate and water in up to 40% of patients with hypokalemia (59). Mostly, the depletions
to the distal potassium secretory site. In combination with have a similar mechanism, namely, concomitant loss, and hypomagne-
hypovolemia-mediated aldosterone action, increased urinary potas- semia per se appears to induce potassium loss. The mechanism leading
sium loss is induced. to hypomagnesemia-induced hypokalemia is largely unknown but may
involve an increase in open potassium channels (i.e., ROMK) (60,61).
Polyuria. Polyuria alone can lead to hypokalemia by the mere Very Low-Calorie Diets. Very low-calorie diets are associated with
quantity of potassium excreted, even if the potassium concentration the development of hypokalemia, especially in persons not taking
in urine is as low as 5–10 mEq/L. potassium supplements. The mechanisms leading to hypokalemia
are not fully clarified, but renal potassium wasting appears to play
Renal Tubular Acidosis. Hypokalemia can occur in both renal an important role.
tubular acidosis types 1 and 2. Distal renal tubular acidosis (type 1)
is caused by a defect in H1 secretion in the distal nephron, resulting Lupus Nephritis. In patients with lupus nephritis, hypokalemia is a
in disturbed ammonium elimination and persistent alkaline urine common finding and usually attributed to renal potassium wasting
(10). Acidosis interferes with proximal sodium handling, leading to due to diuretics and/or corticosteroid exposure and renal tubular aci-
increased distal sodium delivery and thus hypokalemia (10). Proxi- dosis. However, a recent study including 403 patients with a diagnosis
mal renal tubular acidosis (type 2) is a consequence of impaired of lupus nephritis showed that 5% had hypokalemia not explained
bicarbonate reclamation in the proximal tubule. Increased distal otherwise (62). The authors also reported that these forms of hypoka-
delivery of sodium bicarbonate induces potassium loss and thus lemia in lupus nephritis were associated with a distinct pattern of
hypokalemia (10). autoantibodies (62).
2 K+ 2 K+
BS1
Na+
BSC1 3 Na+ 3 Na+
K+ GS
Na+
Cl-
BS3 NCC
BS5 MAGE2 BS3
Cl-
clcnkb Cl-
BS4 MAGE2 clcnkb Cl-
K+ ROMK Cl-
clcnka
BS2
Na+
Figure 2. Pathophysiology of Bartter and Gitelman syndromes. These syndromes involve inactivation mutations in transporters involved in
transcellular reabsorption of sodium and chloride or their regulatory proteins (see text). Bartter syndrome (A) affects the thick ascending limb,
and Gitelman syndrome (B) affects the distal convoluted tubule. Bartter type 3 displays a broad range of phenotypes, from antenatal Bartter to
Gitelman syndrome. Reprinted from ref. 11, with permission.
Nephrology Self-Assessment Program - Vol 23, No 2, June 2024 119
Causes of hypokalemia are manifold, as outlined above. How- is considered to be induced by hypokalemia-triggered vasoconstriction,
ever, it is important to distinguish renal and extrarenal and a shift a reduced medullary blood flow, and intrarenal ischemia, itself associ-
between compartments as the major mechanisms of hypokalemia. ated with intrarenal angiotensin II and endothelin-1 generation (64).
Of note, hypokalemia is caused by a combination of two or more Blockade of both angiotensin II or endothelin-1 resulted in improve-
mechanisms in many patients. ment of the disease (65,66).
Cardiovascular symptoms of hypokalemia are explained by the
direct impact of low potassium levels on the cardiac conduction sys-
tem. Hypokalemia leads to resting membrane hyperpolarization, Na1/
Impact of Hypokalemia on Patient Health
K1-ATPase inhibition, and suppression of potassium channel conduc-
and Outcome tance (67). These mechanisms lead to a prolongation of the action
The clinical consequences of hypokalemia are linked with the physio- potential duration, a reduced repolarization reserve, and early and
logic functions of potassium, the maintenance of the transmembrane delayed afterdepolarization (67). Consequently, the cardiac cell is
gradient. Thus, hypokalemia mainly manifests in neuromuscular, prone to arrhythmias during states of hypokalemia. Hypokalemia-
renal, and cardiovascular symptoms. associated electrocardiographic (ECG) changes generally receive less
In most cases of mild hypokalemia (i.e., serum potassium attention than the more prominent impact of hyperkalemia on the car-
$3.0 mEq/L), symptoms are mild or absent (63). Neuromuscular diac conduction system. In patients with hypokalemia typical ECG
symptoms may include leg cramps, muscle weakness and even paresis, features include widespread ST segment depression, T-wave inversion,
respiratory failure, and constipation or even intestinal paralysis (63). and U waves (68). Different types of arrhythmias such as premature
Hypokalemia is often associated with primary diseases of the kid- ventricular contractions, ventricular fibrillation, atrial fibrillation, and
ney and is common in acute renal failure (8). On the other hand, torsade de pointes may occur in hypokalemia (68). ECG changes vary
hypokalemia may itself induce kidney disease: hypokalemic kidney dis- according to the current serum potassium level (Figure 3) (69). There-
ease is characterized by tubular interstitial nephritis and nephrogenic fore, it is important to consider hypokalemia as a potential cause in
diabetes insipidus (64). The pathogenesis of hypokalemic nephropathy newly developed arrhythmias.
Figure 3. Changing appearance of hypokalemia-associated ECG changes according to serum potassium levels. Shown are successive ECGs of a
single patient. Day 1: K1 2.0 mEq/L, heart rate 69/min, (PR) interval 0.18 s, QRS duration 0.12 s; QTc interval 0.55 s. There is slight fragmen-
tation of the QRS complex, most prominent in aVF. Day 3: K1 3.0 mEq/L, heart rate 80/min, PR interval 0.17 s, QRS duration 0.12 s, QTc
interval 0.48 s. QRS fragmentation is less prominent. Day 10, control ECG taken 10 days after normalization of K1: heart rate 83/min, PR
interval 0.14 s, QRS duration 0.095 s, QTc interval 0.42 s. The black arrows point toward the T waves. Note the initial negative deflection fol-
lowed by the positive component of the T waves. Reprinted from ref. 68, with permission.
120 Nephrology Self-Assessment Program - Vol 23, No 2, June 2024
A recent multicenter cohort study investigated the prevalence of drug and dietary information and physical examination with assess-
ECG changes in hypokalemic patients (70). Overall, 40% of patients ment of volume status are the first approach to all hypokalemic
with hypokalemia ,3.5 mEq/L had ECG changes of which T-wave patients. In many cases with medication-induced or gastrointestinal-
flattening, ST segment depression, and QTc prolongation were the related hypokalemia, the cause will be apparent from the patient’s
most common, occurring in 27%, 16%, and 14%, respectively history. Routine blood testing including renal function and blood
In a prospective observational study of patients suffering from gas analysis to determine the patient’s acid-base status is recom-
heart failure with preserved ejection fraction, the presence of hypokale- mended. Alternative approaches to examine blood potassium con-
mia ,3.5 mEq/L was independently associated with a higher rate of centration, that is, via analysis of potassium concentration in saliva,
heart failure rehospitalization or heart failure death (71). These results are still under investigation (76).
support previous findings from a randomized, controlled trial in Primary aldosteronism should be considered in hypokalemic
patients with heart failure with preserved ejection fraction treated with hypertensive patients with normal renal function, and an analysis of
spironolactone (72). Data from large meta-analyses consistently report plasma aldosterone-renin ratio should be performed (77). It is impor-
adverse outcomes of patients with low serum potassium levels (9). tant to discontinue mineralocorticoid receptor (MR) antagonists
It is known that patients with end-stage renal disease receiving peri- before testing. In patients with a high pretest probability and ambigu-
toneal dialysis have a higher incidence of hypokalemia compared with ous aldosterone-renin ratio, testing should be repeated after discontin-
patients receiving hemodialysis (73). Moreover, hypokalemia was associ- uation of all renin-angiotensin-aldosterone (ACE) inhibitors and
ated with increased mortality in a large collective of .10,000 peritoneal diuretics (78). As an alternative, a liquid chromatography/tandem
dialysis patients (hazard ratio [HR], 1.51; 95% CI, 1.29 to 1.76) (73). mass spectroscopy (triple A analysis) of aldosterone, angiotensin I,
Peritoneal dialysis patients with hypokalemia defined as a time-averaged and Angiotensin II could be performed to diagnose primary aldoste-
serum potassium ,3.5 mEq/L had an increase in risk for infection- ronism, even with ongoing treatment with ACE inhibitors (79).
related mortality (adjusted HR, 1.85; 95% CI, 1.32 to 2.55) (73). In normotensive patients, spot urine analysis can assist in further
In a multicenter, prospective, randomized, controlled trial in differentiating the causes of hypokalemia (80,81). Measurements of
adult peritoneal dialysis patients with hypokalemia defined as at least urine sodium and chloride levels help distinguish tubulopathy and sur-
three values of serum potassium or an average value ,3.5 mEq/L in reptitious causes (10). Heritable tubulopathies require further genetic
the past 6 months, patients were randomized to either protocol-based testing (82).
potassium supplementation (titratable dose of oral potassium chloride Measurements of urinary potassium excretion help to distinguish
to maintain serum potassium of 4–5 mEq/L) or conventional potas- whether the kidney is responsible for or responding appropriately to a
sium supplementation (reactive supplementation when serum potas- nonrenal cause of hypokalemia. Twenty-four-hour urine collection is
sium is ,3.5 mEq/L) over 52 weeks (74). The primary outcome was the most accurate way to measure renal potassium handling, with a
defined as time from randomization to the first episode of peritonitis, potassium concentration of ,25–30 mEq/d considered a normal
whereas secondary outcomes were all-cause mortality, cardiovascular response to hypokalemia (10). Spot urine potassium concentrations
mortality, hospitalization, and conversion to hemodialysis (74). A allow a more immediate assessment, with values of 5–15 mEq/L indi-
total of 167 patients were randomized, and serum potassium increased cating an extrarenal and values .40 mEq/L indicating a renal cause of
to 4.36 6 0.70 mEq/L in the protocol-based treatment group com- hypokalemia (10). However, spot urine potassium measurements are
pared with 3.57 6 0.65 mEq/L in the group treated conventionally limited by variation in urinary concentration related to multiple factors
(mean difference, 0.66; [95% CI, 0.53 to 0.79] mEq/L; P,0.001) such as water intake, volume status, and mineralocorticoid activity and
(74). In the protocol-based group, the median time to first peritonitis therefore need to be used with caution (10,83,84). Fractional excretion
episode was longer (223 [interquartile range, 147–247] versus 133 of potassium showed a high correlation with 24-hour urine potassium
[interquartile range, 41–197] days, P50.03), and patients had a sig- excretion during replacement therapy and was a convenient and reli-
nificantly lower risk of peritonitis (HR, 0.47; [95% CI, 0.24 to able parameter in assessing renal potassium loss in hypokalemia (85).
0.93]) (74). There were no differences in secondary outcomes (74). Alternatively, measurements of the urine potassium-creatinine ratio
Because previous data showed varying mortality risks for different were suggested for the near-constant rate of urinary creatinine secretion
patient populations with hypokalemia, a research group from the Mayo (10). In order to avoid the limitations of spot urine potassium analysis,
Clinic aimed at applying consensus cluster analysis—an unsupervised the transtubular potassium gradient (urine K 3 plasma osmolality)/
machine learning approach—in order to form clusters in patients with (plasma K 3 urine osmolality) was suggested to estimate the renal
hypokalemia and assess mortality risk among these clusters (75). The potassium excretion by the cortical collecting ducts (86,87). The trans-
study showed that machine learning–based cluster formation appears to tubular potassium gradient requires the urine to be minimum equal to
be feasible in a big, heterogeneous population of hospital patients with plasma osmolality and a urine sodium concentration of at least 25
hypokalemia (75). This might bear implications for future (clinical) mEq/L and, overall, has too many limitations to be broadly recom-
research on hypokalemia applying machine learning approaches. mended for routine use (10,17). Hypokalemia caused by a renal disor-
der is usually less responsive to potassium replacement, because of
ongoing urinary potassium loss.
Diagnosis and Management of Hypokalemia
Diagnostic Approach to Hypokalemic Patients Management of Hypokalemic Patients
Once a diagnosis of hypokalemia is established, the cause should be Besides preventing life-threatening consequences and identifying the
identified. Figure 4 depicts a suggested pragmatic approach to the underlying cause, potassium replacement is the mainstay of hypoka-
differential diagnosis of hypokalemia (1). Clinical history including lemia treatment.
Nephrology Self-Assessment Program - Vol 23, No 2, June 2024 121
Figure 4. Pragmatic diagnostic algorithm for hypokalemia. *If hypokalemia seems disproportionately severe to the dose of diuretic, one may still
consider aldosterone excess. **Ideally, after correcting serum potassium levels, with the patient not taking MR antagonists. Other medications,
including ACE inhibitors and angiotensin receptor blockers, may be continued in most cases; interpretation is dependent upon local laboratory
methods and reporting. Typical presentations are as follows. Vomiting: urine Na/Cl . 1.6, low urine Cl; laxative: urine Na/Cl , 0.7, high
urine Cl; active diuretic use: similar to tubulopathy; tubulopathy: urine K/creatinine (Cr) .2.5 mEq/mEq, with urine Na/Cl 1. Tubulopathy
may be confirmed via genetic testing. Cr, creatinine; HPP, hypokalemic periodic paralysis; RTA, renal tubular acidosis. Reprinted from ref. 1,
with permission.
A 12-lead ECG should routinely be performed in all hypokale- thresholds (9,91–93). It is essential to identify patients with hypoka-
mic patients to assess for cardiac consequences of hypokalemia. Con- lemic periodic paralysis from other causes because of the risk of
tinuous ECG monitoring is indicated if hypokalemia-associated relapse and posttreatment hyperkalemia (94).
ECG changes such as prolonged QT are present or if the patient has Potassium can be administered via oral or intravenous route, of
underlying conditions predisposing to arrythmia, that is, recent which oral is considered safer and equally if not more effective. Potas-
myocardial infarction, long QT syndrome, hypomagnesemia, or sium chloride is the preferred choice, because hypokalemic patients
digoxin toxicity (88,89). are often chloride depleted because of vomiting or diuretics, which
Potassium replacement is mainly guided by the risk of adverse further enhances metabolic alkalosis. In acidosis, however, potassium
events imposed by hypokalemia and the extent of potassium deficit. bicarbonate or potassium citrate are preferred (95). A randomized
Continued replacement may be required, because total potassium placebo-controlled trial found similar plasma potassium increase after
deficit may be significant in severe hypokalemia: each 0.3 mEq/L one oral load of 40 mEq potassium citrate compared with 40 mEq
lower serum potassium corresponds to approximately 100 mEq total potassium chloride (30). However, erythrocyte potassium uptake and
body potassium (90). Because absorption into the intracellular com- kaliuresis were higher after potassium citrate. Potassium phosphate
partments can be slow, potassium levels need close monitoring in should only be considered in cases with concomitant hypophosphate-
case of transient hyperkalemia. Multiple studies suggest potassium mia, rarely seen associated with Fanconi syndrome and phosphate
levels from 4 to 5 mEq/L to be ideal, without defining treatment wasting or ketoacidosis (96). Oral potassium supplementation can be
122 Nephrology Self-Assessment Program - Vol 23, No 2, June 2024
dosed at 100–150 mEq/d. A side effect of liquid forms is a bitter 9. Kovesdy CP, Matsushita K, Sang Y, Brunskill NJ, Carrero JJ, Chodick
taste, and tablets may cause irritation of the gastric mucosa (97). G, et al.; CKD Prognosis Consortium: Serum potassium and adverse
outcomes across the range of kidney function: A CKD Prognosis Consor-
Intravenous potassium chloride can be administered at a recom-
tium meta-analysis. Eur Heart J 39: 1535–1542, 2018 PubMed
mended rate of 10–20 mEq/h. In life-threatening cases, rates of up 10. Palmer BF, Clegg DJ: Physiology and pathophysiology of potassium
to 40 mEq/h were infused (98). Intravenous administration intro- homeostasis: Core curriculum 2019. Am J Kidney Dis 74: 682–695,
duces the risk of phlebitis, usually occurring at rates above 10 2019 PubMed
mEq/h. A saline rather than dextrose solution is recommended to 11. Blanchard A: Pathophysiologic approach in genetic hypokalemia: An
minimize the insulin release and intracellular potassium shift. update. Ann Endocrinol (Paris) 84: 298–307, 2023 PubMed
Concomitant hypomagnesemia needs to be corrected simulta- 12. McDonough AA, Fenton RA: Potassium homeostasis: Sensors, media-
neously when treating hypokalemia (59). Regarding coadministra- tors, and targets. Pflugers Arch 474: 853–867, 2022 PubMed
13. Stadt MM, Leete J, Devinyak S, Layton AT: A mathematical model of
tion of magnesium, regardless of magnesium levels, a retrospective potassium homeostasis: Effect of feedforward and feedback controls.
study found no increase in time to serum potassium normalization PLOS Comput Biol 18: e1010607, 2022 PubMed
but more cases of hypermagnesemia (99). 14. McFarlin BE, Chen Y, Priver TS, Ralph DL, Mercado A, Gamba G,
In the long-term treatment of chronic hypokalemia, initiation of et al.: Coordinate adaptations of skeletal muscle and kidney to maintain
renin-angiotensin-aldosterone inhibitors or potassium-sparing diure- extracellular [K1] during K1-deficient diet. Am J Physiol Cell Physiol
tics might be considered an alternative to costly replacement (1). 319: C757–C770, 2020 PubMed
15. Sterns RH, Cox M, Feig PU, Singer I: Internal potassium balance and
the control of the plasma potassium concentration. Medicine (Baltimore)
60: 339–354, 1981 PubMed
4 Etiologies of hypokalemia can be divided into dif- 16. Squires RD, Huth EJ: Experimental potassium depletion in normal
human subjects. I. Relation of ionic intakes to the renal conservation of
ferent categories: (1) transcellular shift of potassium potassium. J Clin Invest 38: 1134–1148, 1959 PubMed
as seen in b-adrenergic stimulation, alkalemia, or 17. Kamel KS, Schreiber M, Halperin ML: Renal potassium physiology:
Integration of the renal response to dietary potassium depletion. Kidney
hypokalemic periodic paralysis; (2) potassium wast- Int 93: 41–53, 2018 PubMed
ing by the kidney as mostly caused by diuretics and 18. Kristensen M, Fenton RA, Poulsen SB: Dissecting the effects of aldoste-
rone and hypokalemia on the epithelial Na1 channel and the NaCl
increased aldosterone activity; and (3) extrarenal cotransporter. Front Physiol 13: 800055, 2022 PubMed
losses via stool or sweat (less common). 19. Nesterov V, Bertog M, Canonica J, Hummler E, Coleman R, Welling
PA, et al.: Critical role of the mineralocorticoid receptor in aldosterone-
dependent and aldosterone-independent regulation of ENaC in the distal
nephron. Am J Physiol Renal Physiol 321: F257–F268, 2021 PubMed
References 20. Nesterov V, Bertog M, Korbmacher C: High baseline ROMK activity in
1. Clase CM, Carrero JJ, Ellison DH, Grams ME, Hemmelgarn BR, the mouse late distal convoluted and early connecting tubule probably
Jardine MJ, et al.; Conference Participants: Potassium homeostasis and contributes to aldosterone-independent K1 secretion. Am J Physiol Renal
management of dyskalemia in kidney diseases: Conclusions from a Kid- Physiol 322: F42–F54, 2022 PubMed
ney Disease: Improving Global Outcomes (KDIGO) Controversies con- 21. Elabida B, Edwards A, Salhi A, Azroyan A, Fodstad H, Meneton P, et al.:
ference. Kidney Int 97: 42–61, 2020 PubMed Chronic potassium depletion increases adrenal progesterone production
2. Cooper LB, Savarese G, Carrero JJ, Szabo B, Jernberg T, Jonsson Å, that is necessary for efficient renal retention of potassium. Kidney Int 80:
et al.: Clinical and research implications of serum versus plasma potas- 256–262, 2011 PubMed
sium measurements. Eur J Heart Fail 21: 536–537, 2019 PubMed 22. Lasaad S, Walter C, Rafael C, Morla L, Doucet A, Picard N, et al.:
3. Overwyk KJ, Pfeiffer CM, Storandt RJ, Zhao L, Zhang Z, Campbell GDF15 mediates renal cell plasticity in response to potassium depletion
NRC, et al.: Serum sodium and potassium distribution and characteris- in mice. Acta Physiol (Oxf) 239: e14046, 2023 PubMed
tics in the US population, National Health and Nutrition Examination 23. Demigne C, Sabboh H, Puel C, Remesy C, Coxam V: Organic
Survey 2009–2016. J Appl Lab Med 6: 63–78, 2021 PubMed anions and potassium salts in nutrition and metabolism. Nutr Res Rev
4. Adamczak M, Chudek J, Zejda J, Bartma nska M, Grodzicki T, 17: 249–258, 2004 PubMed
Zdrojewski T, et al.: Prevalence of hypokalemia in older persons: Results 24. Neal B, Wu Y, Feng X, Zhang R, Zhang Y, Shi J, et al.: Effect of salt
from the PolSenior national survey. Eur Geriatr Med 12: 981–987, 2021 substitution on cardiovascular events and death. N Engl J Med 385:
PubMed 1067–1077, 2021 PubMed
5. Lindner G, Pfortm€ uller CA, Leichtle AB, Fiedler GM, Exadaktylos AK: 25. Agarwal R, Afzalpurkar R, Fordtran JS: Pathophysiology of potassium
Age-related variety in electrolyte levels and prevalence of dysnatremias absorption and secretion by the human intestine. Gastroenterology 107:
and dyskalemias in patients presenting to the emergency department. 548–571, 1994 PubMed
Gerontology 60: 420–423, 2014 PubMed 26. Grimm PR, Tatomir A, Rosenbaek LL, Kim BY, Li D, Delpire EJ, et al.:
6. Crop MJ, Hoorn EJ, Lindemans J, Zietse R: Hypokalaemia and subse- Dietary potassium stimulates Ppp1Ca-Ppp1r1a dephosphorylation of kid-
quent hyperkalaemia in hospitalized patients. Nephrol Dial Transplant ney NaCl cotransporter and reduces blood pressure. J Clin Invest 133:
22: 3471–3477, 2007 PubMed e158598, 2023 PubMed
7. Marti G, Schwarz C, Leichtle AB, Fiedler GM, Arampatzis S, 27. Saha B, Shabbir W, Takagi E, Duan XP, Leite Dellova DCA, Demko
Exadaktylos AK, et al.: Etiology and symptoms of severe hypokalemia J, et al.: Potassium activates mTORC2-dependent SGK1 phosphoryla-
in emergency department patients. Eur J Emerg Med 21: 46–51, 2014 tion to stimulate epithelial sodium channel: Role in rapid renal
PubMed responses to dietary potassium. J Am Soc Nephrol 34: 1019–1038, 2023
8. Ravioli S, Pluess E, Funk GC, Walter P, Schwarz C, Exadaktylos AK, PubMed
et al.: Dyskalemias in patients with acute kidney injury presenting to the 28. Sørensen MV, Saha B, Jensen IS, Wu P, Ayasse N, Gleason CE, et al.:
emergency department are common and independent predictors of Potassium acts through mTOR to regulate its own secretion. JCI Insight
adverse outcome. Int J Clin Pract 75: e13653, 2021 PubMed 5: e126910, 2019 PubMed
Nephrology Self-Assessment Program - Vol 23, No 2, June 2024 123
29. Yang L, Frindt G, Xu Y, Uchida S, Palmer LG: Aldosterone-dependent and on c-Src expression and renal K secretion in the cortical collecting duct. J
-independent regulation of Na1 and K1 excretion and ENaC in mouse kid- Biol Chem 280: 10790–10796, 2005 PubMed
neys. Am J Physiol Renal Physiol 319: F323–F334, 2020 PubMed 51. Ravioli S, Bahmad S, Funk GC, Schwarz C, Exadaktylos A, Lindner G:
30. Wouda RD, Gritter M, Karsten M, Michels EHA, Nieuweboer TM, Risk of electrolyte disorders, syncope, and falls in patients taking thiazide
Danser AHJ, et al.: Kaliuresis and intracellular uptake of potassium with diuretics: Results of a cross-sectional study. Am J Med 134: 1148–1154,
potassium citrate and potassium chloride supplements: A randomized 2021 PubMed
controlled trial. Clin J Am Soc Nephrol 18: 1260–1271, 2023 PubMed 52. Whelton PK, Carey RM, Aronow WS, Casey DE Jr., Collins KJ, Dennison
31. Preston RA, Afshartous D, Rodco R, Alonso AB, Garg D: Evidence for a Himmelfarb C, et al.: 2017 ACC/AHA/AAPA/ABC/ACPM/AGS/APhA/
gastrointestinal-renal kaliuretic signaling axis in humans. Kidney Int 88: ASH/ASPC/NMA/PCNA guideline for the prevention, detection, evalua-
1383–1391, 2015 PubMed tion, and management of high blood pressure in adults: A report of the
32. Al-Qusairi L, Ferdaus MZ, Pham TD, Li D, Grimm PR, Zapf AM, et al.: American College of Cardiology/American Heart Association Task Force on
Dietary anions control potassium excretion: It is more than a poorly Clinical Practice Guidelines. Hypertension 71: e13–e115, 2018 PubMed
absorbable anion effect. Am J Physiol Renal Physiol 325: F377–F393, 53. Hundemer GL, Imsirovic H, Vaidya A, Yozamp N, Goupil R, Madore
2023 PubMed F, et al.: Screening rates for primary aldosteronism among individuals
33. Tahaei E, Pham TD, Al-Qusairi L, Grimm R, Wall SM, Welling PA: with hypertension plus hypokalemia: A population-based retrospective
Pendrin regulation is prioritized by anion in high-potassium diets. Am J cohort study. Hypertension 79: 178–186, 2022 PubMed
Physiol Renal Physiol 324: F256–F266, 2023 PubMed 54. Gupta G, Teo AED, Swee DS, Loh LM, Chuah M, Loh WJ, et al.: Pro-
34. Khiatah B, Frugoli A, Carlson D: The clinical caveat for treating persistent longed hypokalemia and delayed diagnosis of primary aldosteronism: Clini-
hypokalemia in diabetic ketoacidosis. Cureus 15: e42272, 2023 PubMed cal course and risk factors [published online ahead of print Dec 22, 2023].
35. Johansen NJ, Christensen MB: A systematic review on insulin overdose J Clin Endocrinol Metab, doi:10.1210/clinem/dgad7522023 PubMed
cases: Clinical course, complications and treatment options. Basic Clin 55. Monticone S, D’Ascenzo F, Moretti C, Williams TA, Veglio F, Gaita F,
Pharmacol Toxicol 122: 650–659, 2018 PubMed et al.: Cardiovascular events and target organ damage in primary aldoste-
36. Miyakuni Y, Nakajima M, Kaszynski RH, Tarui T, Goto H, Yamaguchi ronism compared with essential hypertension: A systematic review and
Y: A case involving massive insulin overdose: Direct and indirect condi- meta-analysis. Lancet Diabetes Endocrinol 6: 41–50, 2018 PubMed
tions requiring extended management of serum potassium. Am J Case 56. Rizzolo K, Beck NM, Ambruso SL: Syndromes of pseudo-hyperaldoster-
Rep 21: e920078, 2020 PubMed onism. Clin J Am Soc Nephrol 17: 581–584, 2022 PubMed
37. Hung CH, Hua YM, Lee MY, Tsai YG, Yang KD: Evaluation of differ- 57. Seo H, Kim E: Incidence and determinants of piperacillin/tazobactam-
ent nebulized bronchodilators on clinical efficacy and hypokalemia in associated hypokalemia: A retrospective study. Antibiotics (Basel) 11:
asthmatic children. Acta Paediatr Taiwan 42: 287–290, 2001 PubMed 1138, 2022 PubMed
38. Kemperman CJ, Kuilman M, Njio LK: A retrospective and explorative 58. Schlingmann KP, de Baaij JHF: The genetic spectrum of Gitelman(-like)
study of hypokalemia in psychiatric disorders: A beta2-receptor related phe- syndromes. Curr Opin Nephrol Hypertens 31: 508–515, 2022 PubMed
nomenon. Eur Arch Psychiatry Neurol Sci 237: 161–165, 1988 PubMed 59. Whang R, Whang DD, Ryan MP: Refractory potassium repletion. A
39. Rizvi K, Abraham S, Gong J, Nogar J. Severe alkalemia and hypokalemia consequence of magnesium deficiency. Arch Intern Med 152: 40–45,
after chronic exposure to alkaline water. Am J Emerg Med 76: 273e5– 1992 PubMed
273e7, 2024 PubMed 60. Huang CL, Kuo E: Mechanism of hypokalemia in magnesium defi-
40. Kulahcioglu S, Baysal PK, Kup A, Imanov E, Uslu A, Demir S, et al.: ciency. J Am Soc Nephrol 18: 2649–2652, 2007 PubMed
Bidirectional ventricular tachycardia induced by respiratory alkalosis 61. Yang L, Frindt G, Palmer LG: Magnesium modulates ROMK channel-
mediated hypokalemia in a patient with acute ischemic heart failure. Pac- mediated potassium secretion. J Am Soc Nephrol 21: 2109–2116, 2010
ing Clin Electrophysiol 44: 2115–2118, 2021 PubMed PubMed
41. Weber F, Lehmann-Horn F: Hypokalemic periodic paralysis. In: GeneRe- 62. Adomako EA, Bilal S, Liu YL, Malik A, Van Buren PN, Shastri S, et al.:
views, edited by Adam MP, Feldman J, Mirzaa GM, et al., Seattle, WA, Idiopathic hypokalemia in lupus nephritis: A newly recognized entity.
University of Washington, 1993 Kidney360 2: 1553–1559, 2021 PubMed
42. Lin SH, Huang CL: Mechanism of thyrotoxic periodic paralysis. J Am 63. Kardalas E, Paschou SA, Anagnostis P, Muscogiuri G, Siasos G, Vryoni-
Soc Nephrol 23: 985–988, 2012 PubMed dou A: Hypokalemia: A clinical update. Endocr Connect 7: R135–R146,
43. Older J, Older P, Colker J, Brown R: Secretory villous adenomas that 2018 PubMed
cause depletion syndrome. Arch Intern Med 159: 879–880, 1999 PubMed 64. Reungjui S, Roncal CA, Sato W, Glushakova OY, Croker BP, Suga S,
44. Sitprija V: Altered fluid, electrolyte and mineral status in tropical disease, et al.: Hypokalemic nephropathy is associated with impaired angiogene-
with an emphasis on malaria and leptospirosis. Nat Clin Pract Nephrol 4: sis. J Am Soc Nephrol 19: 125–134, 2008 PubMed
91–101, 2008 PubMed 65. Suga S, Mazzali M, Ray PE, Kang DH, Johnson RJ: Angiotensin II type
45. Yu Q: Slc26a3 (DRA) in the gut: Expression, function, regulation, role 1 receptor blockade ameliorates tubulointerstitial injury induced by
in infectious diarrhea and inflammatory bowel disease. Inflamm Bowel chronic potassium deficiency. Kidney Int 61: 951–958, 2002 PubMed
Dis 27: 575–584, 2021 PubMed 66. Suga S, Yasui N, Yoshihara F, Horio T, Kawano Y, Kangawa K, et al.:
46. Knochel JP, Dotin LN, Hamburger RJ: Pathophysiology of intense phys- Endothelin a receptor blockade and endothelin B receptor blockade
ical conditioning in a hot climate. I. Mechanisms of potassium depletion. improve hypokalemic nephropathy by different mechanisms. J Am Soc
J Clin Invest 51: 242–255, 1972 PubMed Nephrol 14: 397–406, 2003 PubMed
47. Cao Y, Donaldson R, Lee D: “Summer hypokalemia” as an initial pre- 67. Weiss JN, Qu Z, Shivkumar K: Electrophysiology of hypokalemia and
sentation of cystic fibrosis in a morbidly obese African American adult: hyperkalemia. Circ Arrhythm Electrophysiol 10: e004667, 2017 PubMed
Case report. BMC Nephrol 21: 462, 2020 PubMed 68. Thu Kyaw M, Maung ZM: Hypokalemia-induced arrhythmia: A case
48. Wang WH, Giebisch G: Regulation of potassium (K) handling in the series and literature review. Cureus 14: e22940, 2022 PubMed
renal collecting duct. Pflugers Arch 458: 157–168, 2009 PubMed 69. Khan U, Afzal U, Kask L, Saeed N, Rajani R, Solheim E, et al.: “Ion-
49. Wang ZJ, Sun P, Xing W, Pan C, Lin DH, Wang WH: Decrease in die- Flux”: The natural course of ECG changes in severe hypokalemia. Curr
tary K intake stimulates the generation of superoxide anions in the kid- Probl Cardiol 49[1 Pt C]: 102158, 2024 PubMed
ney and inhibits K secretory channels in the CCD. Am J Physiol Renal 70. Kildegaard H, Brabrand M, Forberg JL, Platonov P, Lassen AT, Ekelund
Physiol 298: F1515–F1522, 2010 PubMed U: Prevalence and prognostic value of electrocardiographic abnormalities
50. Babilonia E, Wei Y, Sterling H, Kaminski P, Wolin M, Wang WH: in hypokalemia: A multicenter cohort study. J Intern Med 295: 544–556,
Superoxide anions are involved in mediating the effect of low K intake 2024 PubMed
124 Nephrology Self-Assessment Program - Vol 23, No 2, June 2024
71. Otaki Y, Watanabe T, Yamaguchi R, Tachibana S, Sato J, Kato S, et al.: 86. Chacko M, Fordtran JS, Emmett M: Effect of mineralocorticoid activity
Hypokalemia, kidney function, and clinical outcomes in heart failure on transtubular potassium gradient, urinary [K]/[Na] ratio, and fractional
with preserved ejection fraction. Circ J 88: 117–126, 2023 PubMed excretion of potassium. Am J Kidney Dis 32: 47–51, 1998 PubMed
72. Desai AS, Liu J, Pfeffer MA, Claggett B, Fleg J, Lewis EF, et al.: Incident 87. Choi MJ, Ziyadeh FN: The utility of the transtubular potassium gradient
hyperkalemia, hypokalemia, and clinical outcomes during spironolactone in the evaluation of hyperkalemia. J Am Soc Nephrol 19: 424–426, 2008
treatment of heart failure with preserved ejection fraction: Analysis of the PubMed
TOPCAT trial. J Card Fail 24: 313–320, 2018 PubMed 88. Chen EH, Hollander JE: When do patients need admission to a teleme-
73. Torlen K, Kalantar-Zadeh K, Molnar MZ, Vashistha T, Mehrotra R: try bed? J Emerg Med 33: 53–60, 2007 PubMed
Serum potassium and cause-specific mortality in a large peritoneal dialysis 89. Drew BJ, Califf RM, Funk M, Kaufman ES, Krucoff MW, Laks MM,
cohort. Clin J Am Soc Nephrol 7: 1272–1284, 2012 PubMed et al.; American Heart Association; Councils on Cardiovascular Nursing,
74. Pichitporn W, Kanjanabuch T, Phannajit J, Puapatanakul P, Chuengsa- Clinical Cardiology, and Cardiovascular Disease in the Young: Practice
man P, Parapiboon W, et al.: Efficacy of potassium supplementation in standards for electrocardiographic monitoring in hospital settings: An
hypokalemic patients receiving peritoneal dialysis: A randomized con- American Heart Association scientific statement from the Councils on
trolled trial. Am J Kidney Dis 80: 580–588.e1, 2022 PubMed Cardiovascular Nursing, Clinical Cardiology, and Cardiovascular Disease
75. Thongprayoon C, Mao MA, Kattah AG, Keddis MT, Pattharanitima P, in the Young: Endorsed by the International Society of Computerized
Erickson SB, et al.: Subtyping hospitalized patients with hypokalemia by Electrocardiology and the American Association of Critical-Care Nurses.
machine learning consensus clustering and associated mortality risks. Clin Circulation 110: 2721–2746, 2004 PubMed
Kidney J 15: 253–261, 2021 PubMed 90. Asmar A, Mohandas R, Wingo CS: A physiologic-based approach to the
76. Deak AT, Belic K, Meissl AM, Artinger K, Eller K, Rechberger B, et al.: treatment of a patient with hypokalemia. Am J Kidney Dis 60: 492–497,
Salivary potassium measured by genetically encoded potassium ion indi- 2012 PubMed
cators as a surrogate for plasma potassium levels in hemodialysis patients- 91. Bowling CB, Pitt B, Ahmed MI, Aban IB, Sanders PW, Mujib M, et al.:
a proof-of-concept study. Nephrol Dial Transplant 38: 757–763, 2023 Hypokalemia and outcomes in patients with chronic heart failure and
PubMed chronic kidney disease: Findings from propensity-matched studies. Circ
77. Rossi GP, Bernini G, Caliumi C, Desideri G, Fabris B, Ferri C, et al.; Heart Fail 3: 253–260, 2010 PubMed
PAPY Study Investigators: A prospective study of the prevalence of pri- 92. Wahr JA, Parks R, Boisvert D, Comunale M, Fabian J, Ramsay J, et al.;
mary aldosteronism in 1,125 hypertensive patients. J Am Coll Cardiol 48: Multicenter Study of Perioperative Ischemia Research Group: Preopera-
2293–2300, 2006 PubMed tive serum potassium levels and perioperative outcomes in cardiac surgery
78. Mulatero P, Rabbia F, Milan A, Paglieri C, Morello F, Chiandussi L, patients. JAMA 281: 2203–2210, 1999 PubMed
et al.: Drug effects on aldosterone/plasma renin activity ratio in primary 93. Lee S, Kang E, Yoo KD, Choi Y, Kim DK, Joo KW, et al.: Lower serum
aldosteronism. Hypertension 40: 897–902, 2002 PubMed potassium associated with increased mortality in dialysis patients: A
79. Burrello J, Buffolo F, Domenig O, Tetti M, Pecori A, Monticone S, et al.: nationwide prospective observational cohort study in Korea. PLoS One
Renin-angiotensin-aldosterone system triple-A analysis for the screening of 12: e0171842, 2017 PubMed
primary aldosteronism. Hypertension 75: 163–172, 2020 PubMed 94. Lin SH, Lin YF, Chen DT, Chu P, Hsu CW, Halperin ML: Laboratory
80. Palmer BF: A physiologic-based approach to the evaluation of a patient tests to determine the cause of hypokalemia and paralysis. Arch Intern
with hypokalemia. Am J Kidney Dis 56: 1184–1190, 2010 PubMed Med 164: 1561–1566, 2004 PubMed
81. Wu KL, Cheng CJ, Sung CC, Tseng MH, Hsu YJ, Yang SS, et al.: Iden- 95. Hunter RW, Bailey MA: Horses for courses: What is the best oral potas-
tification of the causes for chronic hypokalemia: Importance of urinary sium supplementation strategy? Clin J Am Soc Nephrol 18: 1250–1253,
sodium and chloride excretion. Am J Med 130: 846–855, 2017 PubMed 2023 PubMed
82. Lin SH, Halperin ML: Hypokalemia: A practical approach to diagnosis 96. Wang CC, Shiang JC, Huang WT, Lin SH: Hypokalemic paralysis as
and its genetic basis. Curr Med Chem 14: 1551–1565, 2007 PubMed primary presentation of Fanconi syndrome associated with Sj€ogren syn-
83. Hooft van Huysduynen EJ, Hulshof PJ, van Lee L, Geelen A, Feskens drome. J Clin Rheumatol 16: 178–180, 2010 PubMed
EJ, van ’t Veer P, et al.: Evaluation of using spot urine to replace 24 h 97. Cohn JN, Kowey PR, Whelton PK, Prisant LM: New guidelines for
urine sodium and potassium excretions. Public Health Nutr 17: 2505– potassium replacement in clinical practice: A contemporary review by the
2511, 2014 PubMed National Council on Potassium in Clinical Practice. Arch Intern Med
84. Polonia J, Lobo MF, Martins L, Pinto F, Nazare J: Estimation of popula- 160: 2429–2436, 2000 PubMed
tional 24-h urinary sodium and potassium excretion from spot urine 98. Hamill RJ, Robinson LM, Wexler HR, Moote C: Efficacy and safety of
samples: Evaluation of four formulas in a large national representative potassium infusion therapy in hypokalemic critically ill patients. Crit
population. J Hypertens 35: 477–486, 2017 PubMed Care Med 19: 694–699, 1991 PubMed
85. Li J, Ma H, Lei Y, Wan Q: Diagnostic value of parameters from a spot 99. Tuttle A, Fitter S, Hua H, Moussavi K: The effects of magnesium coad-
urine sample for renal potassium loss in hypokalemia. Clin Chim Acta minstration during treatment of hypokalemia in the emergency depart-
511: 221–226, 2020 PubMed ment. J Emerg Med 63: 399–413, 2022 PubMed
Nephrology Self-Assessment Program - Vol 23, No 2, June 2024 125
Article
Updates in the Risk and Treatment of Hyperkalemia in
CKD Patients
Alice M. Sheridan, MD, FASN
Renal Division, Brigham and Women’s Hospital, Boston, Massachusetts
also note that the safety of a liberal potassium diet in advanced CKD 12. de Boer IH, Khunti K, Sadusky T, Tuttle KR, Neumiller JJ, Rhee CM, et al.:
has not been shown. The final KDIGO suggestion is that interven- Diabetes management in chronic kidney disease: A consensus report by the
American Diabetes Association (ADA) and Kidney Disease: Improving
tional trials should be done to define the optimal potassium intake,
Global Outcomes (KDIGO). Diabetes Care 45: 3075–3090, 2022 PubMed
and, in the interim, an overall healthy diet, such as the Mediterranean 13. Rassi AN, Cavender MA, Fonarow GC, Cannon CP, Hernandez AF,
diet, should be recommended to patients. Peterson ED, et al.: Temporal trends and predictors in the use of aldoste-
rone antagonists post-acute myocardial infarction. J Am Coll Cardiol 61:
Monitoring 35–40, 2013 PubMed
14. Rossignol P, Lainscak M, Crespo-Leiro MG, Laroche C, Piepoli MF,
Patients are generally monitored for hyperkalemia at initiation and after
Filippatos G, et al.; Heart Failure Long-Term Registry Investigators
dose increase of RAAS inhibitors. A deep learning model of detection Group: Unravelling the interplay between hyperkalaemia, renin-
of hyperkalemia from an electrocardiogram was trained to detect angiotensin-aldosterone inhibitor use and clinical outcomes. Data from
K .5.5 mEq/L using 577,581 electrocardiograms (ECGs) from 9222 chronic heart failure patients of the ESC-HFA-EORP Heart Failure
449,380 patients and validated on 61,965 patients with an area under Long-Term Registry. Eur J Heart Fail 22: 1378–1389, 2020 PubMed
the curve of 0.853 to 0.883. This tool performed well to exclude 15. Epstein M, Reaven NL, Funk SE, McGaughey KJ, Oestreicher N, Knis-
hyperkalemia with a negative predictive value .99% (27). However, pel J: Evaluation of the treatment gap between clinical guidelines and the
utilization of renin-angiotensin-aldosterone system inhibitors. Am J
prospective studies are needed to assess the overall utility of this tool in Manag Care 21[Suppl]: S212–S220, 2015 PubMed
clinical practice. 16. Leon SJ, Whitlock R, Rigatto C, Komenda P, Bohm C, Sucha E, et al.:
Hyperkalemia-related discontinuation of renin-angiotensin-aldosterone
system inhibitors and clinical outcomes in CKD: A population-based
Conclusion cohort study. Am J Kidney Dis 80: 164–173.e1, 2022 PubMed
Chronic hyperkalemia is a significant problem among CKD patients 17. Palmer BF, Clegg DJ: Managing hyperkalemia to enable guideline-
and inhibits the optimization of heart failure and CKD outcomes recommended dosing of renin-angiotensin-aldosterone system inhibitors.
Am J Kidney Dis 80: 158–160, 2022 PubMed
with RAAS inhibitors. The use of newer potassium binders, thiazide
18. Agarwal R, Pitt B, Palmer BF, Kovesdy CP, Burgess E, Filippatos G,
and loop diuretics, and correction of acidosis, and possibly dietary et al.: A comparative post hoc analysis of finerenone and spironolactone
restriction, may allow the use of these agents. in resistant hypertension in moderate-to-advanced chronic kidney disease.
Clin Kidney J 16: 293–302, 2022 PubMed
19. Neuen BL, Oshima M, Agarwal R, Arnott C, Cherney DZ, Edwards R, et al.:
References Sodium-glucose cotransporter 2 inhibitors and risk of hyperkalemia in people
1. Palmer BF, Clegg DJ: Physiology and pathophysiology of potassium homeo- with type 2 diabetes: A meta-analysis of individual participant data from ran-
stasis: Core curriculum 2019. Am J Kidney Dis 74: 682–695, 2019 PubMed domized, controlled trials. Circulation 145: 1460–1470, 2022 PubMed
2. Ayach T, Nappo RW, Paugh-Miller JL, Ross EA: Postoperative hyperka- 20. Neuen BL, Oshima M, Perkovic V, Agarwal R, Arnott C, Bakris G, et al.
lemia. Eur J Intern Med 2: 106–111, 2015 PubMed Effects of canagliflozin on serum potassium in people with diabetes and
3. Perazella MA, Biswas P: Acute hyperkalemia associated with intravenous chronic kidney disease: The CREDENCE trial. Eur Heart J 42: 4891–4901,
epsilon-aminocaproic acid therapy. Am J Kidney Dis 33: 782–785, 1999 2021 PubMed
PubMed 21. Agarwal R, Rossignol P, Romero A, Garza D, Mayo MR, Warren S,
4. Segawa K, Nishiyama M, Mori I, Kubota T, Takahashi MP: Hyperkale- et al.: Patiromer versus placebo to enable spironolactone use in patients
mic periodic paralysis associated with a novel missense variant located in with resistant hypertension and chronic kidney disease (AMBER): A
the inner pore of Nav1.4. Brain Dev 45: 205–211, 2023 PubMed phase 2, randomised, double-blind, placebo-controlled trial. Lancet 394:
5. Wilson FH, Disse-Nicodeme S, Choate KA, Ishikawa K, Nelson-Williams 1540–1550, 2019 PubMed
C, Desitter I, et al: Human hypertension caused by mutations in WNK 22. Noel JA, Bota SE, Petrcich W, Garg AX, Carrero JJ, Harel Z, et al.: Risk
kinases. Science 293: 1107–1112, 2001 PubMed of hospitalization for serious adverse gastrointestinal events associated
6. Grimm PR, Coleman R, Delpire E, Welling PA: Constitutively active with sodium polystyrene sulfonate use in patients of advanced age. JAMA
SPAK causes hyperkalemia by activating NCC and remodeling distal Intern Med 179: 1025–1033, 2019 PubMed
tubules. J Am Soc Nephrol 28: 2597–2606, 2017 PubMed 23. Palmer BF, Carrero JJ, Clegg DJ, Colbert GB, Emmett M, Fishbane S,
7. McCormick JA, Ellison DH: Nephron remodeling underlies hyperkalemia et al.: Clinical management of hyperkalemia. Mayo Clin Proc 96: 744–762,
in familial hyperkalemic hypertension. J Am Soc Nephrol 28: 2555–2557, 2021 PubMed
2017 PubMed 24. Laureati P, Xu Y, Trevisan M, Schalin L, Mariani I, Bellocco R, et al.:
8. Cornelius RJ, Maeoka Y, McCormick JA: Renal effects of cullin 3 muta- Initiation of sodium polystyrene sulphonate and the risk of gastrointesti-
tions causing familial hyperkalemic hypertension. Curr Opin Nephrol nal adverse events in advanced chronic kidney disease: a nationwide
Hypertens 32: 335–343, 2023 PubMed study. Nephrol Dial Transplant 35: 1518–1526, 2020 PubMed
9. Heidenreich PA, Bozkurt B, Aguilar D, Allen LA, Byun JJ, Colvin MM, 25. Bounthavong M, Butler J, Dolan CM, Dunn JD, Fisher KA, Oestreicher
et al.: 2022 AHA/ACC/HFSA guideline for the management of heart N, et al.: Cost-effectiveness analysis of patiromer and spironolactone
failure: Executive summary: A report of the American College of therapy in heart failure patients with hyperkalemia. PharmacoEconomics
Cardiology/American Heart Association Joint Committee on Clinical 36: 1463–1473, 2018 PubMed
Practice Guidelines. Circulation 145: e876–e894, 2022 PubMed 26. Clase CM, Carrero JJ, Ellison DH, Grams ME, Hemmelgarn BR, Jardine
10. McDonagh TA, Metra M, Adamo M, Gardner RS, Baumbach A, B€ohm MJ, et al.; Conference Participants: Potassium homeostasis and manage-
M, et al.; ESC Scientific Document Group: 2021 ESC guidelines for the ment of dyskalemia in kidney diseases: Conclusions from a Kidney Dis-
diagnosis and treatment of acute and chronic heart failure. Eur Heart J ease: Improving Global Outcomes (KDIGO) Controversies conference.
42: 3599–3726, 2021 PubMed Kidney Int 97: 42–61, 2020 PubMed
11. KDIGO CKD Work Group: KDIGO 2024 clinical practice guideline 27. Galloway CD, Valys AV, Shreibati JB, Treiman DL, Petterson FL, Gundotra
for the evaluation and management of chronic kidney disease. Kidney VP, et al.; Development and validation of a deep-learning model to screen
Disease: Improving Global Outcomes (KDIGO) CKD Work Group. for hyperkalemia from the electrocardiogram. JAMA Cardiol 4: 428–436,
Kidney Int 105[4S]: S117–S314, 2024 PubMed 2019 PubMed
128 Nephrology Self-Assessment Program - Vol 23, No 2, June 2024
Article
Metabolic Acidosis
Mengyao Tang, MD, MPH, FASN and Andrew Z. Fenves, MD, FASN
Division of Nephrology, Department of Medicine, Massachusetts General Hospital and Harvard Medical School, Boston,
Massachusetts
Respiratory Compensation
Learning Objectives The pathophysiologic process of metabolic acidosis results in a
1. To evaluate metabolic acidosis including high anion gap decrease in serum bicarbonate and pH, which triggers a respiratory
compensatory response. To counteract acidemia and raise the pH
metabolic acidosis and hyperchloremic metabolic acidosis
toward the normal range, there is an increase in minute ventilation
2. To apply knowledge of the anion gap to perform a differential aimed at reducing arterial partial pressure of carbon dioxide
diagnosis of metabolic acidosis (PaCO2). This compensatory response is acute, initiating within
1 hour and typically completed within 12 to 24 hours (3). The respi-
3. To define the management strategies of high anion gap
ratory response is similar irrespective of the underlying cause of the
metabolic acidosis and hyperchloremic metabolic acidosis, metabolic acidosis.
including renal tubular acidosis, on the basis of Several equations and rules are available clinically to determine
pathophysiology and etiology whether respiratory compensation is appropriate by predicting
expected PaCO2. Should the PaCO2 deviate from the expected range,
a concurrent secondary respiratory acid-base disorder may be impli-
cated. The Winter’s formula, that is, PaCO251.53HCO31862, is
Introduction a frequently used equation, although it is important to note that it
Metabolic acidosis is characterized by an increased concentration of was originally derived from a pediatric population (4). Another quick
hydrogen ions and a decreased concentration of bicarbonate bedside approach to approximately predict the expected PaCO2 is
(HCO3) in the body. It is typically identified by a low serum total known as the “thumb rule,” first described by M. Fulop (Figure 1).
carbon dioxide (CO2) level in basic metabolic panels, which mainly The thumb rule implies that by covering the number before the deci-
reflects HCO3 (about 95%) but also includes dissolved CO2 and mal point in the arterial pH value with your thumb, the PaCO2 can
carbonic acid (H2CO3). This differs from a blood gas that reports a be approximated to the visible decimal fraction of the arterial pH (5).
calculated HCO3 concentration and the partial pressure of CO2 sep- For instance, if the pH is 7.30, the PaCO2 should be around
arately. Consequently, serum total CO2 level is usually 2–4 mEq/L 30 mm Hg. It is crucial to recognize that these calculations are most
higher than arterial HCO3 concentration. reliable in cases of mild to moderately severe metabolic acidosis, where
Importantly, the presence of metabolic acidosis does not neces- bicarbonate levels range from 7 to 22 mEq/L. In instances of more
sarily result in acidemia, which is defined as an arterial pH of less severe metabolic acidosis, with bicarbonate levels falling below
than 7.35. This is due to respiratory compensation (note that com- 7 mEq/L, the PaCO2 is expected to be maximally reduced to the
pensation never corrects the primary disorder back to normal) or the 8–12 mm Hg range.
presence of other coexisting acid-base disorders. Similarly, a low
serum bicarbonate level is not always indicative of metabolic acido-
sis; it may also result from appropriate metabolic compensation to Renal Response
primary respiratory alkalosis. The kidneys play a central role in regulating serum bicarbonate con-
In clinical practice, classifying metabolic acidosis on the basis of centration and maintaining acid-base homeostasis, through reclama-
the serum anion gap (AG) is pragmatically valuable for informing tion of filtered HCO3 and excretion of acid. Normally, nearly all
the differential diagnosis. The AG is an important mathematic con- filtered HCO3 is reclaimed, primarily in the proximal tubule, but
struct that compares the serum sodium (Na) concentration with the also in the thick ascending limb and distal nephron, leaving the
sum of the serum chloride (Cl) and HCO3 concentrations. Meta- urine HCO3 free. The excretion of acid, which is physiologically
bolic acidosis can be categorized into two groups on the basis of equivalent to the production of new HCO3, involves the combina-
whether the serum AG is elevated. High AG metabolic acidosis tion of hydrogen ions with titratable acids (such as phosphate) and
(HAGMA) is characterized by an elevated AG, and non-AG meta- ammonia to form ammonium (6).
bolic acidosis is characterized by a normal AG, the latter character- The kidneys can adapt to acid load by increasing excretion of
ized by hyperchloremia and often referred to as hyperchloremic both titratable acids (including mostly phosphate, but also sulfate,
metabolic acidosis. The concept of an AG, first introduced by James hippurate, and citrate) and ammonium to maintain normal serum
L. Gamble in 1923, continues to be a relevant and valuable tool in HCO3 concentration. However, under conditions with a larger acid
contemporary medical practice (1,2). load (or base loss) that leads to metabolic acidosis, the kidneys’
Figure 2. Gamblegram; anion gap (AG); d AG/d HCO3 ratio. (A) A Gamblegram displays two bars, each representing the sum of cations (pri-
marily sodium [Na]) and anions (primarily chloride [Cl], bicarbonate [HCO3], and albumin), respectively. The total concentrations of cations
and anions measured in milliequivalents per liter are equal. (B) The simplified version of Gamblegram focuses on the concentrations of the three
principal electrolytes—Na, Cl, HCO3—introducing an artificial AG. The serum AG is calculated using the formula AG5Na2(Cl1HCO3),
with a normal range typically between 8 and 12 mEq/L. (C) In instances of high AG metabolic acidosis (HAGMA) caused by the accumulation
of any strong acids other than hydrochloric acid, the HCO3 concentration decreases while Cl remains stable, leading to an increased AG. Typi-
cally, the rise in AG is reciprocally proportional to the decrease in HCO3, ideally following a 1:1 d AG/d HCO3 ratio. (D) In instances of non-
AG or hyperchloremic metabolic acidosis, typically due to HCO3 loss or diminished renal acid excretion, the reduction in HCO3 is offset by a
corresponding increase in Cl, thereby maintaining an unchanged AG and d AG/d HCO3 ratio approaching zero. (E) In instances of coexisting
HAGMA and metabolic alkalosis, a markedly increased d AG exceeds a decrease in d HCO3 concentration, thereby distorting the 1:1 ratio.
Reprinted from ref. 1, with permission.
detected through newborn screening, they are generally rare and are L-lactic acidosis is the most common cause of HAGMA in hos-
most commonly diagnosed during the neonatal period or early pitalized patients and is often associated with poor clinical outcomes
childhood (16). Nonetheless, they should also be considered in (20). The causes of lactic acidosis are commonly divided into two
adults presenting with unexplained neurologic disorders, accompa- types: type A, associated with evident tissue hypoxia in shock or
nied by HAGMA, hypoglycemia, hyperammonemia, or acute liver after cardiopulmonary arrest; and type B, where tissue hypoxia is
abnormalities (17). Early detection and management are crucial to absent or not readily apparent, often because of impaired cellular or
prevent irreversible neurologic damage (16). Long-term treatment mitochondrial metabolism or regional areas of ischemia (20). The
strategies often involve dietary modification and enzyme replace- common causes of type A lactic acidosis include sepsis, severe hypo-
ment therapies (18). volemia, significant GI bleeding, cardiac failure, and cardiopulmo-
nary arrest. Treatment of type A lactic acidosis should target the
Lactic Acidosis correction of underlying causes, and often requires hemodynamic
The second category is lactic acidosis, including both L-lactic acidosis support with vasopressors or inotropes. The use of alkali therapies,
and D-lactic acidosis. The L optical isomer of lactate (L-lactate) is the such as intravenous (iv) sodium bicarbonate, is controversial. Propo-
predominant form in humans, and its accumulation represents the nents suggest raising arterial pH can improve tissue perfusion,
majority of clinical cases of lactic acidosis. Lactate, a metabolic prod- reduce arterial vasodilation, improve left ventricular contractility,
uct of pyruvate metabolism, is formed through a reaction catalyzed by and reduce the risks of arrhythmias. In contrast, opponents caution
lactate dehydrogenase, which also involves the conversion of reduced it may lead to volume overload, hyperosmolarity, and decreased ion-
to oxidized forms of nicotinamide adenine dinucleotide (NADH to ized calcium, and potentially exacerbate intracellular acidosis because
NAD1). Normally, the body produces approximately 20 mmol/kg of CO2 accumulation (20,21). Considering the divergent views, we
lactate daily, predominantly through anaerobic glycolysis or alanine believe it is reasonable to administer iv bicarbonate in critically ill
deamination in skeletal muscles (19). Conversely, lactate is primarily patients with severe lactic acidosis and arterial pH less than 7.1,
metabolized back to pyruvate in the mitochondria of the liver, but it but rapid administration of large quantities of bicarbonate should
is also consumed in the kidneys, heart, and other tissues (19). Lactic be avoided. The causes of type B lactic acidosis include a variety
acidosis occurs when lactate production exceeds its consumption. of toxins, drugs (e.g., b-adrenergic agonists, metformin, propofol,
Nephrology Self-Assessment Program - Vol 23, No 2, June 2024 131
linezolid, and nucleoside or nucleotide reverse transcriptase inhibi- context of hyperglycemia, when calculating the serum AG, the origi-
tors), various malignancies (the “Warburg effect”), and thiamine nal measured serum Na should be used instead of the Na concentra-
deficiencies (20). One classic example is metformin-associated lactic tion corrected for hyperglycemia (1). Patients with DKA typically
acidosis, usually resulting from an acute overdose or accumulation present with an initial AG above 20 mEq/L. However, as DKA
of a therapeutic dose in patients with acute kidney injury or advanced resolves, HAGMA can convert to hyperchloremic metabolic acidosis
kidney disease. Although metformin levels can confirm the diagnosis, in individuals with improving kidney functions. This conversion is
they are generally not helpful in acute settings, because results take due to the urinary excretion of ketoacid anions (“potential HCO3”),
several days and should not delay treatment. Severe cases of because improved kidney function allows for the excretion of sodium
metformin-associated lactic acidosis with lactate .20 mmol/L or arte- and potassium salts of ketoacid anions, reducing the AG without
rial pH ,7.1 require emergent renal replacement therapy, with affecting serum HCO3 concentration (28). This process is highly
hemodialysis generally preferred over continuous renal replacement dependent on intact kidney function and is usually not observed in
therapy for expedited metformin elimination (22). patients with ESKD. Management of DKA includes fluid replace-
In contrast, D-lactic acidosis is a rare form of HAGMA that ment, electrolyte repletion, and insulin administration. Although
can occur in patients with short bowel syndrome or other types of many patients with DKA initially present with hyperkalemia, they
GI malabsorption (23). This condition arises from the metabolism often have a large total body potassium deficit. Insulin therapy shifts
of unabsorbed glucose and starch into D-lactic acid by intestinal potassium intracellularly and results in a rapid fall in serum potas-
bacteria. Humans poorly metabolize D-lactic acid, and its systemic sium level, unmasking the true state of potassium balance and neces-
absorption can lead to D-lactic acidosis. Notably, D-lactate is sitating vigilant iv potassium replacement. Alkali therapy such as iv
excreted more rapidly than L-lactate, because the sodium-lactate sodium bicarbonate is typically not indicated for most patients with
cotransporter in the proximal tubule specifically recognizes L-lactate. DKA (26).
As a result, D-lactic acidosis might present with a normal or unex- Alcoholic ketoacidosis typically affects malnourished individuals
pectedly low AG relative to the decrease in bicarbonate (12,23). The with chronic alcohol use disorder who have recently ceased active
clinical manifestations of D-lactic acidosis include episodic meta- drinking due to nausea, vomiting, or abdominal pain (29). As alco-
bolic acidosis (usually occurring after high carbohydrate meals) and hol levels drop, increased catecholamine and cortisol levels amplify
altered mental status (such as confusion, cerebellar ataxia, slurred the fasting hormonal response, leading to increased fatty acid deliv-
speech, incontinence, and nystagmus) (23). It is important to recog- ery to the liver with increased hepatic ketone genesis. Patients
nize that standard laboratory “lactate” assays only routinely measure with alcoholic ketoacidosis often present with concomitant electro-
L-lactate. Quantification of D-lactate must be specifically requested lyte disorders such as hypokalemia, hypophosphatemia, and hypo-
if there is clinical suspicion of D-lactic acidosis. Treatment includes magnesemia. Treatment of alcoholic ketoacidosis usually includes
sodium bicarbonate administration and oral antibiotics (metronida- administration of dextrose solutions, unless the patients are overtly
zole, neomycin, or vancomycin). A low-carbohydrate diet is crucial hyperglycemic, in which case insulin should be administered. Thia-
for preventing the development of D-lactic acidosis in susceptible mine is recommended to be administered alongside dextrose solu-
patients (23). Other causes of D-lactic acidosis include metabolic tions. Notably, dextrose solutions should be withheld initially in
products in diabetic ketoacidosis (DKA) and propylene glycol intox- patients with severe hypokalemia until serum K levels are adequately
ication (1,23). repleted, because dextrose can stimulate insulin release and lead to
intracellular potassium shifts.
Ketoacidosis
The third category is ketoacidosis caused by accumulation of ketone Renal Failure
bodies, acetoacetic acid, and b-hydroxybutyric acid, with the latter Renal failure, whether due to acute kidney injury or progression of
typically being more dominant. Ketones can be detected in routine CKD, represents the fourth category of HAGMA. Our focus here is
urinalysis using nitroprusside tests, but this only identifies the pres- on metabolic acidosis in the context of CKD. The development of
ence of acetoacetic acid. In contrast, serum ketones are typically metabolic acidosis in CKD involves two principal factors: the daily
directly assayed as b-hydroxybutyrate levels. Ketones serve as an acid load from the diet and the diminished acid excretory capacity
energy source when glucose availability is limited. They are predom- of the diseased kidneys. A typical Western diet generates about
inantly produced by the liver, stimulated by a combination of insu- 50–100 mEq of nonvolatile acid each day. In patients with CKD,
lin deficiency and glucagon excess. Common causes of ketoacidosis the diseased kidneys’ ability to excrete this acid load is compromised.
include DKA, alcoholic ketoacidosis, and starvation ketoacidosis. Initially, with a mild reduction in GFR, the kidneys can maintain a
For example, mild ketosis can develop after 12 hours of fasting and relatively normal serum HCO3 concentration by increasing ammo-
usually stabilizes at 8 to 10 mmol/L when hepatic generation and nium excretion per functioning nephron (30). However, as more
ketone utilization reach a steady state (24). Fasting ketosis is gener- nephrons are lost with further deterioration of kidney function, total
ally considered nondetrimental, except in situations with high glu- ammonium excretion decreases. In contrast, titratable acid excretion
cose requirements, such as in pregnant or lactating women (25). remains relatively unchanged until there is severe reduction in GFR
DKA is more common in individuals with type 1 diabetes but (30). In general, the occurrence of metabolic acidosis rises as GFR
can also occur in those with type 2 diabetes (26). Patients with DKA declines. Most early-stage CKD patients with metabolic acidosis
often present with marked hyperglycemia; however, it is important to have a normal serum AG. As CKD advances, particularly approach-
note the rising incidence of euglycemic DKA with the increased use ing ESKD, serum AG increases because of the accumulation of vari-
of sodium-glucose cotransporter-2 (SGLT2) inhibitors (27). In the ous organic and inorganic acids like sulfate, phosphate, urate, and
132 Nephrology Self-Assessment Program - Vol 23, No 2, June 2024
hippurate, a condition sometimes referred to as “uremic acidosis.” HAGMA but is primarily due to lactic acidosis (34). Diagnosis of
Metabolic acidosis in CKD is linked to a higher risk of CKD 5-oxoproline acidosis can be confirmed through the measurement of
progression, bone demineralization, muscle wasting, and increased urine organic acid assay. Treatment primarily consists of stopping
mortality in numerous physiologic and epidemiologic studies (30). acetaminophen use and, in some instances, administering N-acetyl-
Although definitive benefits from large clinical trials are lacking, oral cysteine (35). Additionally, it is worth noting that inherited glutathi-
alkali therapies (most commonly sodium bicarbonate), aimed at one synthetase deficiency is a rare but potential cause of
maintaining serum bicarbonate within the normal range, are com- 5-oxoproline acidosis.
monly recommended for CKD patients with metabolic acidosis (30). Sodium thiosulfate used to treat calciphylaxis is another increas-
ingly recognized cause of HAGMA, although the exact mechanism
Drug Ingestions of action remains unclear (36,37). Not an acid itself, sodium thio-
The fifth category is HAGMA resulting from the ingestion of cer- sulfate may cause acidosis due to its oxidation to sulfate by the liver
tain drugs. Drugs can cause metabolic acidosis through various or intestinal bacteria. Typically, sodium thiosulfate is rapidly
mechanisms (31), such as type B lactic acidosis with metformin. excreted in patients with intact kidney function. However, many
This section highlights a few classic examples with more complex patients receiving this treatment for calciphylaxis have advanced
mechanisms, including salicylate, chronic acetaminophen use, and CKD or are on hemodialysis, resulting in decreased renal excretion
sodium thiosulfate. and a consequent increase in metabolism to sulfate (38). It is impor-
Salicylates are among the oldest medications still in common tant to monitor serum HCO3 levels in calciphylaxis patients receiv-
clinical use today, found in various prescription and over-the-counter ing sodium thiosulfate. Using a high bicarbonate dialysate may
formulations, such as aspirin (acetylsalicylic acid) tablets and analgesic mitigate the metabolic acidosis. If persistent severe metabolic acido-
mixtures. Their wide availability and accessibility make salicylate tox- sis occurs despite adjustments in dialysate bicarbonate for dialysis
icity a common and sometimes fatal occurrence. Salicylate toxicity patients or oral bicarbonate supplementation for non–dialysis-
can manifest with various acid-base disorders, depending on the tim- dependent CKD patients, sodium thiosulfate use should be
ing of presentation, but the most characteristic acid-base disorder in discontinued.
adults is a mixed respiratory alkalosis and HAGMA. The former
results from the direct stimulation of the medullary respiratory center,
increasing ventilation (32). Additionally, salicylates can impair oxida- Toxin Ingestions
tive phosphorylation, inhibit citric acid cycle dehydrogenases, acceler- Ingestion of toxic alcohols and glycols, such as methanol, ethylene
ate glycolysis, and stimulate lipolysis, leading to the accumulation of glycol, diethylene glycol, and propylene glycol, can lead to
acids such as lactic acid and ketoacid, contributing to HAGMA along HAGMA. These substances, although not inherently toxic or acidic,
with the salicylic acid itself (32). In cases of suspected salicylate toxic- are metabolized into potent, toxic organic acids, primarily by alcohol
ity, serum salicylate levels should be measured. However, it is critical dehydrogenase and aldehyde dehydrogenase. For instance, methanol
to recognize that an elevation in serum salicylate concentration may is metabolized into the toxic metabolite formic acid, which can lead
not immediately follow clinical symptom onset and may not correlate to retinal injury; ethylene glycol is broken down into glycolic acid,
with the severity of the acidosis (32). No specific antidote exists for glyoxylic acid, and oxalic acid, resulting in oxalate crystal formation
salicylate toxicity. Treatment options include early GI binding with and acute kidney injury (39). Early recognition and prompt treat-
activated charcoal or whole-bowel irrigation. In cases of metabolic aci- ment, including administration of the alcohol dehydrogenase inhibi-
dosis, urinary alkalinization can be used to enhance renal clearance of tor fomepizole and initiation of emergent dialysis in severe cases, are
salicylates and should be achieved with iv bicarbonate to maintain a critical in preventing fatal outcomes (39).
urine pH above 7.5 (32). Oral bicarbonate is contraindicated because A challenge in the management of these cases is the measure-
it increases gastric pH, potentially accelerating the dissolution and ment of these toxic alcohol or glycol concentrations, because they
absorption of any remaining salicylate. Hemodialysis is the most effec- are typically done through gas chromatography at specialized refer-
tive method for removing salicylates and is indicated in cases of severe ence laboratories. This may delay the results and hinder their practi-
symptoms or complications such as significant fluid and electrolyte cal use in clinical decision making. However, a key clue of potential
disturbances, cerebral edema, acute respiratory distress syndrome, and toxic alcohol/glycol exposure is an elevated serum osmolal gap,
acute kidney injury, regardless of the salicylate level (32). Dialysis which is defined as the difference between measured and calculated
should also be initiated when plasma salicylate levels are above serum osmolality. The serum osmolarity can be directly measured
90 mg/dl, even in the absence of severe symptoms, or above 80 mg/dl by an osmometer. The calculated osmolarity is obtained using the
if kidney function is impaired (32). following formula: 23Na (mEq/L)1blood urea nitrogen (mg/dl)/
The prolonged use of acetaminophen at therapeutic or even 2.81glucose (milligrams per deciliter)/18. A normal osmolal gap is
subtherapeutic doses is increasingly recognized as an underdiagnosed usually less than 6 mOsm/L, and an increased osmolal gap above
cause of HAGMA, particularly among chronically ill and malnour- 10 mOsm/L suggests potential ingestions of substances such as toxic
ished women (33). This is due to the accumulation of 5-oxoproline alcohols or glycols, assuming significant ethanol ingestion is
(pyroglutamic acid), with the depletion of glutathione and cysteine excluded. Ethanol ingestion is the most common cause of an ele-
during acetaminophen detoxification accelerating the generation of vated osmolal gap, and, in such cases, the formula for calculated
5-oxoproline (33,34). Impaired kidney function is also a risk factor. osmolarity should account for the contribution of ethanol by further
This condition is distinct from severe liver failure resulting from adding ethanol (mg/dl)/4.6 in the equation. Furthermore, it is worth
acute acetaminophen overdose, which can similarly present with noting that a poisoned patient can present with both a normal or
Nephrology Self-Assessment Program - Vol 23, No 2, June 2024 133
high osmolal gap, and a normal or high serum AG, dependent on UAG is calculated with the following formula: urine Na1urine
the specific type of toxin ingestion and the stage of metabolism (40). K2urine Cl. Early studies found a strong inverse correlation
Toluene abuse, often through glue sniffing or spray paint inha- between UAG and urinary ammonium excretion, thereby advocat-
lation, is another important consideration in toxin ingestion. Tolu- ing its use as a surrogate of urinary ammonium excretion (44,45).
ene is metabolized into benzoic acid, which is further metabolized Typically, a negative UAG less than 220 mEq/L suggests increased
to hippuric acid. Although the accumulation of benzoic and hippu- ammonium excretion, pointing to GI-induced metabolic acidosis.
ric acids can lead to an elevated AG, hippurate is both freely filtered Notably, UAG is a less reliable estimator of urinary ammonium excre-
and actively secreted by the proximal tubule, leading to rapid renal tion in patients with impaired kidney function or with HAGMA,
excretions (41). As a result, many patients present with hyperchlore- because of increased urinary excretion of unmeasured anions. Further-
mic metabolic acidosis with a normal AG (41). Other important more, recent studies conclude that UAG is more reflective of dietary
features are volume contraction and severe hypokalemia. During the intake of Na, K, and Cl than urinary ammonium excretion, and the
early phases of this disorder, a significant amount of sodium is lost normal range for UAG has increased because of changes in dietary
with hippurate in the urine, leading to volume contraction and sec- intake (46). Alternatively, UOG, the difference between measured
ondary hyperaldosteronism. Additionally, the distal delivery of urine osmolarity and calculated osmolarity ([23(Na1K)]1[urea
sodium, together with hyperaldosteronism, promotes substantial uri- nitrogen in mg/dl]/2.81[glucose in mg/dl]/18), has been proposed as
nary potassium excretion. Over time, the combination of hypokale- an improved method for estimating urine ammonium excretion (47).
mia and metabolic acidosis markedly increases ammonium A UOG over 400 mOsm/kg is suggestive of high urinary ammonium
excretion, culminating in hypokalemic hyperchloremic metabolic excretion and therefore indicative of GI loss. Nevertheless, the reliabil-
acidosis with an elevated urine pH—a pattern often mistaken for ity of UOG in estimating urinary ammonium is also questioned
distal renal tubular acidosis (RTA) (41). However, a very high urine by some recent studies (48). Therefore, both UAG and UOG have
ammonium level effectively rules out distal RTA. limitations in estimation of urinary ammonium excretion, and
direct urinary ammonium measurement should be obtained when fea-
sible (49).
4 Causes of high anion gap metabolic acidosis
include inborn errors of metabolism, lactic acido- Gastrointestinal Loss
sis, ketoacidosis, renal failure, and the ingestion Diarrhea is by far the most common cause of hyperchloremic meta-
of drugs and toxins. bolic acidosis in clinical settings. The pathophysiology involves not
only the loss of bicarbonate in the stool, which is rather modest, but
also the loss of salts from organic acid anions like lactate, acetate,
butyrate, and citrate, which represent potential sources of bicarbon-
Hyperchloremic Metabolic Acidosis ate and are physiologically equivalent (50). In severe cases of
The pathogenesis of hyperchloremic metabolic acidosis is usually due diarrhea that lead to marked hypovolemia, patients may have con-
to bicarbonate loss or reduced renal acid excretion, resulting in a nor- current HAGMA due to lactic acidosis, acute kidney injury, and
mal serum anion gap (AG). Additionally, dilution acidosis is a unique hyperalbuminemia from hemoconcentration. The loss of pancreatic,
form of hyperchloremic metabolic acidosis caused by rapid infusion biliary, or intestinal secretions through drainage or fistulas can pre-
of a large volume of normal saline. As noted previously, hyperchlore- sent similarly to diarrhea-induced metabolic acidosis (50).
mic metabolic acidosis can overlap with high AG metabolic acidosis Another cause of hyperchloremic metabolic acidosis that can be
(HAGMA) in certain conditions, such as D-lactic acidosis, ketoacido- grouped under GI loss is urinary diversion, a surgical procedure
sis, CKD progression, and toluene ingestion. Typically, the remaining redirecting urine flow away from the bladder, often using a segment
causes of hyperchloremic metabolic acidosis are grouped into two of the GI tract. These procedures can lead to hyperchloremic meta-
main categories on the basis of the primary source of the issue: gastro- bolic acidosis through an anion exchanger that absorbs chloride
intestinal (GI) loss and renal tubular acidosis (RTA). in exchange for HCO3, and the metabolism of urinary urea by intes-
tinal bacteria and the absorption of ammonium in the colon.
Urine Anion Gap and Urine Osmolal Gap Although ureterosigmoidostomy (a procedure rarely performed
The differential diagnosis between GI loss and RTA is often clear today) had a high rate of causing acidosis (up to 80% of patients),
from the patient history, serum chemistry, and urine pH. However, ileal, jejunal, or colonic conduits are less likely to cause metabolic
patients with persistent metabolic acidosis from GI causes can acidosis (51,52). However, metabolic acidosis can still occur, espe-
exhibit a urine pH above 5.5, like some patients with RTA. This is cially if the contact time between urine and intestine is prolonged,
due to increased urinary ammonium excretion as a response to pro- such as in cases of stomal stenosis.
longed metabolic acidosis as discussed earlier. In cases of hyper-
chloremic metabolic acidosis with urine pH above 5.5, assessing Renal Tubular Acidosis
urine ammonium excretion helps distinguish between GI loss and RTA refers to a group of disorders where hyperchloremic metabolic
RTA: higher levels usually indicate GI loss of HCO3, whereas lower acidosis occurs because of defects in renal tubular HCO3 reabsorp-
levels suggest RTA (42). Because of the limited availability of urine tion or acid excretion. RTA is classified into three main types on the
ammonium excretion tests in most clinical laboratories, calculations basis of their primary defects (Table 1): distal RTA (type 1, defects
of urine AG (UAG) and urine osmolal gap (UOG) serve as surro- in distal hydrogen ion secretion), proximal RTA (type 2, impaired
gates (43). proximal tubular HCO3 reabsorption), and hyperkalemic RTA
134 Nephrology Self-Assessment Program - Vol 23, No 2, June 2024
Primary defect Impaired distal hydrogen ion Reduced proximal HCO3 Hypoaldosteronism
secretion reabsorption
Common acquired causes Autoimmune disorder (Sjogren, Light chain toxicity, Diabetic kidney disease,
RA), medications (lithium, medications (acetazolamide, medications (NSAIDs,
ifosfamide, and amphotericin) topiramate) K-sparing diuretics, and ACE
inhibitors)
Related conditions Nephrolithiasis or Fanconi syndrome NA
nephrocalcinosis
Serum K level Low Low High
Serum HCO3 level Variable; may be below Usually 12–20 mEq/L Usually .16 mEq/L
10 mEq/L
Urine pH Always .5.3 Variable Variable; usually .5.3
Urinary ammonium excretion Low Variable Low
Treatment Alkali therapy (sodium Alkali therapy (usually K binders, fludrocortisone, loop
bicarbonate, sodium citrate, potassium citrate); thiazide diuretics, alkali therapy
potassium citrate) diuretics (sodium bicarbonate)
ACE, angiotensin-converting enzyme; HCO3, bicarbonate; K, potassium; NSAIDs, nonsteroidal anti-inflammatory drugs; RA, rheumatoid arthritis.
(type 4, hypoaldosteronism) (53–55). Distal and proximal RTAs are be administered to correct metabolic acidosis and mitigate hyperka-
relatively less common compared to type 4 RTA. Type 3 RTA, lemia risks. In certain cases, low-dose fludrocortisone and loop
which is a rare combination of distal and proximal features, will not diuretics may also be beneficial.
be discussed here.
Distal (Type 1) Renal Tubular Acidosis. The primary defect in
Hyperkalemic (Type 4) Renal Tubular Acidosis. Type 4 is the distal RTA is impaired hydrogen ion secretion in the distal
most common form of RTA and is characterized by its associated nephrons, due to either decreased net activity of the proton pump
hyperkalemia. It typically results from hypoaldosteronism, due to or increased luminal membrane hydrogen ion permeability. Distal
either aldosterone deficiency or aldosterone resistance. The acidifica- RTA is often associated with profound hypokalemia due to renal
tion defect is thought to be primarily caused by a reduced rate of potassium wasting. Common causes in adults include autoimmune
proton secretion, and hyperkalemia further exacerbates the condition diseases such as Sjogren disease and rheumatoid arthritis, and drugs
by inhibiting ammoniagenesis and thus decreasing urine ammonium such as lithium, ifosfamide, and amphotericin. In contrast, heredi-
excretion (53). Hypoaldosteronism can have various causes, includ- tary causes of distal RTA are more common in children, such as
ing diabetes, interstitial nephritis, genetic conditions like Gordon mutations in the gene SLC4A1 that codes for chloride-bicarbonate
syndrome, and certain medications such as nonsteroidal anti- exchanger located on the basolateral cell membrane in the distal
inflammatory drugs, K-sparing diuretics, and angiotensin-converting tubule (57). Patients with distal RTA are prone to forming calcium-
enzyme inhibitors. Diabetic kidney disease is often the most fre- containing nephrolithiasis and nephrocalcinosis due to a combina-
quently seen cause of type 4 RTA in clinical settings, arising from tion of hypercalciuria, hypocitraturia, and alkaline urine. In
low plasma renin activity, leading to hyporeninemic hypoaldosteron- children, distal RTA can lead to failure to thrive, growth delay, and
ism (56). Notably, there is also voltage-dependent distal RTA that rickets. The diagnosis of distal RTA requires measurement of the
can present with hyperkalemic RTA, caused by defects in sodium urine pH and estimation of urinary ammonium excretion. Urine
reabsorption in the distal nephron. Some examples of voltage- pH above 5.5 regardless of serum HCO3 level is characteristic of
dependent RTA include severe hypovolemia and urinary tract distal RTA. Reduced urinary ammonium excretion, either by direct
obstruction. In type 4 RTA, checking renin and aldosterone levels measurement or through estimation with UAG or UOG, helps dif-
can help diagnose hyporeninemic hypoaldosteronism. Generally, in ferentiate distal RTA from GI loss. Serum HCO3 can be signifi-
type 4 RTA, urine pH can be reduced below 5.5, and the metabolic cantly reduced and even fall below 10 mEq/L (53). Alkali therapy is
acidosis is typically mild, with serum HCO3 usually over 16 mEq/L. required to treat distal RTA, with the goal of achieving a normal
Treatment is tailored to the underlying cause, with discontinuation serum HCO3 concentration. Alkali therapy is essential for mitigat-
of offending medications if they are nonessential. Managing ing complications such as urinary K wasting, osteopenia, nephro-
serum K levels through dietary modifications or novel potassium lithiasis, and CKD progression in adults, and to restore normal
binders (e.g., patiromer and sodium zirconium cyclosilicate) can growth rate in children. Various forms of alkali therapy can be used,
correct acidosis by enhancing ammoniagenesis and subsequent such as sodium bicarbonate, sodium citrate, and potassium cit-
buffer supply for distal acidification (53). Sodium bicarbonate may rate (53).
Nephrology Self-Assessment Program - Vol 23, No 2, June 2024 135
Proximal (Type 2) Renal Tubular Acidosis. The primary defect 10. Peitzman SJ: The flame photometer as engine of nephrology: A biogra-
in proximal RTA is the reduced capacity to reabsorb filtered HCO3 phy. Am J Kidney Dis 56: 379–386, 2010 PubMed
11. Feldman M, Soni N, Dickson B: Influence of hypoalbuminemia or
in the proximal tubule. Proximal RTA occurs rarely as an isolated
hyperalbuminemia on the serum anion gap. J Lab Clin Med 146: 317–320,
defect and is more commonly associated with the generalized proxi- 2005 PubMed
mal tubule dysfunction known as Fanconi syndrome, with 12. Rastegar A: Use of the DeltaAG/DeltaHCO32 ratio in the diagnosis of
additional features such as hypophosphatemia, renal glucosuria, mixed acid-base disorders. J Am Soc Nephrol 18: 2429–2431, 2007
hypouricemia, and aminoaciduria. Like distal RTA, proximal RTA PubMed
is often associated with hypokalemia due to renal potassium wasting. 13. Mehta AN, Emmett JB, Emmett M: GOLD MARK: An anion gap mne-
Common causes of proximal RTA in adults include light chain tox- monic for the 21st century. Lancet 372: 892, 2008 PubMed
14. Azim A, Hu B, Gilligan S, et al.: How I evaluate a high anion gap meta-
icity in the setting of multiple myeloma or other plasma cell dyscra-
bolic acidosis. Clin J Am Soc Nephrol 19: 525–527, 2024 PubMed
sias, and medications like carbonic anhydrase inhibitors such as 15. Fenves AZ, Mojtahed A, Nisavic M, Massoth LR: Case 13-2019: A
acetazolamide and topiramate. Hereditary causes of proximal RTA 54-year-old man with alcohol withdrawal and altered mental status.
such as cystinosis should also be considered, especially in children. N Engl J Med 380: 1657–1665, 2019 PubMed
In proximal RTA, urine pH varies on the basis of the filtered 16. Schillaci LP, DeBrosse SD, McCandless SE: Inborn errors of metabolism
HCO3 load. In some cases, proximal RTA can be diagnosed by with acidosis: Organic acidemias and defects of pyruvate and ketone
increasing serum bicarbonate concentration toward normal with an body metabolism. Pediatr Clin North Am 65: 209–230, 2018 PubMed
17. Solares I, Heredia-Mena C, Castelbon FJ, Jerico D, Cordoba KM,
intravenous bicarbonate infusion, which will rapidly increase urine
Fontanellas A, et al.: Diagnosis and management of inborn errors of
pH in proximal RTA once the bicarbonate reabsorption threshold is metabolism in adult patients in the emergency department. Diagnostics
exceeded (58). Treatment depends on the specific cause and typi- (Basel) 11: 2148, 2021 PubMed
cally involves stopping any responsible medications. Correcting met- 18. Fukao T, Nakamura K: Advances in inborn errors of metabolism. J Hum
abolic acidosis in proximal RTA is more challenging and requires Genet 64: 65, 2019 10.1038/s10038-018-0535-7 PubMed
more alkali therapy compared with distal RTA, because of signifi- 19. Kreisberg RA: Lactate homeostasis and lactic acidosis. Ann Intern Med
cant bicarbonate diuresis when attempting to raise serum HCO3 92: 227–237, 1980 PubMed
20. Kraut JA, Madias NE: Lactic acidosis. N Engl J Med 371: 2309–2319,
levels. Potassium citrate is often the preferred alkali therapy. Low-
2014 PubMed
dose thiazide diuretics can be used to enhance HCO3 reabsorption, 21. Kraut JA, Madias NE: Treatment of acute metabolic acidosis: A patho-
prevent volume expansion, and boost the efficacy of alkali ther- physiologic approach. Nat Rev Nephrol 8: 589–601, 2012 PubMed
apy (53). 22. Yeku OO, Medford AJ, Fenves AZ, Uljon SN: Case 15-2021: A
76-year-old woman with nausea, diarrhea, and acute kidney failure.
N Engl J Med 384: 1943–1950, 2021 PubMed
Conclusion 23. Bianchetti DGAM, Amelio GS, Lava SAG, Bianchetti MG, Simonetti
Metabolic acidosis is defined by an increased concentration of GD, Agostoni C, et al.: D-lactic acidosis in humans: Systematic literature
review. Pediatr Nephrol 33: 673–681, 2018 PubMed
hydrogen ions and a decreased concentration of HCO3 in the body.
24. Reichard GA Jr., Owen OE, Haff AC, Paul P, Bortz WM: Ketone-body
The serum anion gap (AG) and d AG/d HCO3 ratio may be used production and oxidation in fasting obese humans. J Clin Invest 53:
to identify the etiology of metabolic acidosis. Identifying the cause 508–515, 1974 PubMed
of high AG metabolic acidosis (HAGMA) and hyperchloremic met- 25. Al Alawi AM, Al Flaiti A, Falhammar H: Lactation ketoacidosis: A system-
abolic acidosis informs management. atic review of case reports. Medicina (Kaunas) 56: 299, 2020 PubMed
26. Kitabchi AE, Umpierrez GE, Miles JM, Fisher JN: Hyperglycemic crises
in adult patients with diabetes. Diabetes Care 32: 1335–1343, 2009
References PubMed
1. Fenves AZ, Emmett M: Approach to patients with high anion gap meta- 27. Palmer BF, Clegg DJ: Euglycemic ketoacidosis as a complication of
bolic acidosis: Core curriculum 2021. Am J Kidney Dis 78: 590–600, SGLT2 inhibitor therapy. Clin J Am Soc Nephrol 16: 1284–1291, 2021
2021 PubMed PubMed
2. Kraut JA, Madias NE: Serum anion gap: Its uses and limitations in clini- 28. Adrogue HJ, Wilson H, Boyd AE 3rd, Suki WN, Eknoyan G: Plasma
cal medicine. Clin J Am Soc Nephrol 2: 162–174, 2007 PubMed acid-base patterns in diabetic ketoacidosis. N Engl J Med 307: 1603–
3. Pierce NF, Fedson DS, Brigham KL, Mitra RC, Sack RB, Mondal A: 1610, 1982 PubMed
The ventilatory response to acute base deficit in humans. Ann Intern Med 29. Long B, Lentz S, Gottlieb M: Alcoholic ketoacidosis: Etiologies, evalua-
72: 633–640, 1970 PubMed tion, and management. J Emerg Med 61: 658–665, 2021 PubMed
4. Albert MS, Dell RB, Winters RW: Quantitative displacement of acid- 30. Raphael KL: Metabolic acidosis in CKD: Core curriculum 2019. Am J
base equilibrium in metabolic acidosis. Ann Intern Med 66: 312–322, Kidney Dis 74: 263–275, 2019 PubMed
1967 PubMed 31. Pham AQ, Xu LH, Moe OW: Drug-induced metabolic acidosis [pub-
5. Fulop M: A guide for predicting arterial CO2 tension in metabolic acido- lished online ahead of print Dec 16, 2015]. F1000 Res doi:10.12688/
sis. Am J Nephrol 17: 421–424, 1997 PubMed f1000research.7006.1 PubMed
6. Hamm LL, Nakhoul N, Hering-Smith KS: Acid-base homeostasis. Clin J 32. Palmer BF, Clegg DJ: Salicylate toxicity. N Engl J Med 382: 2544–2555,
Am Soc Nephrol 10: 2232–2242, 2015 PubMed 2020 PubMed
7. Weiner ID, Hamm LL: Molecular mechanisms of renal ammonia trans- 33. Fenves AZ, Kirkpatrick HMI 3rd, Patel VV, Sweetman L, Emmett M:
port. Annu Rev Physiol 69: 317–340, 2007 PubMed Increased anion gap metabolic acidosis as a result of 5-oxoproline
8. Garibotto G, Sofia A, Robaudo C, Saffioti S, Sala MR, Verzola D, et al.: (pyroglutamic acid): A role for acetaminophen. Clin J Am Soc Nephrol 1:
Kidney protein dynamics and ammoniagenesis in humans with chronic 441–447, 2006 PubMed
metabolic acidosis. J Am Soc Nephrol 15: 1606–1615, 2004 PubMed 34. Emmett M: Acetaminophen toxicity and 5-oxoproline (pyroglutamic
9. Harvey AM: Classics in clinical science: James L. Gamble and “Gamblegrams.” acid): A tale of two cycles, one an ATP-depleting futile cycle and the
Am J Med 66: 904–906, 1979 PubMed other a useful cycle. Clin J Am Soc Nephrol 9: 191–200, 2014 PubMed
136 Nephrology Self-Assessment Program - Vol 23, No 2, June 2024
35. Hundemer GL, Fenves AZ: Acquired 5-oxoproline acidemia successfully 47. Dyck RF, Asthana S, Kalra J, West ML, Massey KL: A modification of
treated with N-acetylcysteine. Proc Bayl Univ Med Cent 30: 169–170, the urine osmolal gap: An improved method for estimating urine ammo-
2017 PubMed nium. Am J Nephrol 10: 359–362, 1990 PubMed
36. Nigwekar SU, Thadhani R, Brandenburg VM: Calciphylaxis. N Engl J 48. Raphael KL, Ix JH: Correlation of urine ammonium and urine osmolal
Med 378: 1704–1714, 2018 PubMed gap in kidney transplant recipients. Clin J Am Soc Nephrol 13: 638–640,
37. Hundemer GL, Fenves AZ, Phillips KM, Emmett M: Sodium thiosulfate 2018 PubMed
and the anion gap in patients treated by hemodialysis. Am J Kidney Dis 49. Uribarri J, Goldfarb DS, Raphael KL, Rein JL, Asplin JR: Beyond the
68: 499–500, 2016 PubMed urine anion gap: In support of the direct measurement of urinary ammo-
38. Farese S, Stauffer E, Kalicki R, Hildebrandt T, Frey BM, Frey FJ, et al.: nium. Am J Kidney Dis 80: 667–676, 2022 PubMed
Sodium thiosulfate pharmacokinetics in hemodialysis patients and 50. Gennari FJ, Weise WJ: Acid-base disturbances in gastrointestinal disease.
healthy volunteers. Clin J Am Soc Nephrol 6: 1447–1455, 2011 PubMed Clin J Am Soc Nephrol 3: 1861–1868, 2008 PubMed
39. Kraut JA, Mullins ME: Toxic alcohols. N Engl J Med 378: 270–280, 51. Cruz DN, Huot SJ: Metabolic complications of urinary diversions: An
2018 PubMed overview. Am J Med 102: 477–484, 1997 PubMed
40. Kraut JA, Xing SX: Approach to the evaluation of a patient with an 52. Tanrikut C, McDougal WS: Acid-base and electrolyte disorders after uri-
increased serum osmolal gap and high-anion-gap metabolic acidosis. Am nary diversion. World J Urol 22: 168–171, 2004 PubMed
J Kidney Dis 58: 480–484, 2011 PubMed 53. Palmer BF, Kelepouris E, Clegg DJ: Renal tubular acidosis and manage-
41. Carlisle EJ, Donnelly SM, Vasuvattakul S, Kamel KS, Tobe S, Halperin ment strategies: A narrative review. Adv Ther 38: 949–968, 2021
ML: Glue-sniffing and distal renal tubular acidosis: Sticking to the facts. PubMed
J Am Soc Nephrol 1: 1019–1027, 1991 PubMed 54. Soleimani M, Rastegar A: Pathophysiology of renal tubular acidosis:
42. Rastegar M, Nagami GT: Non-anion gap metabolic acidosis: A clinical Core curriculum 2016. Am J Kidney Dis 68: 488–498, 2016 PubMed
approach to evaluation. Am J Kidney Dis 69: 296–301, 2017 PubMed 55. Rodrıguez Soriano J: Renal tubular acidosis: The clinical entity. J Am Soc
43. Palmer BF, Clegg DJ: The use of selected urine chemistries in the diagnosis Nephrol 13: 2160–2170, 2002 PubMed
of kidney disorders. Clin J Am Soc Nephrol 14: 306–316, 2019 PubMed 56. Lush DJ, King JA, Fray JC: Pathophysiology of low renin syndromes:
44. Goldstein MB, Bear R, Richardson RMA, Marsden PA, Halperin ML: Sites of renal renin secretory impairment and prorenin overexpression.
The urine anion gap: A clinically useful index of ammonium excretion. Kidney Int 43: 983–999, 1993 PubMed
Am J Med Sci 292: 198–202, 1986 PubMed 57. Wagner CA, Unwin R, Lopez-Garcia SC, Kleta R, Bockenhauer D,
45. Batlle DC, Hizon M, Cohen E, Gutterman C, Gupta R: The use of the Walsh S: The pathophysiology of distal renal tubular acidosis. Nat Rev
urinary anion gap in the diagnosis of hyperchloremic metabolic acidosis. Nephrol 19: 384–400, 2023 PubMed
N Engl J Med 318: 594–599, 1988 PubMed 58. Rose BD, Post TW: Renal tubular acidosis. In: Clinical Physiology of
46. Uribarri J, Oh MS: The urine anion gap: Common misconceptions. Acid-Base and Electrolyte Disorders, 5th Ed., New York, McGraw-Hill,
J Am Soc Nephrol 32: 1025–1028, 2021 PubMed 2001, pp 612–625
Nephrology Self-Assessment Program - Vol 23, No 2, June 2024 137
Article
Metabolic Alkalosis
Anand Achanti, MD, and Waleed A. Elsheikh, MD, MBBS
Medical University of South Carolina, Charleston, South Carolina
per kg body weight per day, that is, 70–100 mEq per day, to replace
Learning Objectives what is consumed in buffering. In addition, net acid excretion (uri-
nary ammonium and titratable acid) reflects the daily formation of
1. To describe the underlying physiology of bicarbonate handling
endogenous acid (6,7,9).
within the kidney
2. To identify the alterations to normal physiology that result in Proximal Tubule
generation and maintenance phases of metabolic alkalosis Most (80%–85%) of the filtered HCO32, about 4000 mEq per
day, is reabsorbed in the proximal tubule through an acid-base
3. To evaluate patients with metabolic alkalosis and prescribe exchange facilitated by the apical Na1/H1 Exchanger 3 (NHE3)
targeted therapies based on the determined underlying and the vacuolar H1 Adenosine Triphosphatase (H1-ATPase)
etiology transporter. H1 secreted into the lumen reacts with HCO32 form-
ing carbonic acid (H2CO3), which rapidly undergoes dehydration
catalyzed by carbonic anhydrase IV (CAIV) to dissociate into H2O
Introduction and CO2. The cell membrane is permeable to CO2, and bulk water
Metabolic alkalosis is the most frequently encountered acid-base dis- transport occurs through aquaporin1 (AQP1). Carbonic anhydrase
order in hospitalized patients, with the greatest incidence in critically II (CAII) converts H2O and CO2 to H1 and HCO32 within the
ill patients and during postoperative recovery (1–3). Failed compen- proximal tubular cell cytoplasm. The formed HCO32 enters blood
satory homeostasis and progressive alkalemia (pH.7.44) are associ- through the basolateral electrogenic Na1/HCO32 Cotransporter 1
ated with increased mortality (4–6). To determine the cause of (NBCe1), while the H1 ion is secreted back into lumen through
metabolic alkalosis and administer effective therapy, it is imperative NHE3 to repeat the process of HCO32 reabsorption (6,9–11); see
to recognize those physiologic mechanisms involved in the interac- Figure 1 (ref. 10, figure 1).
tion between the kidneys and lungs that are involved in maintaining Decreased H1 ion concentration in the setting of metabolic
blood pH in the range of 7.36–7.44 and intracellular pH in the alkalosis would be expected to decrease the ability of the proximal
range of 7.0–7.3 (7,8). Bicarbonate (HCO32) excess manifesting as tubule to absorb more HCO32 because of reliance on H1 ions for
metabolic alkalosis (arterial HCO3.28 mmol/L and venous total the reabsorption of HCO32. However, a decrease in GFR from
CO2.30 mmol/L) results from a net reduction in plasma H1 ions conditions like volume depletion results in reduced HCO32 filtra-
(H1) or gain of base. Conversely, it may be viewed as a cation- tion and, consequently, a waning of the efficacy of this frontline
anion imbalance brought on by a state of relative sodium excess or defense against alkalosis. Furthermore, physiologic response to vol-
chloride depletion (7,8). ume depletion enhances sodium reabsorption within the proximal
In this review, we discuss the factors involved in generation of tubule, which increases HCO32 reabsorption as compared with
metabolic alkalosis (Table 1) and counterregulatory mechanisms chloride in the proximal tubule. Volume restoration would be
that maintain pH neutrality and prevent bicarbonate excess. The required to regain normal GFR and increase HCO32 delivery to the
kidneys, through their various nephron segments, discussed in the distal nephron for elimination (12).
next section, are well equipped to accommodate and correct a tran-
sient alkaline load under normal conditions. Electrolyte deficits, Distal Nephron
genetic mutations, or iatrogenic factors that may tip the balance in Most of the rest (10%–15%) of the filtered HCO32 is reabsorbed
favor of alkalinity are discussed and explained further in the genera- in the thick ascending limb (TAL) in a similar manner to the proxi-
tion, maintenance and evaluation of metabolic alkalosis sections. mal tubule, by means of NHE3 and H1-ATPases transporters. The
remaining 5% of HCO32 handling is by the distal nephron. a- and
b-intercalated cells (ICC) and principal cells in the collecting duct
Bicarbonate Regulation and Handling by the Kidney (CD) have a regulatory fine-tuning role in acid-base homeostasis.
The kidneys help regulate and maintain the concentration of The principal cells secrete potassium and reabsorb sodium and water
HCO32 in the body through two main mechanisms: control of losses under the influence of aldosterone and vasopressin. a-ICC secretes
and regeneration. HCO32 is freely filtered by the glomeruli, and its H1 via apical H1-ATPases and H1/K1-ATPases, and, in turn,
reabsorption predominantly occurs in the proximal tubule, but fine- HCO-2 is generated by the action of CAII. HCO32 is then deliv-
tuning of final HCO32 excretion occurs in the distal tubule (6). The ered to the blood through Cl2/HCO32 anion exchange protein1
kidney produces new HCO32 by regenerating approximately 1 mEq (AE1). Increased sodium reabsorption and subsequent potassium
Table 1. Generation and maintenance of metabolic alkalosis, urinary findings, and response to chloride
Response to Chloride
Cause Generation Phase Maintenance Phase Urinary Finding Intake
Table 1 illustrates a variety of causes of metabolic alkalosis. It highlights the generation and maintenance phase associated with each process and expected urinary findings,
followed by expected chloride responsiveness to the associated condition. ECF, extracellular fluid.
Nephrology Self-Assessment Program - Vol 23, No 2, June 2024 139
Figure 1. Schematic depiction of the localization and role of acid-base transporters and carbonic anhydrases in acid (H1) and bicarbonate (HCO3)
transport in kidney tubules. The majority of filtered HCO3 is reabsorbed in the proximal tubule via the coordinated actions of NHE3 (encoded by
SLC9A3) and H1-ATPase on the apical membrane working in tandem with the NBCe1 (encoded by SLC4A4) on the basolateral membrane.
This process is facilitated by the actions of CAIV and CAII on the apical membrane and in the cytoplasm, respectively. In the CCD, H1-ATPase
and H1/K1-ATPase on the apical membrane of a-ICC mediate H1 secretion into the lumen, resulting in the generation of intracellular HCO3
that is transported to the blood via the basolateral Cl2/HCO32AE1 (encoded by SLC4A1). The Cl2/HCO3 exchanger pendrin (encoded by
SLC26A4) on the apical membrane of non–a-ICC mediates HCO3 secretion into the lumen of the CCD in exchange for Cl2 absorption. Sodium
and H2O are absorbed by the ENaC and AQP2, respectively, in principal cells. ADP, adenosine diphosphate; AE1, anion exchange protein 1;
AQP2, aquaporin 2; ATP, adenosine triphosphate; CAII, carbonic anhydrase; CCD, cortical CD; DCT, distal convoluted tubule; ENaC, epithelial
sodium channel; NBCe1, basolateral Na1/HCO32 cotransporter; NHE3, Na1/H1 exchanger 3; NKCC,Na1/K1/2Cl2 cotransporter; PCT,
proximal convoluted tubule; Pi, inorganic phosphate; TALH, thick ascending limb of Henle. Reprinted from ref. 10, with permission.
secretion from principal cells promotes H1 secretion by a-ICC. angiotensin system; however, the actual activity of pendrin is
b-ICC do the opposite and secrete two HCO32 ions in exchange impaired because of the lack of chloride in the tubular lumen. In
for two Cl2 ions through the apical Cl2/HCO32 exchanger (pen- addition to aldosterone, Pendrin can increase the transcription of
drin [SLC26A4)]. The resulting intracellular H1 ions generated ENaC and abundance, which results in increased sodium reabsorp-
from the catalyzation of H2CO3 are moved into systemic circulation tion but, paradoxically, an increased HCO32 reabsorption through
via a basolateral H1-ATPase transporter to further reduce systemic the Na1/HCO32 exchanger 1 on basolateral surface of principal
HCO32 concentration (6,7,13); see Figure 1 (ref. 10, figure 1). cells (NDCBE; SLC4A8) (17). Volume depletion enhances sodium
and chloride reabsorption but impairs the ability to excrete HCO32
that also coincides with a decrease in filtered HCO32 due to a
Pendrin Regulation diminished GFR, resulting in metabolic alkalosis. It is therefore crucial
Pendrin is upregulated in metabolic alkalosis in general but can be in metabolic alkalosis to maintain and increase chloride delivery to the
regulated by other factors such as low chloride distal delivery in b-ICC in order to upregulate pendrin and enable the Cl2/HCO32
states like NaCl restriction without the presence of metabolic alkalo- exchange. Chloride-depleting states like HCl loss through nasogastric
sis (14–16). Volume depletion leads to an upregulation of pendrin suctioning will offset this compensatory mechanism and not allow the
due to decreased distal chloride delivery from activation of renin exchange through pendrin, leading to HCO32 retention.
140 Nephrology Self-Assessment Program - Vol 23, No 2, June 2024
The correlation, however, is inverse in hyperkalemia where pen- functionally interact with SLC26A6 and SLC26A3 through this sul-
drin is downregulated despite aldosterone activation. This results fate transporter and anti-sigma factor antagonist domain, so it may
from an increase in intracellular chloride within the distal convo- have an interaction with pendrin, which is also a SLC26 transporter
luted tubule and CD resulting in an inhibition of both the Na-Cl (22,23). Interestingly, treatment of three CF patients who had simi-
cotransporter and pendrin (15). In other words, pendrin can be dif- lar inappropriate responses to alkaline loading with CFTR modula-
ferently regulated in high aldosterone states of different etiologies. A tors increased bicarbonaturia (24). It remains to be confirmed
recent paper suggests that it is more the anion attached to potassium whether CFTR can directly secrete or provide permeability flow to
than a high-potassium diet that regulates pendrin. In a mouse HCO3 in alkalinity (23). A recent Italian retrospective study of CF
model, low-potassium diet, high KCl, and high-potassium HCO32 patients with pseudo-Bartter syndrome as the initial presentation
(KHCO32) were compared. There was no difference in pendrin reported a lower incidence of pancreatic insufficiency and pseudo-
expression between 1% potassium chloride (KCl) diet versus 5% monal infections compared with their counterparts over a mean
KCl diet. However, when comparing 5% KHCO32 to control or follow-up of 11 years, and a milder pulmonary phenotype, with
high-KCl diet, there was an increased expression of pendrin along more than half exhibiting no pathologic CT chest findings. The
with an increase in serum HCO32 and potassium levels. Increased authors suggest CF evaluation for younger patients who present
aldosterone levels were seen in both the KCl and KHCO32 diets, with Bartter syndrome and have negative neonatal screening tests for
which suggests aldosterone level itself was not enough to upregulate Bartter mutations, because milder phenotypes of CF may be missed
transcription of pendrin but requires other stimuli such as alkalosis and therefore remain a possibility (20). A small pilot study in the
or volume depletion. This was further verified when mice were United Kingdom on CF patients showed no correlation between
switched from KHCO32 to KCl, causing a rapid reversal from met- urinary HCO32 excretion and CF exacerbation (24).
abolic alkalosis to hyperchloremic metabolic acidosis and a decrease Another mechanism that influences renal HCO32 handling
in pendrin transcription and vice versa from switching from high was recently identified. Secretin is an intestinal hormone known to
KCl to KHCO32 (15). cause increased epithelial HCO32 secretion in the pancreas and
Pendred syndrome results from a mutation in this transporter, hepatic ducts. A SCTR is present in the CDs primarily in the
which prevents the excretion of HCO32 in the setting of alkalemia, b-ICC and, to a lesser extent, a-ICC but not in the principal cells
resulting in a more severe metabolic alkalosis in the face of an alkali (25). Secretin levels increase in the setting of acute metabolic alkalo-
load (18). In addition to pendrin mutations, there have recently sis. In a mouse model that was loaded with alkali, urinary and sys-
been described mutations in cystic fibrosis transmembrane conduc- temic responses were determined along with pendrin activity, which
tance regulator (CFTR) and secretin receptor (SCTR) that can also was directly measured in isolated perfused CDs. As in CFTR KO
potentially prolong and exacerbate alkalosis by their interactions animals, secretin did not play a role in acid-base balance in the
with pendrin. absence of alkali loading, with normal b-ICC function and normal
CFTR mutation was originally identified in patients with cystic localization, function, and abundance of pendrin. However, upon
fibrosis (CF), a cohort with propensity to metabolic alkalosis where oral alkali loading, whereas wild-type mice changed from net acid
urinary chloride can be both elevated and low. Previous CF studies excretion to net base excretion, in SCTR knockout mice, there was
showed a markedly decreased capacity to excrete base equivalents no change in net base excretion with continued positive net acid
secondary to an altered or failed renal HCO32 handling response to excretion and accumulation of serum HCO32. Similar to previous
alkaline stimuli. CF patients are also prone to exhibit a pseudo- studies in CFTR and pendrin KO mice, pH tended not to vary dur-
Bartter phenotype due to chloride loss in sweat and the upregulation ing acute base loading, because of the fact of respiratory compensa-
of aldosterone (19). Pseudo-Bartter clinically manifests as hypokale- tion with a decreased minute ventilation. Similar findings were
mia, hypochloremia, hyponatremia, and metabolic alkalosis and is found whether alkali loading was given intravenously (iv) or orally,
estimated to be the initial presentation of CF in about 1%–2% of suggesting that this is not a transport issue. Loss of SCTR resulted
cases in countries with robust perinatal detection programs (20). in an inability to respond to acute metabolic alkalosis, causing a pro-
Berg et al. alluded to a potential CFTR-pendrin interaction, because longed and aggravated alkalosis (26). Berg et al. proposed that secre-
CFTR knockout (KO) mice subjected to alkali load were unable to tin influenced urinary HCO3 secretion is a response to the “alkaline
sufficiently elevate renal HCO32 excretion and developed metabolic tide” of digestion (26). The alkaline tide is caused by HCl secretion
alkalosis in contrast to wild-type mice, suggesting reduced pendrin of the gastric parietal cells in response to food bolus to form chyme
function, although pendrin expression was similar in both CFTR that enters duodenum and triggers secretin release from the S cells.
KO and wild-type animals who were not subjected to alkalosis (21). Secretin increases cAMP production when bound to SCTR located
The decreased pendrin function in alkalosis can be attributed to on the basolateral membrane of b-ICC, which activates pendrin
decreased trafficking of pendrin to the apical membrane in CFTR trafficking to apical membrane and initiates cAMP/protein kinase A
KO mice. Alternatively, some have proposed decreased functional activation of CFTR, which helps recycle chloride for luminal provi-
capacity of the pendrin channel despite insertion into luminal mem- sion required for the pendrin-mediated Cl2/HCO32 exchange.
brane due to the absence of CFTR, as they both colocalize to the (23,26); see Figure 2 (ref. 27, figure 1). It is difficult to determine
luminal membrane of the b-ICC. This has been demonstrated in yet whether secretin release in response to systemic metabolic alkalo-
other tissues, including cultured airway cells and parotid ducts cells. sis from causes other than gastrointestinal base loading originates
A final possibility was discussed in this paper in that CFTR is from the intestine as well versus the hypothalamus and neuroendo-
needed to anchor pendrin into the apical membrane (21). Pendrin crine cells (25). Although the pathophysiology of metabolic alkalosis
has a highly conserved cytoplasmic C-terminal sulfate transporter is still being evaluated, there appears to be both neurohumoral feed-
and anti-sigma factor antagonist domain. CFTR has been shown to back from the gut or brain and intrarenal monitoring that mediates
Nephrology Self-Assessment Program - Vol 23, No 2, June 2024 141
Figure 2. Proposed mechanism of secretin-induced HCO3 secretion in the pancreatic duct cell and in the b-ICC of the CD. Secretin activates
cAMP production via the basolateral G protein–coupled secretin receptor. Stimulation of the cAMP/protein kinase A pathway, in turn, pro-
motes CFTR and pendrin activity and hence an increased HCO3 secretion (Right). This mechanism of activated apical HCO3 secretion is very
similar to what is found in the duct cells of the exocrine pancreas (Left). CA, carbonic anhydrase, IC, Intercalated cells; STAS, sulfate transporter
and anti-s-factor antagonist. Reprinted from ref. 27, with permission.
the CD response to metabolic alkalosis. In summary, the rate-limiting metabolism via two pathways to produce glutamine or urea. The
step in urinary auto-alkalinization is orchestrated in the collecting urea-producing pathway utilizes equal amounts of NH41 and
tubule and is centered around pendrin. Although pendrin-mediated HCO32, whereas the glutamine pathway generates equal H1 for
base secretion depends on luminal chloride availability for HCO-2 ammonia, which requires HCO3 buffering. The net result of the
exchange, its abundance and adequate function seem to be influenced hepatic process is consumption of HCO32 produced during renal
by CFTR and SCTR interactions (27). ammoniagenesis, and homeostasis is thus maintained (10,28–30).
multiple mechanisms, including purging, resulting a loss of H1 ions Consequently, urinary chloride plays a large part in the diagnosis
from vomiting, chronic laxative abuse, and refeeding syndrome, and stratification of metabolic alkalosis for therapy and further
resulting in the conversion of ketoacids to HCO32. work-up.
Diuretic use demonstrates the variability of urinary chloride.
Decrease in Extracellular H1 Urinary chloride during diuretic use and immediately after with-
drawal can result in an elevated urinary chloride due to its effect on
H1 ions may be lost from the gastrointestinal (GI) tract or the kid-
sodium and chloride transport. In the absence of administration of
ney. The most common example is loss of H1 ions from vomiting
chloride-rich fluid, urinary chloride decreases after diuretics are
or nasogastric suction. The subsequent volume depletion results in
stopped, although the timing of the decrease is determined by the
increased release of aldosterone as a result of volume depletion.
half-life of the drug.
Increased distal sodium reabsorption triggered by increased sodium
delivery or aldosterone secretion in response to volume depletion
Hypokalemia
results in an enhanced potassium and H1 efflux (7,9). Testing urine
would reveal low sodium and chloride with evidence of potassium In metabolic alkalosis, there is a compensatory intracellular acidosis.
and HCO32 wasting (due to alkalosis response; an increased filtered In response, HCO32 absorption continues in a process that is
HCO32 load that overwhelms reabsorption capacity and the low enhanced by hypokalemia. Hypokalemia plays a part in the mainte-
luminal chloride available for Cl2-HCO32 exchange) and hence nance phase through several mechanisms. Hypokalemia causes a
a high urinary pH. Diarrhea resulting from a villous adenoma decrease in GFR for unknown reasons, resulting in decreased filtered
HCO32 (34). In the proximal tubule, hypokalemia stimulates
or certain hereditary forms of diarrhea or severe chronic diarrhea
ammoniagenesis, and, therefore, HCO32 generation will also increase.
from laxative abuse all resulting in fecal loss of H1 can also cause
Hypokalemia increases the expression of ammoniagenesis enzymes in
chloride loss.
the proximal tubule and stimulates the uptake of glutamine. Gluta-
Loss of chloride in sweat in cystic fibrosis that typically mani-
mine is used to form ammonia, and, consequently, HCO32 is gener-
fests in newborns results in increased H1 excretion within the kid-
ated (35,36). Increased ammonia within the tubular lumen also
ney. Intracellular H1 ion flux as seen in acute hypokalemia can
competes with potassium for reabsorption through the NKCC2 trans-
result in metabolic alkalosis.
porter within the thick ascending limb (TAL), which further enhances
kaliuresis. Hypokalemia triggers shifting of H1 ions intracellularly,
Metabolic Alkalosis: Maintenance Phase causing intracellular acidosis, and enhances HCO32 reabsorption (34).
Also, H1-K1-ATPase will increase secretion of H1 in favor of potas-
The kidneys can accommodate a transient increase in alkali load by
sium reabsorption in the hypokalemic state. Hypokalemia causes sec-
increasing the excretion fraction of filtered HCO32 as discussed
ondary hyperaldosteronism. Hypokalemia in aldosterone excess states
above and possibly through upregulation of pendrin, resulting in
has also been shown to cause downregulation of the Cl/HCO3
increased HCO32 excretion and chloride reabsorption (33). In the
exchanger pendrin, and therefore, in effect, reduces HCO32 secretion
acute phase, this is aided by the protein and phosphate buffer sys-
by the b-ICC (37) and maintains alkalosis.
tems while the ventilatory response is being mounted. However,
after alkaline load and HCO32 excess, impaired renal handling
would sustain the positive balance. This is true in the case of severely
4 Metabolic alkalosis is generated from a gain of
impaired GFR by virtue of function, potassium depletion through a
multitude of interconnected processes, which often include mineral- base/HCO3 or loss of acid/H1. The response to
ocorticoid excess that may be volume dependent or independent, alkalosis is increased HCO3 excretion in the kid-
and chloride depletion, which probably has the lion’s share. Renal neys and hypoventilation by the lungs, usually
handling and limiting steps were discussed anatomically above. In sufficient unless factors that maintain alkalosis
the next section, we discuss chloride and potassium depletion’s place
in continued alkalosis and why they should be prioritized in man- are present. Factors that maintain alkalosis
agement, and limitations in clinical context, which is further elabo- include volume depletion, chloride depletion
rated on in Management. through gastric or urinary losses, and hypokale-
mia. There can also be genetic mutations that
Chloride Depletion alter the bicarbonate handling within the kidney,
Metabolic alkalosis in the setting of volume depletion is caused by either directly through dysfunctional transporters
the loss of a chloride-rich fluid. The chloride deficit is more impor-
tant than the volume depletion per se in the “generation” and
(e.g., pendrin and CFTR) or indirectly through
“maintenance” of the base excess. It has been shown that the correc- stimulation by hormones.
tion of alkalosis can be achieved by addressing the chloride deficit,
even while maintaining sodium, potassium, and volume deficits,
and even in the presence of persistent alkali loading (12). This is
because the “maintenance” phase of metabolic alkalosis predomi- Management
nantly arises from reduced chloride delivery to the b-ICC, impairing Similarly to all acid-base disorders, management of metabolic alkalo-
the Cl2/HCO32 exchange on the luminal surface by pendrin. sis includes identifying the underlying cause and adjusting treatment
Nephrology Self-Assessment Program - Vol 23, No 2, June 2024 143
for that cause. Alkalemia from metabolic alkalosis tends to be less compensatory formulas to be revealed. There should be caution in
symptomatic than from respiratory alkalosis. This is highlighted by using just base excess of .2 to make a diagnosis of metabolic alkalo-
a case report of an infant with a pH of 7.95 who had an improve- sis, because base excess does not consider respiratory disorders, so an
ment in pH solely with the administration of normal saline (38). elevation of base excess can be either from primary metabolic alkalo-
However, interventions should be done to decrease serum pH to sis or compensation for a respiratory acidosis. It is important to
,7.55, because there is an increase in mortality and systemic com- make a correct diagnosis, because treatments for primary metabolic
plications for pH.7.55 (Table 2) (39). alkalosis can exacerbate acidosis in the latter.
After a diagnosis of metabolic alkalosis is confirmed, then a
comprehensive history is important. The history should include a
Evaluation of Metabolic Alkalosis search for any exogenous alkali exposure such as medications, citrate
Evaluation should start initially by proper evaluation of arterial exposure (massive transfusions, plasmapheresis, continuous renal
blood gas. See Figure 3 for a diagramatic synopsis of evaluation. An replacement therapy), effluent HCO32 in hemodialysis, or lactate
elevation of pH.7.45 and serum HCO32.27 provides a diagnosis exposure in peritoneal dialysis (PD). In particular, the history should
of metabolic alkalosis. However, underlying metabolic alkalosis can include inquiries regarding the presence of gastrointestinal (GI)
be hidden by other acid-base disorders and would need appropriate losses with vomiting, nasogastric (NG) suction, chronic diarrhea,
Figure 3. Evaluation of metabolic alkalosis, an algorithm to determine the underlying cause. AME, apparent mineralocorticoid excess; CAH,
congenital adrenal hyperplasia; GRA, glucocorticoid remediable hypertension; MC, mineralocorticoids; PAC, plasma aldosterone concentration;
PHA, primary hyperaldosteronism; PRA, plasma renin activity; TMA, thrombotic microangiopathy. ‡: CF patients may present with high or
low urinary chloride.
144 Nephrology Self-Assessment Program - Vol 23, No 2, June 2024
Metabolic alkalosis has deleterious effects on various organ systems, and, in intensive care unit settings, it adds another layer of complexity to ventilation, iv fluid, and
hemodynamic support strategy.
risk for refeeding syndrome, and any history of hypercalcemia from load but also, more importantly, a high chloride load. Volume
a variety of causes, including milk alkali syndrome, cystic fibrosis depletion should be addressed by repletion with a high-chloride
(CF), or any exposure to medications that can trigger an alkalosis solution. Adjustment or correction of GI losses, if feasible, should
such as diuretics, large molecular b-lactam antibiotics (sodium peni- be done, because prolonged chloride depletion and volume loss con-
cillin, carbenicillin), steroids, or chronic laxative use. tributes to both phases of metabolic alkalosis. Proton pump inhibi-
Volume analysis becomes crucial to better determine effective tors or H2 blockers should be administered to patients with
arterial circulating volume, because the majority of metabolic alkalosis aggressive nasogastric (NG) suction for prevention of metabolic alka-
found clinically tends to be related to volume. This entails physical losis. Diuretics should be held unless the patient is volume overloaded
exam with orthostatic blood pressures, and laboratory findings includ- (40). Volume expansion with a chloride-rich fluid such as normal
ing urinary chloride. Urinary chloride helps to narrow the differential saline should be administered this allows for increased distal chloride
diagnosis. An elevated urine chloride in the setting of volume deple- delivery, which is exchanged with HCO32 through pendrin and
tion suggests ongoing renal losses of volume, whether from diuretics, decreases secondary hyperaldosteronism. In addition, diminished
genetic conditions such as Bartter syndrome, or electrolyte abnormali- proximal sodium reabsorption also decreases proximal HCO32
ties (hypokalemia or hypomagnesemia) that cause ongoing natriuresis absorption.
even in the setting of volume depletion. Low urine chloride in the set- In countries with limited resources, oral rice broth (250 ml of
ting volume depletion is suggestive of ongoing nonrenal chloride rice in 1500 ml of water) was used for management of mild meta-
losses mainly being in the GI tract but can also be seen in neonates bolic alkalosis in both the outpatient and inpatient settings. The
with cystic fibrosis (CF) due to chloride loss in sweat. Such patients patient population had only a mild alkalosis, with the average pH
tend to have ongoing bicarbonaturia with an elevated urine pH. Non- before treatment being 7.45 (41). The solution was tested for com-
absorbable anions tend to increase distal delivery of sodium in place position, and was inherently acidic and contained a significant
of chloride, which results in hypokalemic metabolic alkalosis. amount of Mg21 and potassium, with an exact composition of pH
In patients with normal volume status and metabolic alkalosis, of 6.0, sodium 150 mmol per L, potassium 5.2 mmol per L,
the urine chloride is high. This is largely related to activation of the HCO32 0.005, Ca21 1.8 mmol per L, PO422 0.4 mmol per /L,
mineralocorticoid receptor and activation of ENaC. Activation of the Mg21 3.2 mmol per L, and glucose 0.06 g per L. Patients with pri-
receptor can be from aldosterone, impairment of 11 b-hydroxysteroid mary or apparent mineralocorticoid excess were excluded. Chloride-
dehydrogenase, impairment of the mineralocorticoid receptor, or depleted or volume-depleted patients were given 2–3 L of rice broth
increased presence of mineralocorticoid precursors, all resulting in per day to drink. There was an improvement in pH and serum
hypertension, hypokalemia, and metabolic alkalosis. The serum con- HCO32 levels.
centrations of aldosterone and plasma renin activity help distinguish
the underlying disorder (see Figure 3 for a summary).
Management of Volume Depletion with Elevated
Urinary Chloride
Management of Volume Depletion with Low Hypokalemia and Hypomagnesemia
Urinary Chloride Correction of hypokalemia and hypomagnesemia (which is felt to
The majority of metabolic alkalosis seen in the clinical setting tends play a role by contributing to hypokalemia) should also be done in
to be characterized by lower urinary chloride and results from gas- order to treat metabolic alkalosis, because they play a part in the
trointestinal (GI) losses or diuretics. In the setting of normal renal maintenance phase of metabolic alkalosis. The mechanism by which
function, management of the underlying disorder or adjustment of hypokalemia contributes to the maintenance of alkalosis is discussed
medication is important. Normal saline provides not only an acid above in Hypokalemia.
Nephrology Self-Assessment Program - Vol 23, No 2, June 2024 145
Correction of Hypercalcemia The main adverse event noted in the meta-analysis was hypokale-
Hypercalcemia from a variety of causes often presents with meta- mia, which is not surprising, because kaliuresis does increase with
bolic alkalosis due to calcium-related natriuresis and subsequent vol- acetazolamide, so serum potassium and sodium should be moni-
ume contraction. This is most notably seen with milk alkali tored closely in therapy. There should be caution in the use of acet-
syndrome, which is characterized by a triad of hypercalcemia, meta- azolamide in patients with severe liver or kidney disease. Similar to
bolic alkalosis, and acute kidney injury resulting from excessive other diuretics, a sulfa allergy is a contradiction to use.
ingestion of calcium and an absorbable alkali. Historically, this has
been related to Sippy’s antacid regimen consisting primarily of milk
and a combination of magnesium carbonate, bismuth subcarbonate, Nonconventional Management of Metabolic Alkalosis
and sodium bicarbonate. There has been a resurgence in milk alkali Standard therapy options may be limited by severe volume overload
syndrome due to increased calcium carbonate intake for osteoporosis or severe organ dysfunction, like in decompensated cirrhosis and heart
management. The risk is further compounded by vitamin D supple- failure or severe renal impairment. As such, renal replacement therapy
mentation. However, current milk alkali syndrome presents with a becomes the next best line of treatment. The reduced concentration
low phosphorus because of lack of exposure to phosphorus in dairy of HCO32 within the effluent and ability to adjust ultrafiltration
products. Management involves mainly fluid along with other medi- (UF) make it a useful tool for refractory metabolic alkalosis. Normal
cation interventions for hypercalcemia when needed. saline can be administered prefilter for further management of alkalo-
sis and the added volume can be accounted for with an adjustment of
UF which may have an additional convective advantage (46). Contin-
Acetazolamide for Management of uous renal replacement therapy has been used to manage metabolic
Metabolic Alkalosis alkalosis by using a low-HCO32 solution. Standard baths tend to
Heart failure with metabolic alkalosis, in addition to complications have a HCO32 concentration of 30 mEq per L; however, low-
listed in Table 1, has been shown to worsen mortality. Acetazolamide HCO32 baths can vary from 22 to 24 mEq per L. A change in bath
is used for patients with persistent extracellular volume overload with improved the serum HCO-2 and pH within 24 hours (47). There is
worsening metabolic alkalosis with or without the continuation of a concern about the use of citrate and its contribution to worsening
other loop or thiazide diuretics. The addition of acetazolamide to metabolic alkalosis due to its metabolism through the TCA cycle pri-
loop diuretics has been shown to improve heart failure, with increased marily in the skeletal muscle and liver, leading to the formation of
total urine output and natriuresis over just loop diuretics, with no bicarbonate. However, a recent meta-analysis did not show any differ-
worsening of adverse events; however, the addition of acetazolamide ence between the groups of citrate and noncitrate when it came to
did not change mortality or rehospitalization (42). There was a more metabolic alkalosis. Citrate acts as a weak acid in the blood and only
rapid correction with intravenous (iv)t versus oral acetazolamide; how- contributes to the alkali load if it is metabolized. In addition, a signifi-
ever, at 48 hours, there was no difference in systemic HCO32 values. cant portion of citrate gets removed by the filter and therefore would
The dosing for acetazolamide for correction of the alkalosis was deter- not contribute to the systemic alkali load (48).
mined by the amount of loop diuretics needed (43). Where dialysis cannot be or should not be performed, intrave-
Posthypercapnia metabolic alkalosis (PHA) results from the nous (iv) hydrochloric acid has been used successfully for manage-
rapid improvement of hypercarbia with improved ventilation but a ment of metabolic alkalosis (49); however, it does require central
delayed resolution of the renal compensation. The suppression of venous access and close monitoring. There are significant complica-
respiratory drive by the persistence of metabolic alkalosis after man- tions associated with this therapy, including central venous catheter
agement of respiratory acidosis has been a concern and a target for erosion, sclerosing phlebitis, and extravasation with either local tissue
several trials, because observational data have shown that the pres- necrosis or mediastinitis (50). Calculation of the base excess is done
ence of alkalosis leads to longer days in the ICU and ventilator by subtracting the desired HCO32 concentration from the actual
dependency (33). This has been more important lately because of HCO32 level and multiplying the difference by the “HCO32
presence of severe respiratory acidosis in the setting of coronavirus space.” The latter has been estimated to be as low as 20% or as high
disease 2019, with about 33% of patients having metabolic alkalosis as 50% of the body weight in kilograms. The value of the HCO32
and higher prevalence of PHA (44). The use of acetazolamide for excess calculated gives the amount of HCI solution in millimoles
management of metabolic alkalosis subsequent to PHA has been required to lower the HCO32 concentration to the desired value. It
evaluated in several trials and a meta-analysis but has shown mini- is not necessary to correct the HCO32 excess abruptly; it can be
mal benefit and discrepant outcomes. A recent meta-analysis revis- done over 12–24 hours. It is recommended that the infusion rate of
ited this issue and showed no improvement in patient-centric the HCI solution should not exceed 0.2 mmol per kg per hour or
outcomes with improvement in mortality, decreased ventilator or 300 mmoles per day (51). Frequent monitoring should take place
ICU time, or decreased hospital stay. The authors highlighted that even if dosing is calculated using the base excess formula, because it
the complexity and heterogeneity of the patient population made it has been shown that this formula is not very accurate in calculating
difficult to compare patient outcomes across the four trials identified the actual dose needed and preventing acidosis.
in this meta-analysis. Additionally, the randomized clinical trials Alternatively, ammonium chloride can be used for correction but
chosen were small, and adverse effect data were unable to be pooled is contraindicated in renal or hepatic failure, which severely limits its
(45). Because of numerous trials and a meta-analysis showing ques- use. Rapid iv infusion of ammonium chloride has been associated
tionable benefits, the use of acetazolamide for management in this with central nervous system toxicity that includes encephalopathy,
situation is left largely to the discretion of the treating physician. irritability, seizures, and coma.
146 Nephrology Self-Assessment Program - Vol 23, No 2, June 2024
Management in the Setting of Elevated Urine and five successive dialysis sessions for the correction of alkalosis,
uncontrolled hypertension, and volume overload. One feature of his
Chloride and Hypervolemia
presentation was recurrent preceding episodes of unexplained large
For patients with an elevated urine chloride where the blood pres-
interdialytic weight gains despite aggressive ultrafiltration (UF) (54).
sure is elevated and the patient does not appear volume depleted,
Management by hemodialysis involves adjustment of the HCO32
the cause of the phenotype of mineralocorticoid excess needs to be
bath, although there are no official guidelines for upper limit of pre-
identified to choose an appropriate medical therapy. This may be
dialysis serum HCO32 (55). In addition, normal saline can be given
pharmacological or surgical when removal of the offending organ is
at dialysis to help normalize the pH. In patients with PD, effluent
feasible; such as in primary hyperaldosteronism with lateralization to
bags with lower lactate concentrations should be considered, or in
an adrenal gland, or evidence of an ACTH-producing tumor and
the case of infants, to avoid using lower size-adjusted daily dialysate
resultant primary Cushing syndrome. Spironolactone may be indi-
exchange volume. Evaluating and stopping secondary causes as dis-
cated for primary hyperaldosteronism, although the use of spirono-
cussed above is also important in either case.
lactone can exacerbate the problem in patients with Geller
syndrome, where the chemical structure of spironolactone is very
similar to progesterone and causes a worsening of hypertension and
metabolic alkalosis due to a defective mineralocorticoid receptor that 4 Prompt management of alkalemia is important,
gets activated in response to progesterone (52). Amiloride could be because it is associated with central nervous sys-
required for Liddle syndrome. The use of dexamethasone can be
tem depression and mortality. In evaluation of
appropriate in cases such as glucocorticoid remedial hypertension.
Screening for licorice use is important. Working through this differ- metabolic alkalosis, volume and chloride assess-
ential is key for starting the appropriate therapy and getting appro- ment play a major role, because correction of
priate outcomes. chloride is key and may be successful even in the
presence of other electrolyte imbalance. Volume
Management of Metabolic Alkalosis in ESKD expansion will also help ameliorate physiologic
In ESKD patients, an elevation of serum HCO32 is most likely renin-angiotensin-aldosterone system activation.
metabolic alkalosis, because any compensation for respiratory acido-
When not physiologic, further therapy is directed
sis would be either not possible or severely impaired. Despite the
notion that metabolic acidosis is the predominant acid-base disorder toward aldosterone inhibition and hypokalemia
in ESKD patients on dialysis, there is a cohort of patients that do correction, because both promote alkalosis.
develop metabolic alkalosis. A retrospective case-controlled study
showed that about a third of patients had interdialytic alkalosis,
which was defined as a serum HCO32 of greater than 26 for .7
months of the first 12-month period. Approximately 17% of those
patients that were alkalotic had persistent interdialytic alkalosis, References
which was defined as persistent alkalosis in a majority of months of 1. Hodgkin JE, Soeprono FF, Chan DM: Incidence of metabolic alkalemia
each subsequent 12-month period or fraction thereof. In persistently in hospitalized patients. Crit Care Med 8: 725–728, 1980 PubMed
alkalotic patients, there was a much greater incidence of cardiac 2. Mæhle K, Haug B, Flaatten H, Nielsen E: Metabolic alkalosis is the
arrhythmias and a greater intradialytic blood pressure decrease. It most common acid-base disorder in ICU patients. Crit Care 18: 420,
was noted that elderly patients were at risk, but gender, race, starting 2014 PubMed
3. Okusawa S, Aikawa N, Abe O: Postoperative metabolic alkalosis follow-
weight comorbidities, and mortality did not differ. There was a 7%
ing general surgery: Its incidence and possible etiology. Jpn J Surg 19:
greater dialysis dose, lower protein catabolic rate, and lower intradia- 312–318, 1989 PubMed
lytic weight gain in persistent alkalosis. Previous observational stud- 4. Wilson RF, Gibson D, Percinel AK, Ali MA, Baker G, LeBlanc LP, et al.:
ies show a correlation between both predialysis HCO32 levels and Severe alkalosis in critically ill surgical patients. Arch Surg 105: 197–203,
dialysate HCO32 levels and increased mortality, cardiovascular hos- 1972 PubMed
pitalizations, and intradialytic hypotension. 5. Anderson LE, Henrich WL: Alkalemia-associated morbidity and mortality
In peritoneal dialysis (PD), there was a higher incidence of met- in medical and surgical patients. South Med J 80: 729–733, 1987 PubMed
abolic alkalosis in the pediatric population, which was attributed to 6. Hamm LL, Nakhoul N, Hering-Smith KS: Acid-base homeostasis. Clin J
Am Soc Nephrol 10: 2232–2242, 2015 PubMed
high lactate concentrations within the PD fluid (53). 7. Tinawi M: Pathophysiology, evaluation, and management of metabolic
Excessive alkali administration or gastrointestinal (GI) causes are alkalosis. Cureus 13: e12841, 2021 PubMed
the only way to develop metabolic alkalosis in these patients. The pri- 8. Park M, Sidebotham D: Metabolic alkalosis and mixed acid-base disturbance
mary mechanism is from exposure to excessive HCO32 or citrate in anaesthesia and critical care. BJA Educ 23: 128–135, 2023 PubMed
within the effluent. However, other forms of alkali administration can 9. Emmett M: Metabolic alkalosis: A brief pathophysiologic review. Clin J
be specific to patients with ESKD, including medications such as cal- Am Soc Nephrol 15: 1848–1856, 2020 PubMed
10. Do C, Vasquez PC, Soleimani M: Metabolic alkalosis pathogenesis, diag-
cium carbonate, sodium bicarbonate (baking soda), sevelamer carbon-
nosis, and treatment: Core curriculum 2022. Am J Kidney Dis 80: 536–
ate, citrate, and acetate. Massive transfusions and crack cocaine use 551, 2022 PubMed
should be evaluated in appropriate patients as well. There is a case 11. Felder RA, Jose PA, Xu P, Gildea JJ: The renal sodium bicarbonate
report of severe alkalemia pH.7.71 due to sodium bicarbonate pica cotransporter NBCe2: Is it a major contributor to sodium and pH
intake in an ESKD patient requiring intubation for respiratory failure homeostasis? Curr Hypertens Rep 18: 71, 2016 PubMed
Nephrology Self-Assessment Program - Vol 23, No 2, June 2024 147
12. Luke RG, Galla JH: It is chloride depletion alkalosis, not contraction 34. Aronson PS, Giebisch G: Effects of pH on potassium: New explanations
alkalosis. J Am Soc Nephrol 23: 204–207, 2012 PubMed for old observations. J Am Soc Nephrol 22: 1981–1989, 2011 PubMed
13. Zacchia M, Capolongo G, Rinaldi L, Capasso G: The importance of the 35. Tizianello A, Garibotto G, Robaudo C, Saffioti S, Pontremoli R, Bruz-
thick ascending limb of Henle’s loop in renal physiology and pathophysi- zone M, et al.: Renal ammoniagenesis in humans with chronic potassium
ology. Int J Nephrol Renovasc Dis 11: 81–92, 2018 PubMed depletion. Kidney Int 40: 772–778, 1991 PubMed
14. Wagner CA: Pendrin—A new target for diuretic therapy? J Am Soc 36. Han KH: Mechanisms of the effects of acidosis and hypokalemia on renal
Nephrol 27: 3499–3501, 2016 PubMed ammonia metabolism. Electrolyte Blood Press 9: 45–49, 2011 PubMed
15. Tahaei E, Pham TD, Al-Qusairi L, Grimm R, Wall SM, Welling PA: 37. Xu N, Hirohama D, Ishizawa K, Chang WX, Shimosawa T, Fujita T,
Pendrin regulation is prioritized by anion in high-potassium diets. Am J et al.: Hypokalemia and pendrin induction by aldosterone. Hypertension
Physiol Renal Physiol 324: F256–F266, 2023 PubMed 69: 855–862, 2017 PubMed
16. Loffing J, Pech V, Loffing-Cueni D, Abood DC, Kim YH, Chen C, 38. Garry P, Tormey WP, Harte M: Arterial blood pH of 7.95 in an adult
et al.: Pendrin abundance, subcellular distribution, and function are with pyloric stenosis. Clin Chem 39: 1554–1555, 1993 PubMed
unaffected by either aENaC gene ablation or by increasing ENaC chan- 39. Achanti A, Szerlip HM: Acid-base disorders in the critically ill patient.
nel activity. Pflugers Arch 475: 607–620, 2023 PubMed Clin J Am Soc Nephrol 18: 102–112, 2023 PubMed
17. Chambrey R, Eladari D: Novel insight on physiological regulation of the 40. Vaziri ND, Byrne C, Barton CH, Robertson D, Jay K, McDonald L:
Cl2/HCO3 exchanger pendrin. Am J Physiol Renal Physiol 324: F431– Prevention of metabolic alkalosis induced by gastric fluid loss using H2
F432, 2023 PubMed receptor antagonist. Gen Pharmacol 16: 141–144, 1985 PubMed
18. Kandasamy N, Fugazzola L, Evans M, Chatterjee K, Karet F: Life-threat- 41. Aledan H, Rasheed J, Jasim Z, Razak MA: Treatment of metabolic alka-
ening metabolic alkalosis in Pendred syndrome. Eur J Endocrinol 165: losis with rice broth. Curr Res Nutr Food Sci 9: 588–596, 2021
167–170, 2011 PubMed 42. Mullens W, Dauw J, Martens P, Verbrugge FH, Nijst P, Meekers E,
19. Mantoo MR, Kabra M, Kabra SK: Cystic fibrosis presenting as pseudo- ADVOR Study Group, et al.: Acetazolamide in acute decompensated heart
Bartter syndrome: An important diagnosis that is missed! Indian J Pediatr failure with volume overload. N Engl J Med 387: 1185–1195, 2022 PubMed
87: 726–732, 2020 PubMed 43. Addison JD, Peterson EJ, Meyenburg L: Intravenous or oral acetazol-
20. Terlizzi V, Padoan R, Leonetti G, Vitullo P, Tosco A, Taccetti G, et al.: amide for treatment of diuretic-induced alkalosis in patients with heart
Cystic fibrosis and CFTR-related disorder with electrolyte imbalance at failure. Ann Pharmacother 57: 1241–1247, 2023 PubMed
diagnosis: Clinical features and outcome in an Italian cohort. Eur J 44. Alfano G, Ferrari A, Fontana F, Mori G, Magistroni R, Guaraldi G,
Pediatr 182: 5275–5283, 2023 PubMed et al.: FC 025: Acid base disorders in COVID-19. Nephrol Dial Trans-
21. Berg P, Svendsen SL, Hoang TTL, Praetorius HA, Sorensen MV, Leipzi- plant 36: gfab145.001, 2021 PubMed
ger J: Impaired renal HCO3- secretion in CFTR deficient mice causes 45. Bemand TJ, Chatoor R, Natale P, Strippoli G, Delaney A: Acetazol-
metabolic alkalosis during chronic base-loading. Acta Physiol (Oxf) 231: amide for metabolic alkalosis complicating respiratory failure with
e13591, 2021 PubMed chronic obstructive pulmonary disease or obesity hypoventilation syn-
22. Berg P, Svendsen SL, Sorensen MV, Larsen CK, Andersen JF, Jensen- drome: A systematic review. Thorax 78: 1004–1010, 2023 PubMed
Fangel S, et al.: Impaired renal HCO3- excretion in cystic fibrosis. J Am 46. Mohammedzein A, Naguib T: Case in point: Correction of dialysis-
Soc Nephrol 31: 1711–1727, 2020 PubMed induced metabolic alkalosis. Fed Pract 38: 190–194, 2021 PubMed
23. Hegyi P, Seidler U, Kunzelmann K: CFTR-beyond the airways: Recent 47. K€oglberger P, Klein SJ, Lehner GF, Bellmann R, Peer A, Schw€arzler D,
findings on the role of the CFTR channel in the pancreas, the intestine et al.: Low bicarbonate replacement fluid normalizes metabolic alkalosis
and the kidneys. J Cyst Fibros 22 Suppl 1: S17–S22, 2023 PubMed during continuous veno-venous hemofiltration with regional citrate antic-
24. Spoletini G, Fitch G, Gillgrass L, Etherington C, Clifton I, Peckham oagulation. Ann Intensive Care 11: 62, 2021 PubMed
DG: Urinary bicarbonate and metabolic alkalosis during exacerbations in 48. Jacobs R, Verbrugghe W, Dams K, Roelant E, Couttenye MM, Devroey
cystic fibrosis. ERJ Open Res 8: 00669–02021, 2022 PubMed D, et al.: Regional citrate anticoagulation in continuous renal replace-
25. Berg P, Svendsen SL, Ayasse N, Sorensen MV, Leipziger J: Secretin: A hor- ment therapy: Is metabolic fear the enemy of logic? A systematic review
mone for HCO3- homeostasis. Pflugers Arch 476: 545–554, 2024 PubMed and meta-analysis of randomised controlled trials. Life (Basel) 13: 1198,
26. Berg P, Jensen T, Andersen JF, Svendsen SL, Modvig IM, Wang T, et al.: 2023 PubMed
Loss of the secretin receptor impairs renal bicarbonate excretion and 49. Guffey JD, Haas CE, Crowley A, Connor KA, Kaufman DC: Hydrochlo-
aggravates metabolic alkalosis in mice during acute base-loading. J Am ric acid infusion for the treatment of metabolic alkalosis in surgical inten-
Soc Nephrol 34: 1329–1342, 2023 PubMed sive care unit patients. Ann Pharmacother 52: 522–526, 2018 PubMed
27. Berg P, Svendsen SL, Sorensen MV, Schreiber R, Kunzelmann K, Leipzi- 50. Buchanan IB, Campbell BT, Peck MD, Cairns BA: Chest wall necrosis
ger J: The molecular mechanism of CFTR- and secretin-dependent renal and death secondary to hydrochloric acid infusion for metabolic alkalosis.
bicarbonate excretion. J Physiol 599: 3003–3011, 2021 PubMed South Med J 98: 822–824, 2005 PubMed
28. Weiner ID, Verlander JW: Renal ammonia metabolism and transport. 51. Bazilinski NG, Dunea G, Ing TS: Treatment of metabolic alkalosis in
Compr Physiol 3: 201–220, 2013 PubMed renal failure. Int J Artif Organs 10: 284–286, 1987 PubMed
29. Weiner ID: Roles of renal ammonia metabolism other than in acid-base 52. Geller DS, Farhi A, Pinkerton N, Fradley M, Moritz M, Spitzer A, et al.:
homeostasis. Pediatr Nephrol 32: 933–942, 2017 PubMed Activating mineralocorticoid receptor mutation in hypertension exacer-
30. Harris AN, Skankar M, Melanmed M, Batlle D: An update on kidney bated by pregnancy. Science 289: 119–123, 2000 PubMed
ammonium transport along the nephron. Adv Kidney Dis Health 30: 53. Tzvi-Behr S, Bnaya A, Becker-Cohen R, Rinat C, Weinbrand-Goichberg
189–196, 2023 PubMed J, Choshen S, et al.: Metabolic alkalosis in infants treated with peritoneal
31. Krishnan N, Alpern RJ: Metabolic alkalosis. Nephsap 20: 145–159, 2022 dialysis. Pediatr Nephrol 37: 1889–1895, 2022 PubMed
32. Yi Y: Post-hypercapnic alkalosis: A brief review. Electrolyte Blood Press 21: 54. Sarwal A, Kim S, Gnanasekaran I: Life-threatening alkalosis from baking
18–23, 2023 PubMed soda pica in an end stage renal disease hemodialysis patient. Hemodial Int
33. Wall SM: Regulation of blood pressure and salt balance by pendrin- 26: E16–E18, 2022 PubMed
positive intercalated cells: Donald Seldin Lecture 2020. Hypertension 79: 55. Mudunuru SA, Navarrete J, O’Neill WC: Metabolic alkalosis in hemodi-
706–716, 2022 PubMed alysis patients. Semin Dial 36: 24–28, 2023 PubMed
148 Nephrology Self-Assessment Program - Vol 23, No 2, June 2024
Self-Assessment Questions
osmolality 240 mOsm/kg H2O, urine osmolality 90 B. Improved likelihood of neurologic and functional
mOsm/kg H2O, and urinary sodium 10 mmol/L recovery at discharge
Upon admission to the intensive care unit, he was C. Decrease in intracranial pressure
given 100 ml of 3% NS. Urine output increased to 900 D. Acute kidney injury
ml/h for 3 hours after 3% NS administration.
8. A 64-year-old woman with a past medical history of
Which of the following is a risk factor for osmotic cholelithiasis and hypertension is admitted to the
demyelination? medical intensive care unit for septic shock. A contrast
computed tomography of the abdomen and pelvis is
A. Hypertension
suggestive of bile duct dilatation suspicious for stones
B. Hypokalemia
in the common bile duct, which is confirmed in the
C. Receiving iv D5W
magnetic resonance cholangiopancreatography. She
D. Older age
receives fluid resuscitation with 5 L of isotonic fluids
and is started on norepinephrine infusion through a
Article 2: Hypernatremia: Pathophysiology, central venous catheter. She has anuric acute kidney
Evaluation, Treatment, and Updates injury but has not received any renal replacement ther-
6. A 30-year-old pregnant woman at 32 weeks of gestation apy. An endoscopic retrograde cholangiopancreatogra-
is referred by her obstetrician to the nephrology clinic phy is performed with extraction of common bile duct
stones, and she is on broad-spectrum antibiotics. Her
because she has been complaining of excessive thirst and
hemodynamic status improves over the next 2 days,
frequent urination for the past several weeks. She has
and she is weaned off the norepinephrine infusion. She
been drinking five to six bottles of water every day and
has 21 pitting presacral edema on examination. Her
has to use the restroom to urinate every 2 hours. A basic
serum creatinine level is currently 4.2 mg/dl, and her
metabolic panel sent 2 days before the visit reveals that
baseline serum creatinine before hospitalization was 0.8
her plasma sodium concentration is 146 mEq/L, her mg/dl. Her serum sodium concentration is 151 mEq/L.
serum creatinine levels are 0.6 mg/dl, her serum blood
urea nitrogen concentration is 14 mg/dl, and her serum Which of the following best explains the cause of her
glucose concentration is 146 mg/dl. Her pregnancy has hypernatremia?
been complicated by gestational diabetes mellitus, and
A. Total body free-water deficit due to lack of access
she is currently on insulin therapy. Before the pregnancy,
to water
she had a history of drinking three to four bottles of
B. Total body sodium and water gain with tonicity
water every day and urinating five times a day; however, imbalance between fluid losses and fluid gains
she did not have a prior diagnosis of diabetes mellitus. C. Decreased metabolic water production from sepsis-
Urine tests revealed urine osmolality of 220 mOsm/kg, induced mitochondrial dysfunction
urine sodium of 35 mEq/L, and 24-hour total collec- D. Vasopressin resistance
tion of 3.5 L of urine.
9. A 35-year-old man with a past medical history of end-
What is the most likely cause of her polyuria? stage renal disease on hemodialysis is admitted to the
A. Primary polydipsia neurosurgical intensive care unit after a motor vehicle
B. Gestational diabetes mellitus with osmotic diuresis crash. Computed tomography scan of the head shows
C. Intracranial space occupying lesion evidence of right-sided subdural hematoma and cere-
D. Vasopressinase secretion with underlying partial bral edema with 3-mm midline shift. He is also hyper-
vasopressin disorder kalemic to a potassium concentration of 6.4 mEq/L.
Nephrology is consulted, and a decision is made to
7. A 58-year-old man with no past medical history fell initiate continuous venovenous hemofiltration (CVVH)
while climbing the stairs and was brought to the emer- while also maintaining permissive hypernatremia to
gency department. He was intubated for airway pro- 150 mEq/L through 3% hypertonic saline infusion
tection, and a computed tomography scan of the head through a central venous catheter. The calculated
revealed left-sided large intraparenchymal cerebral replacement fluid rate is 2200 ml/h, which will be
hemorrhage with a 5-mm midline shift. He is given a administered prefilter, and the blood flow rate is
23.4% hypertonic saline bolus and started on mainte- 230 ml/min with no net ultrafiltration. His current
nance 3% hypertonic saline solution for a target plasma plasma sodium is 140 mEq/L, and the sodium concen-
sodium concentration of 150 mEq/L. tration in the replacement fluid is also 140 mEq/L.
Which of the following statements is the most likely While starting CVVH, which of the following is the
outcome of targeting a plasma sodium concentration recommended infusion rate for the 3% hypertonic
of 150 mEq/L in this patient? saline?
A. Improved likelihood of 6-month survival A. 30 ml/h
150 Nephrology Self-Assessment Program - Vol 23, No 2, June 2024
What is the most likely explanation for this patient’s What is the most likely mechanism leading to this
hypokalemia? patient’s hypokalemia?
A. Hypokalemia due to a low potassium diet over a A. Loss of potassium via gastric fluid in combination
prolonged period of time with a low potassium intake
B. Osteoporosis-related hypokalemia due to high bone B. The combination of increased sodium bicarbonate
turnover delivery to the collecting duct and increased vol-
C. The combination of hypokalemia, hypertension, ume depletion–induced aldosterone activity
and alkalemia direct toward Liddle syndrome as C. Bulimia nervosa is the underlying problem: the
the cause of this patients hypokalemia patient consumes large amounts of laxatives and
D. Thiazide diuretic–associated hypokalemia diuretics
E. Hidden laxative abuse D. Potassium is shifted intracellularly because of
hyperventilation, explaining hypokalemia
12. In the case presented above, an 84-year-old woman E. The patient is in distress, causing a hyperadrener-
presents with potassium 2.9 mEg/L in the setting of gic state, which, in turn, causes hypokalemia
symptoms of progressive weakness and malaise; she
has a past history of arterial hypertension and osteo- 14. In the case presented above, a 29-year-old man pre-
porosis and medications including calcium, 25OH sents with a potassium of 2.4 mEq/L in the setting of
Nephrology Self-Assessment Program - Vol 23, No 2, June 2024 151
nonbloody emesis for .48 hours. He is otherwise 26%, urine chloride ,10 mEq/L, urine pH 7.2. Stool
healthy and does not take any medication on a regular pH ,5; stool chloride 90 mEq/L.
basis. The patient has a BMI of 18 kg/m2, a blood
pressure of 98/52 mm Hg, a pulse of 104/min, and a What is the most likely reason for hypokalemia?
respiratory rate of 18, and is afebrile. Clinical examina- A. Laxative abuse
tion is unremarkable. B. Surreptitious vomiting
In addition to very low potassium, laboratory investi- C. Periodic hypokalemic paralysis
gations reveal sodium of 135 mEq/L and serum creati- D. Congenital chloride diarrhea
nine of 1.16 mg/dl (102 mmol/L). Venous blood gas E. Gitelman syndrome
analysis reveals a bicarbonate level of 38 mEq/L.
A 12-lead electrocardiogram is performed in this patient;
it shows widespread depression of the ST segment as Article 4: Updates in the Risk and Treatment of
well and presence of U waves.
Hyperkalemia in CKD Patients
What is your first step for further management of the 17. You are consulted on a 19-year-old man who presents
patient? with blood pressure 149/88 mm Hg. He describes
A. Discharge home with oral potassium supplemen- early-onset hypertension in his mother and two mater-
tation with potassium chloride 10 mEq three times nal uncles and his grandfather. He is mildly volume
daily in combination with an oral antiemetic overloaded on exam. Labs are remarkable for Na 136,
B. Monitor the patient with continuous electrocardio- K 6.0, Cl 105, and CO2 18 mEq/L.
gram monitoring and start intravenous potassium
supplementation with potassium chloride What antihypertensive agent is most likely to be
C. Potassium supplementation should not be started effective?
until results for serum magnesium levels are A. Losartan
available B. Chlorthalidone
D. Start the patient on iv spironolactone to counteract C. Metoprolol
ongoing renal potassium loss D. Spironolactone
E. Administer intravenous HCl via a central line to
correct metabolic alkalosis 18. You are consulted on a 49-year-old man with persistent
hyperkalemia. He has stage 3b with eGFR 32 ml/min
15. An 18-year-old woman presents to the renal outpatient per 1.73 m2 in the setting of longstanding diabetes,
department for metabolic evaluation after an episode of hypertension, and, more recently, heart failure with
nephrolithiasis with calcium phosphate kidney stones reduced ejection fraction. Laboratory values show serum
and nephrocalcinosis. The patient recently received a potassium that ranges from 4.8 to 5.9 mEq/L, and he
cochlear implant for hearing loss. The physical examina- has had multiple ER visits because of hyperkalemia. His
tion was unremarkable, except for low blood pressure urine albumin/creatinine ratio is 1200 mg/g. Medica-
(95/65 mm Hg). Laboratory findings: venous blood tions include Entresto, spironolactone, and carvedilol.
gas analysis: pH 7.32, Base Excess 28.2 mEq/L, K
3.2 mEq/L, Cl 116 mEq/L, Na 141 mEq/L; fractional Which maneuver is most likely to improve clinical
excretion of potassium: 23%, urine calcium excretion 11 outcome?
mEq/D, urine pH 7.0. A. Discontinue Entresto to prevent hyperkalemia
B. Discontinue spironolactone to prevent hyperkalemia
What is the most likely reason for hypokalemia? C. Add empagliflozin and a loop diuretic
A. Hereditary distal renal tubular acidosis (RTA) D. Provide a list of high-potassium foods to avoid
B. Ureteral diversion
C. Ketoacidosis 19. You are consulted on a patient for CKD and hyperka-
D. Laxative abuse lemia. The patient is a 64-year-old man with CKD
stage 4 with eGFR 18 ml/min per 1.73 m2 secondary
16. A 19-year-old healthy man with intermittent episodes to type 2 diabetes with urine protein/creatinine ratio
of moderate hypokalemia is presenting in the nephrol- of 1.5 g despite ACE inhibitor. He is hypertensive and
ogy outpatient department for further evaluation. He appears volume overloaded on exam. Laboratory values
does not complain of any symptoms (i.e., pain). After are notable for K 5.9 mEq/L, CO2 18 mEq/L.
repeated questions, he mentions that he has had pulpy
stools four to five times per day, for as long as he can Which of the following maneuvers is likely to control
remember. Laboratory findings: blood gas analysis: his potassium and improve outcome?
serum K: 2.6 mEq/L, pH 7.5, HCO3 37.5 mmol/L, A. Add a loop diuretic
pCO2 49.3 mm Hg. Fractional excretion of potassium B. Add sodium bicarbonate
152 Nephrology Self-Assessment Program - Vol 23, No 2, June 2024
C. Add loop diuretic 1 sodium bicarbonate with presented with episodes of confusion, slurred speech,
addition of sodium zirconium cyclosilicate 10 g and ataxia worsening over the past week. He reported
one or two times weekly increased intake of high-carbohydrate meals recently.
D. Add sodium zirconium cyclosilicate and titrate to His vital signs were stable, but he appeared disor-
15 g daily iented. Initial laboratory investigations revealed the
following: sodium 140 mEq/L, potassium 4.0 mEq/L,
Article 5: Metabolic Acidosis chloride 102 mEq/L, bicarbonate 15 mEq/L, glucose
90 mg/dl, urea nitrogen 9 mg/dl; and creatinine
20. A 72-year-old woman presented with a 2-day history
of high fevers, a productive cough with yellow spu- 0.9 mg/dl, and albumin 4.4 g/dl. Arterial blood gas anal-
tum, and increasing shortness of breath. Her vital ysis showed pH 7.28, pCO2 28 mm Hg, pO2 99 mm Hg,
signs were notable for a blood pressure of 90/60 mm HCO3 13 mEq/L. Lactic acid was 1.0 mmol/L. Urinal-
Hg and a pulse of 130/min. Initial laboratory investi- ysis was unremarkable. Ethanol level was negative.
gations revealed the following: sodium 138 mEq/L, Measured serum osmolarity was 290 mOsm/L.
potassium 3.5 mEq/L, chloride 111 mEq/L, bicarbon-
ate 17 mEq/L, glucose 133 mg/dl, blood urea nitrogen What is the next diagnostic work-up to confirm the
29 mg/dl, creatinine 1.2 mg/dl, and albumin 2.1 g/dl. diagnosis in this patient?
An arterial blood gas analysis showed the following: A. Urine sediment examination
pH 7.30, pO2 of 75 mm Hg, pCO2 31 mm Hg, HCO3 B. Liver function tests
15 mEq/L. C. Serum D-lactate measurement
D. Volatile alcohol panel
On the basis of the patient’s clinical presentation
and laboratory findings, which of the following acid- 23. A 76-year-old woman with a medical history signifi-
base abnormalities exist in this patient? cant for hypertension, lung cancer status post chemo-
A. Mixed metabolic acidosis and respiratory alkalosis therapy and radiation, and chronic pain presented
B. Primary respiratory alkalosis with metabolic with a 3-day history of progressive lethargy, confu-
compensation sion, and dyspnea. She has taken acetaminophen 650
C. Hyperchloremic (normal anion gap) metabolic mg four times daily and amlodipine 10 mg daily for
acidosis the past several years. Her social history was notable
D. High anion gap metabolic acidosis (HAGMA) with for moderate, consistent alcohol consumption and for-
respiratory compensation mer tobacco use. On examination, she was notably mal-
nourished, with a BMI of 17 kg/m2, and demonstrated
21. A 50-year-old man with a history of type 2 diabetes pre- tachypnea with labored respirations and altered mental
sented with 4 days of malaise, fatigue, and decreased status. Arterial blood gas analysis revealed a pH of 7.21,
exercise capacity. His medications included empagliflo- pCO2 of 21 mm Hg, pO2 of 105 mm Hg, and bicarbon-
zin 25 mg daily, metformin 1000 mg twice a day, and ate level of 8 mEq/L. Urinalysis was negative for
baby aspirin 81 mg daily for the past 2 years. He ketones. Additional laboratory tests showed serum
recently started a ketogenic diet, with reduced carbohy- sodium 140 mEq/L, potassium 3.8 mEq/L, chloride
drate intake. On physical exam, his blood pressure was 106 mEq/L, bicarbonate 10 mEq/L, BUN 20 mg/dl,
120/80 mm Hg with a pulse of 120. He was uncomfort- creatinine 1.8 mg/dl, and glucose 92 mg/dl. Her
able, with epigastric tenderness. Labs showed Na albumin was 3.0 g/dl, lactate was 0.5 mmol/L, and
132 mEq/L, K 5.3 mEq/L, serum HCO3 9 mEq/L, Cl measured serum osmolality was 290 mOsm/L.
99 mEq/L, glucose 165 mg/dl, and creatinine 0.8 b-Hydroxybutyrate was undetectable.
mg/dl. Serum albumin was 4.2 g/dl. Urine dipstick
showed 31 ketones and 31 glucose. ABGs showed a What is the MOST likely cause of the patient’s
pH of 7.20, pCO2 20 mm Hg, pO2 99 mm Hg, and high anion gap metabolic acidosis?
HCO3 8 mEq/L. Lactic acid was 1.2 mmol/L, ethanol A. Ethylene glycol ingestion
level was ,10 mg/dl, salicylate level was undetectable, B. 5-Oxoproline (pyroglutamic acid)
and b-hydroxybutyrate was 9.89 mmol/L. C. Alcoholic ketoacidosis
D. Lactic acidosis
What is the most likely diagnosis?
A. Metformin-associated lactic acidosis 24. A 68-year-old woman with a longstanding history of
B. Euglycemic DKA induced by an SGLT2 inhibitor Sj€
ogren disease and hypertension presented with persis-
C. Alcoholic ketoacidosis tent weakness and polyuria. She was taking amlodipine
D. Salicylate toxicity for hypertension. She also had some mild diarrhea in
the past month. On examination, she was mildly dehy-
22. A 45-year-old man with a history of short bowel syn- drated. Laboratory tests showed a serum sodium of
drome due to extensive small bowel resection 140 mEq/L, potassium 2.5 mEq/L, chloride 110 mEq/L,
Nephrology Self-Assessment Program - Vol 23, No 2, June 2024 153
bicarbonate 17 mEq/L, and creatinine of 1.1 mg/dl. The On the basis of these findings, what is the most
ABG was pH 7.32, pO2 95 mm Hg, pCO2 32 mm Hg, likely cause for this patient’s metabolic alkalosis?
HCO3 16 mEq/L. Her urine pH was 6.5, urine sodium A. Vomiting
78 mEq/L, urine potassium 30 mEq/L, urine chloride B. Bartter syndrome
68 mEq/L, urine urea nitrogen 800 mg/dl, and urine C. Milk alkali syndrome
osmolarity 550 mOsm/kg. Urinalysis was negative for D. Liddle syndrome (pseudohyperaldosteronism)
glucose, ketones, and protein. E. Nonabsorbable anion
What is the MOST likely cause of the patient’s hyper- 26. In the absence of an alkali load, a patient with which
chloremic metabolic acidosis? defect can still present with metabolic alkalosis?
A. Distal (type 1) RTA A. Pendrin mutation
B. Proximal (type 2) RTA B. CFTR mutation
C. Hyperkalemic (type 4) RTA C. Secretin receptor mutation
D. Gastrointestinal bicarbonate loss due to diarrhea D. Von Hippel Lindau gene mutation